Neurosurgery Review AK Mahapatra, Raj Kumar, Udit Singhal
INDEX
×
Chapter Notes

Save Clear


MCQs in Neurosurgery Review

3
  1. Impairment of optokinetic nystagmus is seen in lesion of the
    1. Frontal lobe
    2. Parietal lobe
    3. Occipital lobe
    4. Temporal lobe
Ans. B. Parietal lobe: It is a normal, physiologic phenomenon, sometimes affected by disease. OKN tape is used for examination. A patient with a hemianopsia and normal OKN response is more likely to have occipital lobe lesion, with vascular pathology being common with asymmetric OKN, the lesion is more likely to be nonvascular that is tumor. (Cogan's rule)
  1. Causes of hyperintensity on MRI TIWI are
    1. Very high protein content
    2. Melanin
    3. Hypercellularity with low cytoplasm as seen in medulloblastoma or pinealoblastoma
    4. Fat
    5. Edema
Ans. A to D. E is false
  • Causes of hyperintensity on TI are paramagnetic effect of hemorrhage (subacute or chronic), paramagnetic material melanin ion, very high protein content, fat (teratoma, dermoid lipoma) and in flow related enhancement in tumor vessels.
  1. In fundoscopy, how many vessels course over the disc margin?
    1. 8 to 12
    2. 12 to 14
    3. 15 to 20
    4. 20 to 25
Ans. A. i.e. 8 to 12 vessels course over the disc margin and the darker arterioles are approx 2/3rd the size of the light colored venules
  1. Paratonia/Gagenhalten is due to lesion in
    1. Frontal lobe
    2. Parietal lobe
    3. UMN lesion
    4. Extrapyramidal lesion4
Ans. A. Paratonia/Gagenhalten (fluctuating changes in resistance to repetitive passive movements) with frontal lobe or thalamic disorder
  1. Xanthochromic CSF not due to blood product but due to protein is when level is
    1. > 150 – 200 mg/dl
    2. > 200 – 250 mg/dl
    3. > 250 – 300 mg/dl
    4. > 350 – 400 mg/dl
Ans. A. i.e. [>1.5 to 2 g/L] (150–200 mg/dl)
  1. Posterior iliac crest corresponds to
    1. L2 – L3 interspace
    2. L3 – L4 interspace
    3. L4 – L5 interspace
    4. L5 – S1 interspace
Ans. B
  1. Headaches occurring after lumbar puncture occur in about
    1. 5 – 10%
    2. 10 – 20%
    3. 20 – 30%
    4. 30 – 40%
Ans. A. i.e. 5 – 10%. Such headaches typically begin 12 to 48 hrs after the procedure and may last from several days to 2 weeks. These headaches are positional in character
  1. Radiation level in CT per examination is
    1. 3 – 5 cGy
    2. 5 – 10 cGy
    3. 10 – 15 cGy
    4. 15 – 20 cGy
Ans. A. i.e. 3 – 5 cGy. CT is safe and reliable. Radiation levels are between 3 and 5cGy per examination. Most frequent complications are associated with use of contrast. Ionic and non-ionic are in use. A rise in serum creatinine of at least 1 mg/dl within 48 hr of administration is a definition of contrast nephropathy5
  1. Regarding MRI
    1. The T1 relaxation rate is the time for 63% of the protons to return to their normal equilibrium state
    2. The T2 relaxation rate is the time for 63% of the protons to become dephased owing to interaction among other protons
    3. Approx 5% of the population experience claustrophobea in the MR environment
    4. Unlike CT patient cooperation is important
    5. T2W images are more sensitive than T1W images to edema or myelin destruction
Ans. All are correct.
  • The rate of return to equilibrium of perturbed proton is called the relaxation rate. Rate is different for different, normal and pathologic tissues. T1W images produced by keeping TE and TR are relatively short. T2W image has long TR and TE. Fat and subacute hemorrhage has short T1, therefore having high intensity on T1WI.
  1. About intravenous MR contrast agent
    1. Gadolinium forms the basis of all current intravenous MR contrast agent
    2. Gadolinium is paramagnetic
    3. Reduces T1 and T2 relaxation time resulting in high signal on T1W image
    4. Approx 0.2 ml/kg body weight is administered IV (10 – 15 ml for the average sized adult)
    5. Hypersensitivity reactions are common
Ans. A to D are correct
  • E is false. It reduces the T1 and T2 relaxation time of nearby water protons resulting in high signal on T1W. Metal is chelated to an agent DTPA which allow renal excretion without toxicity. It does not cross a normal blood brain barrier. Allergic reactions extremely rare. Recently Gadolinium has been linked to the development of the nephrogenic systemic fibrosis (NSF) or nephrogenic fibrosing dermopathy(NFD). Occurred in patients with moderate to end stage renal disease6
  1. MRI is considered safe for patients at magnetic field level up to
    1. 2 T
    2. 3 T
    3. 4 T
    4. 5 T
Ans. B. MRI is considered safe for patients at magnetic field upto 3 T. Serious injuries have been caused by the high magnetic field used
  1. The radiation exposure during conventional myelography
    1. 04 to 8 cGy
    2. 08 to 12 cGy
    3. 12 to 16 cGy
    4. 16 to 20 cGy
Ans. A. Conventional myelography involves high concentration and volume of contrast material. Radiation exposure is 4 to 8 cGy- (one of the more radiation intense procedure). Gonads should be shielded if possible. CT myelography has replaced conventional myelography
  1. For cerebral angiography, the contrast is injected into
    1. Femoral artery
    2. Radial artery
    3. Common carotid
    4. Internal carotid
Ans. A. Femoral artery: The femoral route is used most commonly. Most feared complication is stroke. 4 percent risk for TIA and stroke 1 percent permanent deficit and very low < 0.1 percent for death
  1. In astrocytic tumor, DNA is commonly lost on chromosomes
    1. 17P, 13Q and 9Q
    2. 18P, 14Q and 9P
    3. 18Q, 15Q and 9Q
    4. 17P, 13Q and 9P
Ans. D7
  1. Important predictors of a tumor's potential for rapid growth and aggressive invasion of normal surrounding tissue is
    1. Hypercellularity and mitotic activity
    2. Hypercellularity and nuclear atypia
    3. Endothelial proliferation and necrosis
    4. Nuclear and cytoplasmic atypia
Ans. C.
  1. Drug of choice in tic doulourex is
    1. Phenytoin
    2. Gabapentin
    3. Baclofen
    4. Carbamazepine
Ans. D. Carbamazepine is the drug of choice and is effective initially in 75% of patients. It should be started gradually as a daily dose of 100 mg and increased to 200 mg qid
  1. “Parosmia” is for those
    1. Who recognize and name the odours quickly
    2. Who recognize, but cannot name them
    3. Who can detect a smell and earily distinguish differences but can neither recognize nor name them
    4. Those for whom each odour smells the same but is distorted and unpleasant
    5. Those who can small nothing in one or both nostril
Ans. D. Parosmia is not uncommon in incomplete olfactory recovery following head injuries. It may also occur in depressive or schizophrenic states. Option ‘E’ is anosmia
  1. Causes of Anosmia are
    1. Parkinson's disease
    2. Syphilis
    3. Sarcoidosis
    4. Head injuries
    5. Heavy smoking
Ans. All of the above
  • Local acute or chronic inflammatory nasal disease and head injury are by far the most common causes. Head injury and atrophy of the olfactory bulbs may be accompanied by CSF rhinorrhea8
  1. About testing of visual acuity
    1. Snellen's type charts are used for testing distant vision
    2. Jaegar type cards for near vision
    3. 6/5 – 6/6 are within the average normal range
    4. Jaegar type card must be held 30 cm from the patient's eye
    5. Average acuity lies between J-2 and J-4
Ans. All of the above are true
  1. Phenomenon of visual inattention is common in
    1. Frontal lobe disorder
    2. Parietal lobe disorder
    3. Temporal lobe disorder
    4. Occipital lobe disorder
Ans. B
  • Anton's syndrome (denial of visual deficit) and Redlichs phenomenon are seen in occipital lobe disorder
  1. About visual examination
    1. Bjerrum's screen enlarges the central area (out to 30) and make it easier to detect scotomata and blind spot
    2. Distance of the patient eye from the fixation point is usually 330 mm for the perimeter and 2000 mm for the Bjerrum's screen
    3. Field for red is greater than that for white object.
    4. By using red object, early defect may sometime be detected
Ans. A. B. D. are correct
  • Field for red is smaller than that for white object
  1. Match the following appearance of disc
    1. Primary optic atrophy – Whole disc is quite white
    2. Consecutive optic atrophy – Whole disc is pale
    3. Temporal pallor – Lesion of the papillomacular bundle
    4. Myopic disc – Very pale, appears greatly enlarged and a crescent of pallor around it
    5. Hypermetropic disc – Pink, edge appear blurred, small disc vessel and fields are normal
Ans. All are true9
  1. About the optic disc
    1. The temporal side of the disc is usually paler than the nasal side
    2. Normally the nasal edge of the disc is usually blurred.
    3. In papilledema, despite the degree of swelling, vision is well preserved; visual fields showing only enlargement of the blind spot
    4. In papillitis, the degree of swelling is slight, usually unilateral, veins not engorged, humping slight, disc area not greatly enlarged
    5. In papillitis, the vision is grossly disturbed due to large central or centro-caecal scotoma
Ans. All are true
  1. Foster Kennedy syndrome is due to tumor in
    1. Superior frontal region
    2. Inferior frontal region
    3. Inferior temporal region
    4. Inferior occipital region
Ans. B. It consists of a triad of
  1. Ipsilateral anosmia
  2. Ipsilateral optic atrophy
  3. Contralateral papilledema
    Foster Kennedy syndrome is found in olfactory groove meningioma and medial one third sphenoid meningioma
  1. Causes of pseudopapilledema are
    1. Obliquity of the optic nerves
    2. Drusen bodies
    3. Juxta-papillary choroiditis
    4. Haziness of the vitreous
    5. Pseudotumor cerebri
Ans. A to D
  • E. is false, i.e. pseudotumor cereberi. There is true papilledema but not due to intracranial tumor. Pseudopapilledema swelling is mild but without vessel engorgement and symptomic throughtout the patient's life10
  1. Spidery black “bone corpuscles” pigmentation spreading toward center is seen in
    1. Toxoplasmosis
    2. Retinitis pigmentosa
    3. Cerebromacular degeneration
    4. Choroidoretinitis
Ans. B
  1. Cotton-wool patches are seen in
    1. Papilledema
    2. Renal failure
    3. Polyarteritis nodosa
    4. SLE
    5. Retinal embolism and severe anemias
Ans. All are true.
  • Cotton wool spot is essentially a focal ischemic reaction (microinfarct) of injured axons
  1. About fluorescein and fundus photography
    1. Sterilized solution of 10% fluorescein is used
    2. In retinal artery, occlusion failure of filling is seen
    3. Gross exudation is seen in central venous occlusion and true papilledema
    4. Gross exudation is absent in pseudopapilledema
    5. In true optic atrophy, the disc remains dark throughout serial photographs
Ans. All are true
  1. Horner's syndrome consists of all EXCEPT
    1. Miosis
    2. Mydriasis
    3. Enophthalmos
    4. Anhydrosis
Ans. B.
  • Horner's Syndrome consists of miosis, ptosis, enophthalmos, dryness and warmth of that half of the face11
  1. The commonest cause of dilated pupil is
    1. Vascular accident in midbrain
    2. Tentorial herniation
    3. Carotid artery aneurysm
    4. Mydriatic
Ans. D.
  • A. B. C. all cause mydriasis but D. is the commonest cause
  1. Holms Adie syndromes consist of all EXCEPT
    1. Pupillary reflex to light is absent
    2. Pupillary reflex to accommodation is present
    3. Response to 1% pilocarpine
    4. Present in young males
    5. In full syndrome, ankle and knee jerks are absent and occasionally there is complete areflexia
Ans. D.
  1. About the action of ocular muscles
    1. Superior rectus elevates the eye when it is turned outwards
    2. Inferior oblique elevates the eye when it is turned inwards
    3. Inferior rectus depress the eye when it is turned inward
    4. Superior oblique depress the eye when it is turned inwards
    5. Medial rectus moves the eye horizontally inwards
Ans. A, B, D and E. are correct
  1. Superior oblique results in
    1. Extorsion, Depression, Adduction
    2. Intorsion, Depression, Adduction
    3. Intorsion, Depression, Abduction
    4. Extorsion, Depression, Abduction
Ans. C
  1. Deviation of the head and eyes to the right occur in all EXCEPT
    1. Lesion of right frontal lobe
    2. Lesion of left frontal lobe
    3. Lesion of left side of pons
    4. Irritative lesion of left frontal lobe
    5. Irritative lesion of right side of pons
Ans. B.
  • Lesion of left frontal lobe will result in deviation to left side12
  1. Ataxic nystagmus is seen in
    1. Arnold-Chiari malformation
    2. Syringobulia
    3. Labyrinthine disease
    4. Internuclear ophthalmoplegia
Ans. D.
  1. The commonest cause of vertical nystagmus is
    1. Drugs
    2. Arnold-Chiari
    3. Syringobulbia
    4. Basilar invagination
Ans. A.
  1. About vertical nystagmus
    1. Nystagmus on looking upwards
    2. Oscillation in up and down direction
    3. Never labyrinthine in origin
    4. Quick phase is most often upward
    5. Essentially due to intrinsic disturbance in brainstem
Ans. B to E. are correct
A. is false
  1. Match the following nystagmus
    1. Downbeat nystagmus – Pineal tumor
    2. Convergence retraction nystagmus – Arnold-Chiari malformation
    3. See-saw nystagmus – Internuclear ophthalmoplegia
    4. Ocular bobbing – lesion in pontine tegmentum
    5. Ataxic nystagmus – Suprasellar tumor
Ans. D.
  • Downbeat nystagmus is seen in Arnold-Chiari malformation
  • Convergence retraction nystagmus is seen in Pineal tumor (Perinaud's syndrome)
  • See-saw nystagmus is seen in Suprasellar lesion anterior to third ventricle
  • Ataxic nystagmus – Internuclear opthalmoplegia13
  1. Peduncular nystagmus is seen in
    1. Macular abnormalities
    2. Chorioidoretinitis
    3. Albinism
    4. High infantile myopia
    5. Opacities of the media
Ans. All are true
  1. About horizontal nystagmus
    1. In more peripheral lesion, the quick phase is away from the lesion
    2. Amplitude is greater in the direction of quick phase
    3. In cerebellar lesion, the quick phase and the greatest amplitude is toward the side of the lesion
    4. In cerebello pontine angle tumor, both central and peripheral effect, but the amplitude is greater toward the side of the lesion
    5. Nystagmus due to central lesion tend to be more chronic
Ans. All are true
  1. About optokinetic nystagmus
    1. Is a normal phenomenon
    2. Quick phase is back towards the primary position, this is the reverse of all other forms
    3. Absent in deep parietal lobe lesion
    4. Particular value in patient with a homonymous hemianopia
    5. Optokinetic response is absent when the drum is rotated away from the lesion
Ans. A to D are correct
  1. About jaw jerk
    1. Absent jaw jerk is rarely helpful
    2. In many normal people, no response is obtained
    3. Afferent and efferent is by V nerve
    4. Exaggerated jaw jerk point UMN lesion below pons
    5. Exaggerated jaw jerk is seen in pseudobulbar palsy, motor neuron disease and quite often in multiple sclerosis
Ans. A, B, C and E. D is false14
  1. Loss of corneal reflex is seen in
    1. Cerebello pontine angle tumor
    2. Carotid aneurysm
    3. Superior orbital fissure tumor
    4. Rostral 2/3 of spinal trigeminal tract lesion
    5. Caudal 1/3 of spinal trigeminal tract lesion
Ans. A to D are correct. E is false.
  • The afferent corneal reflex is as follows:
  • First order neurons of the ophthalmic nerve (CN V-1) are found in the trigeminal ganglion. Their axons enter the pons and descend in the spinal trigeminal tract. They enter the spinal trigeminal nucleus in its rostral portion and synapse on second order neurons which project to the ipsilateral and contralateral facial nuclei. Axons from third order neurons in the faciar nuclei innervate the orbicularis oculi muscles bilaterally (directly and consensually). Trigeminal tractotomy at caudal levels produces facial anesthesia without interruption of the corneal reflex.
  1. “Emotional Facial Weakness” occurs in
    1. UMN lesion
    2. LMN lesion
    3. Ipsilateral thalamic lesion
    4. Contralateral thalamic lesion
Ans. D.
  • A deviation of the mouth on smiling, which disappear on voluntary movement, constitute the so called ‘Emotional Facial Weakness’. This occurs in deep seated lesions of the opposite thalamus, or its connection with frontal lobe
  1. Schirmer's test is abnormal in lesion of the following EXCEPT
    1. Lesser petrosal nerve
    2. Greater petrosal nerve
    3. Sphenopalatine ganglion
    4. Lacrimal nerve
Ans. A15
  1. Most common cyanotic heart disease associated with brain abscess in children is
    1. Tetralogy of Fallot
    2. VSD
    3. Patent foramen ovale
    4. Transposition of great vessels
Ans. A
  1. Regarding brain abscess
    1. 85 to 95% of cerebellar abscesses are associated with ear or mastoid infections
    2. Four stages of brain abscess formation is best described by following infection with beta-hemolytic streptococci
    3. Encapsulation is frequently more complete on cortical side than on the ventricular side
    4. Capsule is extensive in abscesses resulting from hematogenous spread than in those arising from a contiguous focus of infection
    5. Propensity of abscess rupture is greater laterally than medially
Ans. A to C are correct
  • D and E. are false. Abscess from hematogenous spread has thin capsule. Abscess from contiguous spread has thick capsule
  1. Subdural empyema complicating meningitis is due to
    1. S. pneumoniae
    2. H. influenzae
    3. Both
    4. None
Ans. C. Due to S. pnuemoniae and H. influenzae but most commonly due to H. influenzae
  1. Most important associated pathogen in patient with cavernous sinus thrombosis is
    1. S. aureus
    2. Streptococci pneumoniae
    3. Bacteroides
    4. H. influenzae
Ans. A.16
  1. Pseudomembranes in case of chronic SDH is derived from
    1. Dura
    2. Arachnoid
    3. Both
    4. None
Ans. A
  1. Normal ICP is between
    1. 02 – 12 mmHg
    2. 04 – 16 mmHg
    3. 06 – 18 mmHg
    4. 10 – 20 mmHg
Ans. A.
  1. Most common cause of death in a patient of head injury is
    1. Raised ICP
    2. Hemorrhage
    3. Coma
    4. Respiratory complication
Ans. A. At least 50% of patients who die as a result of head injury do so because of uncontrolled increase in ICP
  1. First operation for a glioma was done by
    1. Bailey and Cushing (1926)
    2. Virchow (1863)
    3. Bennett and Godlee (1884)
    4. Harvey Cushing
Ans. C.
  • Gliomas initially classified by Bailey and Cushing (1926)
  1. About tumor marker
    1. PLAP (Placental alkaline phosphatase)– positivity indicates germ-cell differentiation
    2. Alpha-fetoprotein indicates the presence of yolk sac elements
    3. hCG positivity indicates the presence of syncytiotrophoblastic elements
    4. Vimentin is being called the “default” intermediate filament protein in CNS tumors
    5. Menigiomas typically exhibit EMA-positivity, a feature that helps in differential diagnosis with hemangiopericytoma
Ans. All of the above are true17
  1. Most widely used “Pan-endocrine” marker
    1. Synaptophysin
    2. NSE
    3. Chromogranin
    4. None of the above
Ans. C.
  1. All of the following are Intermediate Filament Protein EXCEPT
    1. Nestin
    2. Vimentin
    3. S-100
    4. Cytokeratin
    5. EMA
Ans. E
  1. The p53 protein is coded by a tumor suppressor gene on chromosome
    1. 17p 13.1
    2. 18p 13.1
    3. 17q 13.1
    4. 18q 13.1
Ans. A
  1. Which gene amplification constitutes a hallmark of primary glioblastomas?
    1. EGFR
    2. Myc
    3. Both
    4. None
Ans. A.
  • EGFR gene amplification constitutes a hallmark of primary glioblastomas, more than 60% of which show EGFR expression. All glioblastomas with EGFR amplification also show simultaneous loss of [chromosome 10]. Myc is the most common oncogene amplified in medulloblastomas
  1. About contribution to neurosurgery
    1. Moorey and Hinshaw in 1979 introduced the first MRI
    2. Yasargil used operating microscope in 195718
    3. CUSA (Cavitron ultrasonic aspirator) by Epstein in 1983
    4. Laser by Tew in 1983
    5. Introduction of intra operative endoscopy to brain surgery by Oppel in 1987
Ans. All of the above are true
  • James M Drake et al 1991 (First computer and robot assisted resection of thalamic astrocytoma)
  1. Match the following:
    1. Tuberous sclerosis – Subependymal giant cell astrocytoma
    2. NF-2 – Bilateral acoustic schwannoma
    3. NF-1 – Optic Glioma
    4. VHL syndrome – Hemangioblastoma
    5. Gardner syndrome – Medulloblastoma
Ans. A to D are correct
  1. NE (norepinephrine) containing cell bodies are located in
    1. Locus ceruleus and ventral tegmental nuclei
    2. Locus ceruleus and lateral tegmental nuclei
    3. Ventral and lateral tegmental nuclei
    4. Lateral and dorsal tegmental nuclei
Ans.B.
  1. Baclofen acts as
    1. Agonist on GABAa receptor
    2. Antagonist on GABAa receptor
    3. Agonist on GABAb receptor
    4. Antagonist on GABAb receptor
Ans. C. It is an antispasticity agent
  1. Which of the following is a positive abnormality?
    1. Spasticity
    2. Hemiparesis
    3. Loss of momory
    4. Impaired sensation
Ans. A.
  • Negative abnormality means loss of function
  • Positive abnormality indicates inappropriate excitation (seizure, spasiticity)19
  1. Wallenberg's syndrome is due to occlusion of
    1. AICA
    2. SCA
    3. PCA
    4. PICA
Ans. D.
  • Posterior Inferior Cerebellar Artery
  1. About Central Nervous System
    1. Brain weighs about 2% of the body weight
    2. Spinal cord is 42 to 45 cm in length
    3. Spinal cord is about 2% of the weight of the brain
    4. About 20% of extra cellular fluid is present of total brain volume
    5. Conus end at the L1 or L2 level of the vertebral column
Ans. All of the above are true
  1. About spinal cord
    1. Until the third month of fetal life, the spinal cord is as long as the vertebral canal
    2. At birth, the cord extends to about the level of the third lumbar vertebra
    3. At the 6th month of fetal life, the caudal end lies at the level of the first sacral vertebrae
    4. The adult level of the L1 – L2 junction is reached after the third year of life
    5. It is about 28 gm in weight
Ans. All are true
  1. About branching of spinal nerves
    1. Medial posterior primary division is sensory
    2. Lateral posterior primary division is motor
    3. Anterior primary division form plexuses
    4. White rami communicans present in the thoracic and upper lumber nerves
    5. Sinuvertebral nerves carry sensory and vasomotor innervation to the meninges
Ans. All of the above are true20
  1. Inverse stretch reflex is mediated by
    1. Ia
    2. Ib
    3. II
    4. III
Ans. B.
  1. Number of intrafusal fibres contained within the capsule of the muscle spindles are
    1. 02 – 10
    2. 10 – 20
    3. 20 – 30
    4. 30 – 40
Ans. A
  1. Gamma motor neuron make up % of fibres in the ventral roots
    1. 10 – 15%
    2. 15 – 25%
    3. 25 – 30%
    4. 30 – 40%
Ans. C.
  1. Center of accommodation reflex is
    1. Midbrain
    2. Pons
    3. Occipital cortex
    4. Medulla
Ans. C.
  1. Center for cremastric reflex is
    1. L1
    2. L2
    3. L3
    4. L4
Ans. A. i.e. [L1]
  • [Afferent – Femoral]
  • [Efferent – Genitofemoral]21
  1. All are true about inverse stretch reflex EXCEPT
    1. It is monosynaptic
    2. It is disynaptic
    3. Mediated by Golgi tendon organ
    4. Afferent carried by Ib fibres
Ans. A. i.e. monosynaptic is a false statement
  1. The largest cerebellar peduncle is
    1. Superior cerebellar peduncle
    2. Middle cerebellar peduncle
    3. Inferior cerebellar peduncle
    4. All of the above
Ans. B
  1. GVE fibres are present in
    1. III
    2. IV
    3. VI
    4. VII
    5. IX
Ans. A, D and E
  1. Number of true olfactory nerves are
    1. 16 – 20
    2. 18 – 30
    3. 20 – 40
    4. 30 – 50
Ans. B. (09 to 15 on each side)
  1. External landmarks of the hypothalamus are all EXCEPT
    1. Optic chiasm
    2. Tuber cinereum
    3. Mamillary bodies
    4. Tuberculum cinereum
Ans. D
  1. Hypothalamus is about
    1. 3% of the brain weight
    2. 6% of the brain weight22
    3. 8% of the brain weight
    4. 1% of the brain weight
Ans. A. 3% of brain weight, i.e, 4 gm
  1. Response to heat and cold is integrated by
    1. Anterior and medial hypothalamus respectively
    2. Anterior and lateral hypothalamus respectively
    3. Anterior and posterior hypothalamus respectively
    4. Posterior and anterior hypothalamus respectively
Ans. C.
  • Response to heat integrated by Anterior hypothalamus
  • Response to cold integrated by Posterior hypothalamus
  1. Wernicke's area is present in
    1. Posterior third of superior temporal gyrus
    2. Anterior third of superior temporal gyrus
    3. Middle third of superior temporal gyrus
    4. All of the above
Ans. A.
  1. Pure word deafness occur due to lesion in area
    1. 41
    2. 22
    3. 21
    4. 20
Ans. B.
  1. Calcar avis is present in the
    1. Frontal horn
    2. Temporal horn
    3. Occipital horn
    4. None of the above
Ans. C. It is the elevation of the ventricular wall produced by the calcarine fissure
  1. Tapetum is derived from the
    1. Rostrum of the corpus callosum
    2. Genu of the corpus callosum
    3. Body of the corpus callosum
    4. Splenium of the corpus callosum
Ans. C.23
  1. Choroid plexus is absent in all EXCEPT
    1. Frontal horn
    2. Occipital horn
    3. Cerebral aqueduct
    4. Temporal horn
Ans. D.
  1. The most common site of obstruction leading to hydrocephalus is
    1. Foramen of Monro
    2. Cerebral Aqueduct
    3. Foramen of Magendie
    4. Foramen of Luschka
Ans. B.
  1. Number of intervertebral disc present in the spinal column are
    1. 22
    2. 23
    3. 24
    4. 25
Ans. B. Twenty three intervertebral disc extend from the C2–3 to the L5-SI intervertebral level. No intervertebral disc exist between the cranium and C1, between C1 and C2, in the sacrum or in the coccyx
  1. In diffusion weighted imaging, the apparent diffusion content (ADC) is higher in
    1. Epidermoid
    2. Arachnoid cyst
    3. Abscess
    4. Necrotic tumor
    5. Lymphoma
Ans. B and D. Diffusion weighted imaging reflects the brownian motion of tissue water. DWI has a sensitivity and specificity of over 90% for distinguishing epidermoid (low ADC) from arachnoid cyst (high ADC) and distinguishing abscesses (low ADC) from necrotic tumor (high ADC). An inverse correlation between minimum ADC and tumor cellularity has been verified in a wide variety of tumor. Lesions having low ADC appear hyperintense on DWI and those with high ADC appear hypointense on DWI24
  1. Tight junction between ependymal cell is present in
    1. Upper third ventricle
    2. Lower third ventricle
    3. Lateral ventricle
    4. Fourth ventricle
Ans. B.
  1. Anterior condylar canal transmit
    1. Emissary vein from the transverse sinus
    2. Hypoglossal nerve and a meningeal artery
    3. Both of the above
    4. None of the above
Ans. B.
  • XII nerve and a meningeal artery. Anterior condylar canal is also called Hypoglossal canal. Posterior condylar canal transmit emissary vein from the transverse sinus
  1. Under normal condition, the pressure in the small cerebral arteries is maintained at
    1. 250 mm H2O
    2. 350 mm H2O
    3. 450 mm H2O
    4. 550 mm H2O
Ans. C.
  1. The most common and severe atherosclerotic lesion are in the
    1. Carotid bifurcation
    2. Origin of the vertebral artery
    3. Upper part of the basilar artery
    4. Lower part of the basilar artery
Ans. A.
  1. Drugs effective in acute attack of migraine are
    1. β-blockers
    2. Ca2+ channel blockers
    3. Tricyclic antidepressants
    4. 5HT agonist
    5. 5HT antagonist
    25
Ans. D
  • Ergotamine, DHE are partial agonists at 5-HT 1D/1B receptor
  • Sumatriptan is 5-HT 1D agonist
  • Rest are used for prophylaxis
  • Methysergide is 5-HT 1D antagonist [side effect is retroperitoneal fibrosis]
  1. Contribution to the corticospinal tract by giant cell of BETZ is about
    1. 03 – 05%
    2. 05 – 10%
    3. 01 – 03%
    4. 10 – 15%
Ans. A.
  1. How many % of fibres cross at pyramidal decussation?
    1. 85%
    2. 87%
    3. 90%
    4. 100%
Ans. B.
  • About 10% of the pyramidal tract does not cross in the pyramidal decussation and in addition up to 3% of the descending fibres in the lateral corticospinal tract are uncrossed.
  1. The telencephalon gives rise to each of the following EXCEPT
    1. Amygadla
    2. Caudate nucleus
    3. Claustrum
    4. Globus pallidus
    5. Putamen
Ans. D.
  • Globus pallidus: It is derived from diencephalon
  1. Muscle stretch reflexes are abnormal in all lesions of the following EXCEPT
    1. Motor end plate
    2. Myopathy
    3. Cerebellar system
    4. Basal ganglia
Ans. D26
  1. Number of retinotopic maps located in the area 19 are
    1. 1
    2. 2
    3. 3
    4. 4
Ans. C
  • Area 17 – one retinotopic map (V1)
  • Area 18 – two retinotopic maps (V2, V3)
  • Area 19 – three retinotopic maps (V3A, V4, V5)
  1. All of the following are true match EXCEPT
    1. Medial striate artery – Middle cerebral artery
    2. Recurrent artery of Heubner - ACA
    3. Medial lenticulostriate artery - MCA
    4. PCA – Basilar artery
    5. Labyrinthine artery - AICA
Ans. A
  1. Number of Betz cells in a single hemisphere is approx
    1. 10,000
    2. 20,000
    3. 30,000
    4. 40,000
Ans. D.
  1. Primary CNS tumor spreading through CSF are
    1. Oligodendroglioma
    2. Hemangioblastoma
    3. Primary CNS melanoma
    4. Medulloblastoma
Ans. All of the above are true
  1. The most common primary PNET responsible for extraneural spread is
    1. Medulloblastoma
    2. Meningioma
    3. Pineoblastomas
    4. Ependymomas
Ans. A. Medulloblastoma is the most common PNET27
  1. The most important factor related to radiation myelopathy is
    1. Total radiation dose
    2. Amount of tissue radiated
    3. Extent of cord shielding
    4. Rate of application
Ans. D
  1. With large field technique (>10cm of cord) the risk of radiation myelopathy is negligible with
    1. < 2.3 Gy in 42 days
    2. < 3.3 Gy in 42 days
    3. < 4.3 Gy in 42 days
    4. < 5.3 Gy in 42 days
Ans. B
  1. With small field technique, the risk of radiation myelopathy is negligible with
    1. < 2.3 Gy in 42 days
    2. < 3.3 Gy in 42 days
    3. < 4.3 Gy in 42 days
    4. < 5.3 Gy in 42 days
Ans. C. 4.3 Gy [i.e. .717 Gy/wk]
  1. The upper limit of radiation per fraction to prevent radiation myelopathy is
    1. 1 Gy / Fraction
    2. 2 Gy / Fraction
    3. 3 Gy / Eraction
    4. 4 Gy / Fraction
Ans. B.
  1. Following neurosurgeon is a Nobel laureate
    1. William Harvey Cushing
    2. Walter Dandy
    3. M Gazi Yasargil
    4. Egas Moniz
Ans. D.
  • Egas Moniz received nobel prize in 1949 for initiation of psychosurgery (Prefrontal leukotomy)28
  1. Maximum recommended radiation dose to optic nerve is
    1. 100 cGy
    2. 50 cGy
    3. 10 cGy
    4. 1 cGy
Ans. A
  • Eye lens
    – 100 cGy
    Optic nerve
    – 100 cGy
    Skin in bone
    – 050 cGy
    Thyroid
    – 010 cGy
    Gonads
    – 001 cGy
    Breast
    – 003 cGy
  1. About inverted supinator jerk
    1. Finger flexion is the only response
    2. Contraction of the brachioradialis and elbow flexion being absent
    3. Striking the bicep tendon may produce extension of the elbow
    4. Indicates a cord lesion at the fourth or fifth cervical level causing a lower motor neuron lesion at C5
    5. It is invaluable in localizing lesion responsible for spastic paraparesis which have no sensory abnormality and applies particularly to cervical spondylosis
Ans. All are true
  1. Lesion of the lateral leminiscus produce
    1. Bilateral complete deafness
    2. Bilateral partial deafness, greater in the contralateral ear
    3. Bilateral partial deafness, greater in the ipsilateral ear
    4. Unilateral, ipsilateral deafness
    5. Unilateral, contralateral deafness
Ans. B.
  • Fibres in the lateral leminiscus are both crossed and uncrossed with preponderance of crossed fibres.
  1. J shaped sella is seen in
    1. Pituitary adenoma
    2. ESS (Empty sella syndrome)29
    3. Craniopharyngioma
    4. Optic nerve glioma
    5. Hurler's syndrome
Ans. D and E
  1. Method of Oon is related to
    1. Anterior hypophysis
    2. Posterior hypophysis
    3. Pineal gland
    4. Superior colliculi
Ans. C.
  • There are a number of criteria to distinguish an abnormal amount of displacement in the AP or vertical direction. A simple method is the method of Oon. To locate the normal position of a calcified pineal gland on a lateral plain skull X-Ray, draw a line from the tuberculum sellae to the tip of Clivus. 1 cm from the top of this line, draw a 5 cm perpendicular line. The pineal gland normally lies within 1 cm radius circle
  1. “Insensitive space” as related to myelography
    1. L3 – L4
    2. L4 – L5
    3. L5 – S1
    4. None of the above
Ans. C.
  1. About Marcus Gunn pupil
    1. Affected pupil is larger than the other
    2. Consensual reflex is greater than direct
    3. Seen in multiple sclerosis
    4. Also called as afferent pupillary defect
    5. Detected by swinging flash light test
Ans. All of the above are true
  1. Mydriasis due to 3rd nerve compression is usually
    1. 05 – 06 mm
    2. 06 – 07 mm
    3. 07 – 08 mm
    4. 08 – 09 mm30
Ans. A.
  • A pharmacologically dilated pupil is very large (07 – 08 mm)
  • Miosis in Horner's syndrome is [02 – 03 mm)
  1. The most frequent site for traumatic SAH is
    1. Convexity
    2. Basal cistern
    3. Tentorial edge
    4. Sylvian fissure/Interhemispheric
Ans. A.
  • Convexity fissure > Sylvian fissure/Interhemispheric > Basal cistern > Tentorial edge
  1. The most common site of cerebral contusion associated with a subdural hematoma is the
    1. Frontal pole
    2. Cerebral convexity
    3. Temporal pole
    4. Occipital pole
Ans. C.
  • Temporal pole > Frontal pole > Cerebral convexity
  1. About subdural hematoma
    1. Subacute SDH is between 03 to 10 days
    2. Acute SDH has better prognosis than EDH
    3. Acute SDH can be evacuated by twist drill
    4. SDH is more common in infants and elderly patients
    5. MRI is more sensitive than CT for detecting isodense SDH
Ans. D and E
  • In anemic patients, SDH is isodense and hematocrit has to fall below 23% to become isodense
  1. The most common fracture associated with EDH is
    1. Parietal bone
    2. Squamous part of temporal bone
    3. Pterous part of temporal bone
    4. Frontal bone31
Ans. A.
  • Antero inferior portion of the parietal bone cause rupture of anterior branch of middle meningeal artery (most commonly)
  1. The most common source of bleeding in EDH is
    1. Anterior division of middle meningeal artery
    2. Posterior division of middle meningeal artery
    3. Superior cerebral vein
    4. Dural sinuses
Ans. A.
  1. The spinal canal diameter of children reaches adult size by age
    1. 3 to 5 years
    2. 6 to 8 years
    3. 10 to 12 years
    4. 14 to 16 years
Ans. B. i.e. 6 to 8 years. The spinal canal grows as the vertebrae grow. But once the neurocentral synchondroses and the midline posterior arch ossify and close, the spinal canal no longer grow. In the newborn and during infancy, the spinal canal is oval in shape and its transverse diameter larger than sagittal diameter. At birth to age 3 months, the sagittal diameter of the spinal canal is 1.0 cm in the cervical region and 1 to 1.3 cm in the lumbar region. At the end of the first decade of life, the spinal canal should approach adult size, whereby the sagittal diameter is 15 to 27 mm in the cervical, 17 to 22 in the thoracic and 15 to 27 mm in the lumbar spine
  1. Which of the following drugs is useful in Ischemic head injury [NMDA antagonist]?
    1. MK – 801
    2. Phencyclidine
    3. Dextromethorphan
    4. Ketamine
Ans. C.
  • Phencyclidine: MK – 801, Ketamine has a lot of side effects in man. However, dextromethorphan may be useful in future32
  1. Diffuse axonal injury was described by
    1. Howship in 1816
    2. Benett and Godlee in 1884
    3. Virchow in 1863
    4. Strich in 1956
Ans. D.
  • Howship, in 1816, first described growing skull fracture
  • Bennett and Goalec in 1884 first operated on Glioma
  1. The most common segment of optic nerve to be injured in closed head injury is
    1. Intracranial
    2. Intraocular
    3. Intracanalicular
    4. Intraorbital
Ans. C
  1. About hemodynamics in brain
    1. CPP = MAP – ICP
    2. Normal adult CPP is > 50 mmHg
    3. CPP would have to drop below 40 mmHg in a normal brain before CBF would be impaired
    4. Emergency measures should be initiated if ICP > 20 – 25 mmHg
    5. Normal ICP in adults and older children is < 10 – 15 mmHg; young children 03 – 07 mmHg; in term infants 1.5 – 06 mmHg respectively
Ans. All of the above are true
  1. Traube Hering Waves are
    1. Lundenberg Type A waves
    2. Lundenberg Type B waves
    3. Lundenberg Type C waves
    4. None of the above
Ans. C.
  • Lundenberg Type A waves or plateau waves — >50 mmHg, 05 — 20 minutes
  • Lundenberg Type B waves — 10 — 20 mmHg, 30 sec — 02 minutes
  • Lundenberg Type C waves — 04 — 08 per minute33
  1. About anastomotic veins
    1. Superior anastomotic vein is the Vein of Labbe.
    2. Vein of Trolard is present in parietal lobe
    3. Vein of Labbe is the vein that crosses the temporal lobe between the sylvian fissure and the transverse sinus
    4. Inferior anastomotic vein is Vein of Trolard
Ans. B and C. Vein of trolard drain into superior sagittal sinus.
  1. Neurofibromatosis type I is associated with
    1. 17 p 11.2
    2. 17 p 12.2
    3. 17 Q 11.2
    4. 17 Q 12.2
Ans. C.
  1. Neurofibromatosis type II is associated with
    1. 22 p 11.2
    2. 22 q 11.2
    3. 22 p 12.2
    4. 22 q 12.2
Ans. D.
  1. High dose barbiturate therapy in trauma results in
    1. Reverse steal phenomenon
    2. Decrease CMRO2
    3. Free radical scavenging
    4. Reduced intracellular calcium
    5. Lysosomal stabilization
Ans. All of the above are correct
  1. Classic CT appearance of EDH occur in
    1. 84%
    2. 74%
    3. 94%
    4. 64%
Ans. A.
  • Classic appearance – 84%
  • Crescent shaped – 05%34
  1. Spinal trigeminal tract sometime descend as low as
    1. C2
    2. C3
    3. C4
    4. C5
Ans. C.
  • Normally extend from Pons to C3 level.
  1. The best view to demonstrate cervical articular masses is
    1. Pillar's view
    2. Towne's view
    3. Swimmer's view
    4. Caldwell – Luc's view
Ans. A
  • Pillar's view is reserved for cases suspected of having articular mass fracture. The head is rotated to one side, the X-ray tube is off centered 2 cm from midline in the opposite direction and the beam is angled 25 caudal cornered at superior margin of thyroid cartilage
  1. The best view for cervicothoracic junction is
    1. Pillar's view
    2. OMO view [open mouth odontoid view]
    3. Swimmer's view
    4. Towne's view
    5. Caldwell-Luc's view
Ans. C
  1. The most common incomplete spinal cord injury is
    1. Posterior cord syndrome
    2. Anterior cord syndrome
    3. Central cord syndrome
    4. Brown-Sequard syndrome
Ans. C.
  • Central cord syndrome: Most common type of incomplete spinal cord injury syndrome usually seen following acute hyper extension injury in an older patient35
  1. The worst prognosis in incomplete spinal cord injury is seen in
    1. Posterior cord syndrome
    2. Anterior cord syndrome
    3. Central canal syndrome
    4. Brown-Sequard syndrome
Ans. B.
  • Anterior cord syndrome only 10 – 20% recover functional motor control
  1. Syndrome having the best prognosis of any of the incomplete spinal cord injury is
    1. Brown-Sequard syndrome
    2. Anterior spinal cord syndrome
    3. Posterior spinal cord syndrome
    4. Central spinal cord syndrome
Ans. A.
  • Approximately 90% of patients with this condition will regain the ability to ambulate independently as well as urinary and anal sphincter
  1. The most important structure in maintaining atlanto-occipital stablility are
    1. Tectorial membrane + Alar ligament
    2. AAO ligament + PAO ligament
    3. Cruciate ligament
    4. Transverse ligament + AA Portion of the alar ligament
Ans. A.
  1. Characteristic “cock robin” head position is seen in
    1. Atlanto occipital dislocation
    2. Atlanto axial rotatory subluxation
    3. Anterior atlanto axial dislocation
    4. Posterior atlanto axial dislocation
Ans. B.
  • 20° lateral tilt to one side, 20° rotation to the other and slight flexion seen in atlanto axial rotatory subluxation36
  1. About Odontoid fracture
    1. Odontoid # comprise 10 – 15% of all cervical spine fractures
    2. Flexion is the most common mechanism of injury
    3. Suggestive finding is the tendency to support the head with hands when going between the upright and supine position
    4. Early surgery is recommended for all type I A #
    5. Surgery is recommended for odontoid type B II # in patients > 7 years age with displacement > 6 mm
Ans. A, B, C and E are correct.
  1. The most effective Cervico Thoracic Orthesis for bracing against flexion – extension and rotation is
    1. Yale brace
    2. SOMI brace
    3. Guilford brace
    4. None of the above
Ans. A.
  1. The most common type of burst # of thoracolumbar region is
    1. # of both end plates
    2. # of superior end plates
    3. # of inferior end plates
    4. Burst rotation
Ans. B
  • Fracture of superior end plates seen at thoracolumbar junction—Mechanism axial load and flexion
  1. About intervertebral disc all are true EXCEPT
    1. The disc is a vascular structure at birth
    2. The disc is smaller in the newborn and infant
    3. Vascularity gradually decreases throughout infancy and early childhood until about age 4 when disc becomes avascular
    4. Best imaged on MRI because of its high water content
Ans. B. The disc is larger in the newborn and infant
  1. About pediatric spinal meningioma al are true EXCEPT
    1. Most frequently occur in the cervical and thoracic regions
    2. The marked female predominance noted in adults is not reproduced in children
    3. NF type 2 diagnosis should be investigated in all childrens37
    4. Cystic changes seen in 50% of pediatric spinal meningiomas
    5. Histopathologically spinal meningioma do not share the same subtype as seen with intracranial tumors
Ans. E. Histopathologically, spinal meningiomas share the same subtype as seen with intracranial tumors. Psammomatous lesion seen to be the most common. Cystic change occur less than 5% in adult tumors. Clear cell meningioma has been seen in children and adults, with many patients presenting before the age of 10 years, and present most frequently in lumbar spine. They recur at a high rate. Complete resection is the primary therapy for intradural meningioma
  1. The most common cause of strokes in patients under 45 year of age is
    1. Trauma
    2. Atherosclerosis
    3. Embolism
    4. Vasculopathy
Ans. A
  • Trauma
    – 22%
    Atherosclerosis
    – 20%
    Embolism
    – 20%
    Vasculopathy
    – 10%
  1. The most common lacunar manifestation is
    1. Pure sensory CVA
    2. Pure motor hemiparesis
    3. Ataxic hemiparesis
    4. PMH sparing the face
Ans. A.
  • Pure sensory CVA
  • Pure motor hemiparesis (2nd most common)
  1. About Anterior choroidal artery
    1. Branch of ICA
    2. Supply the choroid plexus of temporal horn
    3. Historically ligation was done for parkinsonism
    4. Causes contralateral hemiparesis, hemihypesthesia, homonymous hemianopia
    5. Supply the ventral half of the internal capsule
Ans. All are correct38
  1. Terson syndrome which occur in case of aneurysmal SAH is
    1. Subhyaloid hemorrhage
    2. Intraretinal hemorrhage
    3. Vitreous humor hemorrhage
    4. Paroxysmal headache
Ans. C.
  • [Hemorrhage inside the vitreous humor] Occur in 4–27% patients of subarachnoid hemorrhage.
  1. The most sensitive test for SAH is
    1. Lumbar puncture
    2. MRI
    3. Non contrast CT
    4. Cerebral angiogram
Ans. A.
  1. The “Gold standard” for evaluation of cerebral aneurysm is
    1. Lumbar puncture
    2. MRA
    3. CTA
    4. Cerebral angiogram
Ans. D
  1. The maximal frequency of rebleeding in aneurysmal SAH is on
    1. Day 1
    2. Day 2
    3. Day 3
    4. Day 4
Ans. A
  • The maximal frequency of rebleeding is in the 1st day (4% on day 1), then 1.5% daily for 13 days 15 – 20% rebleed within 14 days, 50% will rebleed within 6 months
  • The highest risk of rebleeding occurred in the first 6 months following SAH
  1. The single most common site of aneurysm presenting with SAH is
    1. Anterior communicating artery aneurysm
    2. PCoA aneurysm
    3. Distal anterior cerebral artery aneurysm
    4. MCA aneurysm
Ans. A.39
  1. Ophthalmic artery aneurysm result in following visual defect EXCEPT
    1. Ipsilateral monocular superior nasal quadrantopsia
    2. Junctional scotoma
    3. Ipsilateral inferior nasal cut
    4. Bitemporal hemianopia
Ans. D
  1. Ophthalmic artery in relation to optic nerve is
    1. Superomedial
    2. Inferomedial
    3. Superolateral
    4. Inferolateral
Ans. D
  1. The most common primary non glial tumor to have hemorrhage is
    1. Meningioma
    2. Schwannoma
    3. Primary CNS lymphoma
    4. Pituitary adenoma
Ans. D
  • Pituitary apoplexy occurring in 0.6% to 12.3% of pituitary adenoma. Histologically verified hemorrhage within pituitary adenoma is observed in 9.6–17% of cases. Pituitary adenomas bleed 5.4 times more frequently as compared to other intracranial tumor (Wakai 1982)
  1. The most common posterior circulation aneurysm is
    1. Basilar tip aneurysm
    2. VA – PICA aneurysm
    3. VA – BA aneurysm
    4. VA – AICA aneurysm
Ans. A
  • Basliar bifurcation aneurysm AKA Basilar tip aneurysm is the most common posterior circulation aneurysm. Account for 5 % of all aneurysms. VA-PICA aneurysm is the second most common posterior circulation aneurysm40
  1. About Mycotic aneurysm
    1. 4% of intracranial aneurysm
    2. Occurs in 3–15% of patients with subacute bacterial endocarditis
    3. Most common location distal MCA branches
    4. Most probably start in the adventitia (outer layer) and spread inwards
    5. Streptococcus is a common organism
Ans. All are true
  • These aneurysms usually have fusiform pathology and are usually friable
  • Typical SAH occur in 20% of patients
  1. Intervertebral disc constitute how much % of entire height of vertebral column
    1. 15%
    2. 25%
    3. 35%
    4. 45%
Ans. B.
  • 25%–It varies from 20–33%, but roughly it is 25%
  1. The most common vascular malformation is
    1. Venous angioma
    2. AVM
    3. Cavernous angioma
    4. Capillary telangiectasia
Ans. A.
  1. The most common angiographically occult cryptic vascular malformation (AOVM) is
    1. Cavernous angioma
    2. Telangiectasis
    3. Venous angioma
    4. AVM
Ans. D
  • AVM > Cavernous angioma > Telangiectasis > Venous angioma41
  1. The most common cause of lobar ICH in elderly normotensive patients is
    1. Tumor
    2. AVM
    3. Trauma
    4. Amyloid angiopathy
Ans. D
  • Cerebral amyloid angiopathy is the most common cause of lobar ICH in elderly normotensive patients
  1. The most common source of Tumoral intracerebral hemorrhage is
    1. GBM
    2. Lymphoma
    3. Choriocarcinoma
    4. Bronchogenic carcinoma
Ans. D.
  1. Tumors associated with Intracerebral hemorrhage
    1. Low grade astrocytoma
    2. Meningioma
    3. GBM
    4. Pituitary adenoma
    5. Acoustic neuroma
Ans. All are true
  1. The most common site of Intracerebral hemorrhage is
    1. Putamen
    2. Cerebellum
    3. Thalamus
    4. Lobar
Ans. A
  1. The physiological calcification is seen in
    1. Arachnoid granulation
    2. Choroid plexus
    3. Pteroclinoid ligament
    4. Habenular commisure
    5. Vein of Galen
Ans. A to D42
  1. The most common benign primary intra-orbital neoplasm is
    1. Lymphangioma
    2. Capillary hemangioma
    3. Lymphoma of the orbit
    4. Cavernous hemangioma
Ans. D.
  1. The normal spinal canal/vertebral body ratio is approx
    1. 1
    2. 0.5
    3. 0.8
    4. 1.5
Ans. A
  1. About nucleus pulposus all are true EXCEPT
    1. Disc is made up of annulus fibrosus and the nucleus pulposus
    2. Nucleus pulposus not in the center but lie a little anterior
    3. It has a volume of 0.5 ml and is about 0.5 cm in diameter
    4. Remnant of notochord
Ans. C. It has a volume of 0.2 ml and is about 0.7 cm in diameter
  1. All of the following are correct match EXCEPT
    1. Fusiform gyrus – lateral occipito temporal gyrus
    2. Lingual gyrus – medial occipito temporal gyrus
    3. Heschl gyrus – primary auditory area
    4. Entorhinal area – primary olfactory area
Ans. A to C.
  1. The parahippocampal gyrus is separated from the fusiform gyrus by
    1. Rhinal sulcus
    2. Calcarine sulcus
    3. Collateral sulcus
    4. Parieto-occipital sulcus
Ans. C.
  1. The Vein of Galen is present in
    1. Quadrigeminal cistern
    2. Ambient cistern43
    3. Suprasellar cistern
    4. Interpeduncular cistern
Ans. A.
  1. The Suprasellar cistern consists of
    1. Optic chiasma
    2. Internal carotid artery
    3. Circle of willis
    4. Pituitary stalk
    5. All of the above
Ans. E. All of the above
  1. Which Fisher grade is associated with severe vasospasm in SAH?
    1. Grade 1
    2. Grade 2
    3. Grade 3
    4. Grade 4
Ans. C.
  1. Characteristic “Popcorn” appearance seen on T2WI on MRI is seen in which vascular malformation
    1. AVM
    2. Cavernous angioma
    3. Venous angioma
    4. Capillary telangiectasia
Ans. B.
  1. “Medusa Head” appearance is seen on cerebral angiography in which vascular malformation
    1. AVM
    2. Cavernous angioma
    3. Venous angioma
    4. Capillary telangiectasia
Ans. C
  1. Tumor occurring in midline is
    1. Oligodendroglioma
    2. Epidermoid cyst44
    3. Cerebral metastases
    4. Rhabdomyosarcoma
Ans. A.
  • Oligodendroglioma (most common in frontal lobe)
  1. The most common primary sites which metastasise to calvarium is
    1. Breast
    2. Liver
    3. Seminoma
    4. Follicular carcinoma thyroid
Ans. A.
  • The descending order of frequency of primary sites which metastasise to calvarium is Breast >Lung>Prostate> Lymphoma
  1. Primary tumor associated with leptomoningeal metastases are
    1. Melanoma
    2. Ca of breast and lung
    3. Lymphoma
    4. Leukemia
    5. Ependymomas
Ans. All are true
  • Primary brain tumor having leptomeningeal spread pattern are GBM, ependymomas and pinealoblastomas
  • Dural metastases – Most common sites of primary are breast, lung and kidney
  1. Calcified brain metastases occur with
    1. Bronchiogenic Ca
    2. Ca breast
    3. Ovarian Ca
    4. Renal cell Ca
Ans. C.
  • Ovarian Ca due to secretion of mucin
  • Cystic metastases can be seen with lung and breast45
  1. The most common site of origin of meningioma in sellar and parasellar region is
    1. Dorsum sellae
    2. Clinoid process
    3. Tuberculum sellae
    4. Medial sphenoid wing
    5. Diaphragm sellae
Ans. C.
  1. The most common site of epidermoid cyst in the head is
    1. CP angle
    2. Intraventricular (IVth ventricle)
    3. Suprasellar
    4. Parasellar
Ans. A
  • Parasellar is the second most common site in the head after the CP angle
  1. Posterior pituitary bright spot on TIWI is absent in all EXCEPT
    1. Hand Schuller – Christian disease
    2. Stalk transection
    3. Pituitary adenoma
    4. Hyothalamic glioma
Ans. C.
  • Pituitary adenoma arises from adenohypoysis. Hand Schuller Christian disease consist of Diabetes inspidus, Exophthalmos, Lytic bone lesion, AAI – Atlanto axial instability
  1. Condition causing Atlanto axial instability are
    1. Down's syndrome
    2. Upper respiratory tract infection
    3. Mastoiditis
    4. Gout
    5. Tuberculosis
Ans. All of the above are true
  • Other options are parotitis, seronegative arthropathy, rheumatoid arthritis, morquio's syndrome46
  1. Condition causing abnormal redistribution of K+ from the intracellular to the extracellular compartment occur in
    1. β receptor blocker
    2. Succinylcholine administration
    3. Acute acidemia
    4. Reperfusion of ischemic limb
    5. Insulin deficiency
Ans. All are true
  1. Steroid responsive conditions are
    1. Sarcoidosis
    2. Lymphocytic Hypophysitis
    3. Tolosa Hunt syndrome
    4. None of the above
    5. All of the above
Ans. E
  1. Negative prognostic factors for low-grade glioma are all EXCEPT
    1. Largest diameter of tumor > 6 cm
    2. Histology is astrocytoma
    3. Age > 30 years
    4. Tumor crosses midline
    5. Neurologic deficit present
Ans. C. Age > 30 years
  1. Which of the following is a WHO Grade II tumor?
    1. Ganglioglioma
    2. Dysembryoplastic neuroepithelial tumor
    3. Central neurocytoma
    4. Paraganglioma of the filum terminale
Ans. C.
  • WHO Grade 1 tumors are Gangliolioma, Gangliocytoma, DNET, and paraganglioma of the filum terminale
  1. The most common feature of 3rd ventricular mass in children is
    1. Endocrine disturbance
    2. Visual disturbance
    3. Hydrocephalus
    4. Mental and behavioural changes
47
Ans. C
  • Diencephalic syndrome which comprises hyperkinesis and alrtness associated with emaciation in infants is the only specific syndrome associated with hypothalamic gliomas
  1. The most common posterior fossa neoplasm in children is
    1. Medulloblastoma
    2. Ependymoma
    3. Brainstem glioma
    4. Cerebellar astrocytoma
Ans. D.
  • Cerebellar astrocytoma > medulloblastoma > Ependymoma > Brainstem glioma
  1. % of the CSF entering in the spinal subarachnoid space is
    1. 20%
    2. 30%
    3. 10%
    4. 40%
Ans. A.
  1. Differential pressure across the arachnoid villi is about
    1. 10 – 20 mmHg
    2. 20 – 50 mmHg
    3. 30 – 70 mmHg
    4. 40 – 80 mmHg
Ans. B.
  1. Colpocephaly commonly seen in neonate is
    1. Disproportionate dilatation of occipital horn
    2. Disproportionate dilatation of frontal horn
    3. Disproportionate dilatation of temporal horn
    4. Disproportionate dilatation of third ventricle
Ans. A
  • Colpocephaly – dilatation of occipital horn + trigone
  1. Each of the following is characteristic of an acoustic neuroma EXCEPT
    1. Beksey type III or IV audiogram
    2. Loudness recruitment48
    3. Low short – increment sensitivity index
    4. Poor speech discrimination
    5. Pronounced tone decay
Ans. B
  • An absence of loudness recruitment is characteristic of a nerve trunk lesion, including an acoustic neuroma. Recruiting deafness occurs with a lesion in the organ of Corti, e.g. Meniere's disease. The other responses are characteristic of a retrocochlear (nerve) lesion
  1. Base of dens develop from
    1. Proatlas
    2. First occipital sclerotome
    3. First cervical sclerotome
    4. Second cervical sclerotome
Ans. C. i.e. first cervical sclerotome
  • Base of dens, which is an analogue of the C1 body develops from first cervical sclerotome
  1. The most common genetic abnormality in medulloblastoma is
    1. Isochromosome of chromosome 17
    2. Deletion of the short arm of chromosome 17
    3. Deletion on chromosome 1 q and 10 q
    4. Amplification of MYC
Ans. A.
  • Isochromosome of chromosome 17, arises in approx 50% of cases. Deletion of the short arm of chromosome 17 occur in 30–45% cases
  1. How much genes contribute in formation of human CNS?
    1. 10,000
    2. 12,000
    3. 14,000
    4. 16,000
Ans. B.
  • 40% of total = 40 × 30,000 / 100 = 12,000
  1. All tremors are absent on rest but pronounced on voluntary movement EXCEPT
    1. Senile tremor49
    2. Metabolic tremor
    3. Essential tremor
    4. Parkinson tremor
Ans. D
  1. All of the tumors show calcification EXCEPT
    1. Ependymoma
    2. Chordoma
    3. Pinealoblastoma
    4. Germinoma
Ans.D.
  • Germinoma does not show calcification
  1. About pediatric intracranial aneurysms all are true EXCEPT
    1. Cerebral aneurysm in childhood occur at a rate of 0.5–4.6%
    2. Male predominance and inversion of sex ratio is present
    3. ICA bifurcation is the most common location
    4. Rebleeding in the pediatric population is similar to that in adults
    5. Vasospasm better tolerated in young age group.
Ans. D. Rebleeding in the pediatric population is significantly higher than adults and is related to delayed diagnosis because of clinical polymorphism of SAH in children
  1. Pterional craniotomy is indicated in following aneurysms EXCEPT
    1. Anterior communicating artery aneurysm
    2. Posterior communicating artery aneurysm
    3. Distal anterior cerebral artery aneurysm
    4. Middle cerebral artery aneurysm
Ans. C
  1. Successful weaning from a ventilator is suggested by presence of which of the following?
    1. An alveolar arterial gradient of more than 350 mmHg
    2. A PaO2/FIO2 ratio of less than 200
    3. A PaCO2 over 55 mmHg
    4. A tidal volume [VT] of over 5ml/kg
Ans. D50
  1. About Pediatric disc disease all are true EXCEPT
    1. The segmental distribution of disc herniation in children is similar to that seen in adults
    2. Disc herniation is most often sub-ligamental and more than 80% are posterolateral in position
    3. 30 to 60% children with symptomatic lumbar disc herniation have a direct history of trauma
    4. Well managed by conservative treatment
Ans. D. Unlike the degenerative discs often found in adults, the highly elastic pediatric disc respond less well to conservative treatment.
  1. Venous angle is an important landmark for
    1. Aqueduct of sylvius
    2. Foramen of Monro
    3. Trigone of lateral ventricle
    4. Occipital horn of lateral ventricle
Ans. B.
  1. About MRI, all are true EXCEPT
    1. Most pathology are hyperintense on T2WI
    2. Most pathology are hyerintense on T1WI
    3. CSF is hypointense on T1WI but hyperintense on T2WI
    4. Gray matter is hyperintense than white matter on T2WI
Ans. B.
  1. Reduction of sensory nerve action potential and para spinal muscle fibrillation is seen in
    1. Plexus lesion
    2. Root lesion
    3. Both of the above
    4. None of the above
Ans. C.
  1. The most common muscle involved in the peroneal nerve palsy is
    1. EDL
    2. Peroneus longus
    3. Tibialis anterior
    4. EHL51
Ans. D.
  • Extensor hallucis longus
  • EHL > Tibialis anterior > Toe extensor > Peroneus longus + Peroneus brevis
  1. Tarsal tunnel syndrome involves the following nerve
    1. Anterior tibial nerve
    2. Posterior tibial nerve
    3. Common peroneal nerve
    4. Musculocutaneous nerve of leg
Ans.B.
  1. Endoneurium is intact in which of the following degree of nerve injury
Ans. B.
  1. Rotatory paralysis is characteristic of tumor in the following location
    1. Clivus
    2. Falx
    3. Foramen magnum
    4. Olfactory groove
    5. Tuberculum sellae
Ans. C.
  1. Fall of temperature by 1°C slows conduction in motor nerves by
    1. 1 m/s
    2. 2 m/s
    3. 3 m/s
    4. 4 m/s
Ans. B.
  1. Abductor policis longus is supplied by
    1. Anterior interosseus nerve
    2. Posterior interosseus nerve52
    3. Median nerve
    4. Radial nerve
Ans. B.
  1. Root value of lumbosacral trunk is
    1. L5, S1, S2
    2. L3, L4, L5
    3. L4, L5
    4. L5, S1
Ans. C.
  1. Meralgia paresthetica is due to involvement of a nerve which has the following root value
    1. T12, L1
    2. L1, L2
    3. L2, L3
    4. L3, L4
Ans. C.
  • (Lateral femoral cutaneous nerve of thigh)
  1. The number of Na+ channel per sqmicro meter of membrane of long axon of unmyelinated neuron is
    1. 110
    2. 120
    3. 130
    4. 140
Ans. A.
  1. About CNS
    1. Brain weighs about 2% of the body weight
    2. Blood flow per min in brain is greater than visceral organ
    3. Cerebral blood flow per minute in children is less than that of adults
    4. Glycogen in brain is greater than in muscles
    5. Glucose is responsible for 99% of production of ATP in brain
Ans. A.53
  1. Gerstmann syndrome is due to a lesion in
    1. Area 39, Right side
    2. Area 39, Left side
    3. Area 40, Right side
    4. Area 40, Left side
Ans. B.
  • Area 39 (angular gyrus), Left side
  1. About histopathologic finding and immunohistochemistry in schwannoma and Neurofibroma
    1. Neurofibroma react to vimentin, S −100 protein and leu-7
    2. Schwannomas are positive for S −100 protein and collagen IV or laminin
    3. Verocaybodies are more common in spinal schwannomas
    4. Neurofibromas cannot be separated from the entire nerve
    5. Schwannoma can be dissected free of the affected nerve
Ans. All the above are true
  1. In Vernet's syndrome there is
    1. External ophthalmoplegia
    2. Inability to shrug shoulders
    3. Taste disturbance in anterior 2/3rd of tongue
    4. Dysphagia
    5. Sensory loss in face [onion bulb distribution]
Ans. B and D
  • Vernet's syndrome → [IX, X, XI] intracranial involvement
  1. About myelination in PNS and CNS
    1. Schwann cell is responsible for myelination in PNS
    2. Oligodendrocyte myelinate about 60 axons in CNS
    3. Schimdt – Lantermann incisure is present in both CNS and PNS axons
    4. Ranvier's nodes absent in CNS axon
    5. Mesaxon is absent in PNS axon but present in CNS axon
Ans. A to C.
  1. Which of the following sensory sensations recover first after neurotemesis?
    1. Heat and cold sensation54
    2. Light touch and discrimination
    3. Poorly localized superifical pain
    4. Pain caused by deep pressure
Ans. D
  • Order of recovery of sensation after neurotemesis is pain caused by deep pressure followed by poorly localized superficial pain followed by heat and cold sensation and at last light touch and discrimination
  1. About relation of spinal cord segment
    1. T12 segment lies opposite to T9 vertebrae
    2. T8 segment lies opposite to T6 vertebrae
    3. C8 segment lies opposite to T1 vertebrae
    4. L5 segment lies opposite to T12 vertebrae
    5. Sacrococcygeal segment lies opposite to L1 vertebrae
Ans. A, B, D and E
  1. A patient having following values
    • SBP = 110 mmHg
    • DBP = 70 mmHg
    • ICP = 10 mmHg
      has cerebral perfusion pressure of
    1. 63 mmHg
    2. 73 mmHg
    3. 83 mmHg
    4. 93 mmHg
Ans. B.
  • CPP = MAP – ICP
  • = (DBP + SBP – DBP/3) – ICP
  1. About histological stains in neurology
    1. Osmic acid is used for staining myelin which are stained black
    2. Holtzer stain is used for showing gliosis
    3. Trypan blue is used for showing cerebral oedema
    4. Nissl (cresyl blue) is not used for staining myelin
    5. Nissl granule stain best with eosinophilic stain
Ans. A to D are correct
  • Nissl granule is stained basophilic55
  1. About seizures
    1. Hyperventilation is used for evocation of petitmal (Absence) seizures
    2. Photic stimulation is used for evocation of tonic and clonic seizures
    3. Sleep deprivation is used for evocation of partial complex seizures
    4. All of the above are true
Ans. D
  1. Hypokalemia is associated with
    1. Insulin excess
    2. Metabolic alkalosis
    3. Myocardial infarction
    4. Delirium tremens
    5. Hypothermia
Ans. All of the above are true
  • Refeeding syndrome: Hypokalemia may occur in the malnourished patient after the initiation of parenteral nutrition
  • Hypomagnesemia frequently accompanies hypokalemia and generally must be corrected to replenish K+.
  • Amphotericin B causes hypokalemia
  • Theophylline toxicity
  • RTA (Renal tubular acidosis) 1,2,3
  • Cushing Disease
  • Hyperaldosteronism (Conn's syndrome)
  1. Hyperkalemia is associated with
    1. Adrenergic receptor blockade
    2. Acute acidemia
    3. Digitalis intoxication
    4. Reperfusion of ischaemic limb
    5. Succinyl choline administration
Ans. All are true
  • Also seen in insulin deficiency, cell lysis (after chemotherapy), ACE inhibitors, K sparing diuretic non-steroidal anti-inflamnatory drug (NSAID)56
  1. Parts of the Ethmoid bone are
    1. Cristae galli
    2. Cribriform plate
    3. Superior nasal conchae
    4. Middle nasal conchae
    5. Inferior nasal conchae
Ans. A to D
  1. Anterior lacrimal crest is formed by
    1. Frontal process of maxilla
    2. Orbital plate of frontal bone
    3. Lacrimal bone
    4. Ethmoid
Ans. A.
  1. About genetics in neurosurgery
    1. Intron and exon both code for protein
    2. G2 phase is between S and M phase
    3. Protein synthesis occur during the GI phase
    4. G2 phase is most susceptible to radiation
    5. GI phase duration is variable in different tissues
Ans. B to E
  1. About Echinococcus infection
    1. Man is a definitive host
    2. Caused by T. solium
    3. Dog is secondary host
    4. Hydatid cyst in brain is dissected by hydraulic dissection
    5. Albendazole may be used in medical treatment
Ans. E
  • Echinococcus caused by encysted larvae of the dog tapeworm echinococcus granulose. Dog is the primary definitive host of the adult worm. Intermedaite hosts for the larval stage include sheep and man. Man is infected either by eating food contaminated with ova or by direct contact with dogs. CNS involvement occur in 3%. Produce cerebral cyst confined to white matter. Primary cysts are usually solitary. Secondary cysts usually multiple. CT density of the cyst similar to CSF does not enhance. Little surrounding edema contains 57germinating parasitic particles (400,000 scolices/ml). Medical treatment with albendazole 400 mg BD, Paed 15 mg/Kg/day × 28 days.
  1. About tremors
    1. It is an action of simple muscle and its antagonist
    2. In hyperthyroidism there are coarse tremors
    3. Fine tremors have frequency less than 10/s
    4. Multiple sclerosis can present with intention tremor
    5. In parkinsonism there is intention tremor
Ans. A.
  • Tremors are rhythmic, involuntary, oscillation of a body part. Tremors differ in distribution rate and amplitude whether they occur at rest or during voluntary contraction. Resting tremor is highly characteristic of Parkinsonism. Intention tremor or end point tremor implicates a lesion of cerebellum. Fine tremors have frequency greater than 10/s
  1. In multiple sclerosis
    1. Two lesions at the same site are diagnostic of MS
    2. In CSF IgM is raised
    3. Associated with retrobulbar neuritis and blindness
    4. Plaques are characteristically present in gray matter
    5. Trigeminal neuralgia can be present in MS
Ans. E
  • A demyelinating disease affects only white matter of the cerebrum, optic nerves and spinal cord. Produce multiple plaque of various ages in different locations in the CNS. Typically causes exacerbation and remissions in various locations in the CNS. Diagnosing MS after a single, acute remitting clinically isolated syndrome very risky. CSF oligoconal bands or CSF IgG (serum IgG must be normal). Support the diagnosis
  1. About immunoglobulin
    1. IgG is the only immunoglobin to cross the placenta
    2. IgD is related with complement fixation of the foreign body
    3. IgA is present in the body secretion
    4. IgE is related with type I hypersensitivity
    5. IgM is also related with complement fixation of the foreign body58
Ans. A, C, D and E are correct
  • IgG, IgA, IgM causes complement fixation
  1. Hypothalamic nucleus associated with water metabolism are
    1. Supra optic
    2. Ventromedial
    3. Arcuate nucleus
    4. Paraventricular
    5. Lateral nucleus
Ans. A and D
  1. Structures associated with auditory system are
    1. Inferior olivary nucleus
    2. Superior colliculus
    3. Lateral geniculate body
    4. Trapezoid body
    5. Spiral ganglion
Ans. D and E
  1. Cortex associated with olfaction are
    1. Lingual gyrus
    2. Cuneus
    3. Pyriform
    4. Precuneus
    5. Entorhinal
Ans. C and E
  1. About Bell's Palsy
    1. Hereditary, diabetes, pregnancy can be the cause
    2. It is the most common cause of facial nerve palsy
    3. More proximal involvement indicates good prognosis
    4. It is peculiar in case that chorda tympani is not involved
    5. All the above are true
Ans. A and B are correct.
  1. About Medulloblastoma
    1. Defect is on the short arm of chromosome 17
    2. Leptomeningeal spread is rare
    3. Prognosis is good if presentation is below 3 years and residual volume is less than 1.5 sqm59
    4. Chemotherapy is preferred over radiotherapy if age is less than 3 years
    5. After resection a CT or MRI must be done after 24 to 48 hrs
Ans. A and D
  • ErbB2 over expression in MB is associated with worse prognosis
  1. About CNS lymphoma
    1. Periventricular in position
    2. CNS lymphoma prognosis better than non CNS lymphoma
    3. CNS lymphoma is of Hodgkins type
    4. May disappear on CT after giving steroids
    5. Primary CNS lymphoma associated with EB virus
Ans. A, D and E
  • Are seen at all ages. PCL represents about 1to 2% of all intra cranial neoplasms and approx 1% of primary non-Hodgkins Lymphoma. PCL occur in about 2% of AIDS cases. PCL in immunologically normal patient has increased three fold in the last decade. Corticosteroids induce apoptosis in lymphoma cells, contributing to cytotoxic effects rather than reduction in cerebral edema. PCNSL is an unusual form of NHL that is restricted to CNS. Poor prognostic factors in PCNSL are Age > 60, elevated serum LDH. Elevated CSF protein. Location of tumor in deep structures
  1. About VHL syndrome
    1. Autosomal recessive inheritance
    2. Ocular retinal angioma
    3. Associated with glioma, meningioma
    4. Defect on short arm of chromosome 3
    5. Associated with pheochromocytoma
Ans. B, D and E
  • It is an autosomal dominant disorder with incomplete penetrance. Defect on the short arm of chromosome 3 common lesions. Retinal angiomas 40 to 50%. Hemangioblastomas (40 to 80%). Cerebellum 75% of hemangioblastomas in VHL. Spinal cord 25% of hemangioblastomas in VHL. Renal cell carcinoma, Pheochromocytoma (10%)60
  1. About Milard Gubler syndrome
    1. Lesion in midbrain
    2. Ipsilateral VIth nerve palsy
    3. Contralateral hemiparesis
    4. Contralateral nuclear VII palsy
    5. Contralateral weakness of facial muscle
Ans. C
  • Facial and abducens palsy and contralateral hemiplegia (corticospinal tract) from lesion in base of pons usually ishemic infarct, occasionally tumor.
  1. About branches of Internal carotid artery
    1. Lingual artery
    2. Posterior communicating artery
    3. Ascending pharyngeal artery
    4. Middle meningeal artery
    5. Middle cerebral artery
Ans. B and E
  • Branches of ICA are ophthalmic artery, anterior choroidal artery, posterior communicating artery, superior hypophyseal artery, meningo hypophyseal trunk, terminal branches, anterior cerebral artery and middle cerebral artery
  1. Muscle supplied by radial nerve
    1. Tricep
    2. Brachioradialis
    3. Abductor policis longus
    4. Brachialis
    5. Posterior part of Deltoid
Ans. A to D
  1. Lesion of temporal lobe produces
    1. Anton's syndrome
    2. Balint's syndrome
    3. Dysconnection syndrome
    4. Object agnosia
    5. Complex partial seizure
Ans. D and E61
  1. About Dandy Walker syndrome
    1. Agenesis of Vermis
    2. Dilatation of IVth ventricle
    3. Small posterior fossa
    4. High torcula
    5. Atresia of Foramen Magendie and Luschka
Ans. A, B, D and E
  • Atresia of foramina of Magendie and Luschka result in agenesis of the cerebellar vermis with a large posterior fossa cyst communicating with an enlarged IVth ventricle. Some retrocerebellar arachnoid cyst mimic Dandy Walker. Do not have vermian agenesis and the cyst does not open into IVth ventricle. Vallecula is absent in Dandy Walker syndrome. Hydrocephalus occur in 90% of cases. Dandy Walker malformations is present in 2–4 % of all agenesis of the corpus callosum in 17%. And Occipital encephalocele is 7%, if no hydrocephalus patient may be followed. If necessary shunt the posterior fossa cystin rare patient with aqueductal stenosis lateral ventricle is also shunted
  1. About nuclear IIIrd nerve palsy
    1. Ipsilateral ptosis
    2. Ipsilateral medial rectus palsy
    3. Contralateral superior rectus palsy
    4. Bilateral dilated pupil
    5. Ipsilateral intorted eye
Ans. B to E
  1. About upper motor neuron Vth nerve palsy
    1. Bilateral UMN supply to motor nucleus
    2. Unilateral UMN lesion leads to deviation of jaw to the side of the lesion
    3. Bilateral UMN lesion leads to absence of jaw jerk
    4. UMN lesion is often less harmful
Ans. A and D
  1. The most common artery and lobe to be involved in AVM are
    1. Middle cerebral artery and Parietal lobe
    2. Anterior cerebral artery and Frontal lobe
    3. Posterior cerebral artery and Occipital lobe
    4. Middle cerebral artery and Parietal lobe62
Ans. A
  • MCA and Parietal lobe: The parietal lobe is the commonest region involved in the supratentorial lesions. There is no significant hemispheric preference. Middle cerebral artery is the most common artery to be involved in AVM
  1. Tract present in the inferior cerebellar peduncle are
    1. Dentatorubral
    2. Posterior Spinocerebellar
    3. Anterior Spinocerebellar
    4. Darsal external arcuate
    5. Vestibulocerebellar
Ans. B, D and E.
  1. About Valsalva's maneuver
    1. It is a cardiovascular reflex
    2. It is expiration against closed glottis
    3. It is used to expirate cyst from the brain
    4. It can be used to measure raised ICP
    5. It is used to check post-operative hemostasis in intracranial surgery
Ans. All of the above are true
  1. About Cerebello Pontine Angle which of the following statements is true?
    1. No vein is found in Cerebello Pontine angle
    2. Schwannomas are rare in CP angle
    3. Meningiomas constitute the most common CP angle tumor
    4. Menigiomas constiute about 10% of all Cerebello Pontine Angle tumor
Ans.D
  1. Structure present in cell body (soma) are
    1. Nissl substance
    2. Lysosomes
    3. Golgi complex
    4. Neurofilament
    5. Microfilament
Ans. All of the above are true63
  1. Principal ligaments stabilizing atlanto-axial joints are
    1. Cruciate ligament
    2. Transverse ligament
    3. Alar ligament
    4. Ligamentum flavum
    5. Ligamentum nuchae
Ans. A, B and C
  • The principal stabilizing ligments of CI are the transverse atlantal and alar ligaments. The secondary stabilizing ligaments are more elastic and weaker than the primary ligaments and include the apical ligament, anterior and posterior atlanto-occipital membranes, tectorial membrane, ligamentum flavum, anterior and posterior longitudinal ligament and the capsular ligaments. The atlas and axis are united by the cruciform ligament, ALL, PLL and capsular ligament. The cruciform ligament has transverse and vertical component. Transverse component is called transverse ligament is the strongest stabilizer of CI and allows the atlas to pivot around the dens. Vetical component called paired alar ligament. Prevent excessive rotation and 1 ateral bending of CI.
  1. Posterior fossa AVM account for how many percentage of all parenchymal AVMs
    1. 4%
    2. 8%
    3. 15%
    4. 24%
Ans. C 15%
  • 85 % of all pial AVMs are found in the cerebral hemispheres and 15% occur in the posterior fossa. Although they can be found in virtually any location, the typical parenchymal AVM extends from the subpial surface of the brain through the cortex and underlying white matter. AVMs are often shaped like a cone with its base on the cortex and apex pointing towards the ventricle
  1. All are true about traumatic subdural hygroma EXCEPT
    1. Contain prealbumin
    2. Lack membranes
    3. Density of fluid is similar to that of CSF
    4. Most common site is inter hemispheric fissure64
Ans. D.
  • Most likely location of arachnoid tears are in the sylvian fissure or the chiasmatic cistern.
  1. Hemangioma Calcificans term is used for
    1. Venous Angioma
    2. Cavernous Angioma
    3. Capillary Telangiectasia
    4. AVM
Ans. B.
  • Heavily calcified cavernoma. Term given by Penfield and Ward
  1. Drugs used for spasticity are
    1. Benzodiazepines
    2. Baclofen
    3. Tiazidine
    4. Hyaluronidase
    5. Phenothiazine
Ans. All of the above are true
  • Drugs used for spasticity are diazepam activate GABAa receptor, increases presynaptic inhibition of alpha motor neuron. Useful in patients with complete spinal cord injuries. Baclofen activates GABAb receptor. Dantrolene reduce depolarization induced calcium influx into srcoplasmic reticulum. Act on all skeletal muscles. Progabide activates both GABAa and GABAb receptor. Useful for severe flexor spasm. Phenothiazine reduce gamma spasticity. Tiazidine and hyaluronidase are also used.
  1. Drugs used to treat neurocysticercosis are
    1. Albendazole
    2. Metronidazole
    3. Praziquentel
    4. Niclosamide
    5. Pyrantal Pamoate
Ans. A and C
  • Praziquentel and Albendazole are used to treat neurocysticercosis. Praziquantel given as 50mg/kg divided in 3 doses for 15 days. Significant reduction in symptoms and 65in number of cysts seen on CT. Also drug of choice for intestinal stage infestation. Albendazole 15mg/kg per day in 2–3 dose is given for 3 months. Niclosamide may be given orally to treat adult tapeworms in the GI tract. Praziquentel is drug of choice.
  1. About Vascular malformation all are true EXCEPT
    1. Venous angioma is the most common of vascular malformation
    2. Cavernous angioma lack neural parenchyma between the vessels
    3. Venous angioma are low flow and low pressure
    4. Cavernoma readily respond to radiation
Ans. D.
  • Venous angioma represents the venous drainage of the area. Neural parenchyma is between the vessels. Most common in regions suppled by the MCA or in the region of Vein of Galen. These are low flow – low pressure. Surgery is indicated only for documented bleeding or for intractable seizures. Cavernoma are well circumscribed benign vascular hamartoma, consisting of irregular thick and thin walled or large draining vein. Do not respond to radiation including stereotactic surgery.
  1. Complications of overshunting are
    1. Slit ventricles
    2. Orthostatic hypotension
    3. Subdural hematomas
    4. Stenosis of sylvian duct
    5. Craniosynostosis and microcephaly
Ans. All of the above are true
  • Complications of overshunting are self explainable. It is due to over-drainage of CSF. High pressure shunt or anti siphon device should be applied
  1. GVE fibres are present in cranial nerves
    1. III
    2. IV
    3. VI
    4. IX
    5. X
Ans. A, D and E66
  1. A 54 year old patient presents with diplopia. All are true EXCEPT
    1. The presence of ptosis, and divergent squint is suggestive of a lesion of cranial nerve III
    2. If diplopia is present when reading or looking down, then a lesion of cranial nerve VI is likely
    3. The pupil is likely to be spared in diabetic lesion of cranial nerve III
    4. Fluctuating diplopia is suggestive of myasthenia gravis
    5. If caused by an acute palsy of cranial nerve III, and associated with acute severe headache and signs of menningism, then a posterior communicating artery aneurysm should be excluded
Ans. B
  1. In Foville's syndrome there is
    1. VII nerve paresis
    2. Ipsilateral paralysis of conjugate gaze
    3. Contralateral corticospinal hemiplegia
    4. Horner's syndrome
    5. None of the above
Ans. A to D
  1. In Melkerson–Rosenthal syndrome there is
    1. Recurring facial palsy
    2. Recurring facial edema
    3. Lingua plicata
    4. Familial begin in childhood
    5. Cause is unknown
Ans. All of the above are true
  1. In Encephalotrigeminal angiomatosis (Sturge-Weber syndrome) there are
    1. Congenital nevi or angiomas
    2. Ipsilateral leptomeningeal angioma
    3. Intra cortical calcification
    4. Cerebral hemiatrophy
    5. Ipsilateral hemiparesis and focal convulsions
Ans. A to D
  • Localized cerebral cortical atrophy and calcification especially cortical layers 2 and 3 with a predilection for the occipital 67lobe. Calcification appear as curvilinear double parallel lines tram tracking. Cortical atrophy cause contralateral hemiparesis and homonymous hemianopsia occipital lobe involvement. Ipsilateral portwine facial naevus (nevus flammeus) in distribution of V1. Convulsive seizures contralateral to the facial nevus and cortical atrophy. Most cases are sporadic. Reccesive inheritance with chromosome 3 is being implicated. Treatment is supportive
  1. In primary optic atrophy
    1. Disk paler than normal and somewhat smaller
    2. Opaque white or a blue white color
    3. Disk margin district
    4. Physiologic cup increased in depth and size
    5. Lamina cribrosa is prominent and may extend to the margin of the disk
Ans. All of the above are true.
  • In primary optic atrophy, the disc is paler than normal, more sharply demarcated from the surrounding retina. Physiologic cup abnormally prominent and extend to the margin of the disc. Appear perceptibly smaller. Bowtie or band optic atrophy refers to pallor of the disc that may develop in an eye with temporal visual field loss, following a lesion of the optic chiasma or tract
  1. Causes leading to primary optic atrophy
    1. Multiple sclerosis
    2. Toxic amblyopia
    3. Diabetes
    4. Hereditary optic atrophy
    5. Paget's disease
Ans. All of the above are true
  • Pressure on the optic nerves is due to pituitary tumors, craniopharyngiomas, meningiomas
  1. About testing of vision
    1. Vision which can be improved by looking through a pin hole usually can be improved by glasses
    2. In field defects due to neurologic disease, red is usually lost first (Red desaturation)68
    3. In disease of the choroid the field for blue lost first
    4. The field of vision is wider in the inferior and lateral quadrant than in the superior and lateral quadrant
    5. The tangent screen is valuable in measuring the size of physiologic blind spot and in demonstrating central defect
Ans. All of the above are true
  1. About visual field changes
    1. Concentric contraction frequently seen is an early objective finding in optic atrophy either primary or secondary
    2. Tubular contraction regarded as sign of hysteria
    3. Spiral contraction is probably more diagnostic of fatigue
    4. Wernicke's hemianopic phenomenon localize the lesion anterior to LGB
    5. In tract lesions, the hemianopia is usually incongruous
Ans. All of the above are true
  • If the lesion is posterior to the lateral geniculate body within the optic radiation, the light reflex is not lost and defect is usually congruous
  1. About visual field defect
    1. The most frequent site of the lesion responsible for a homonymous defect is occipital lobe
    2. Field defect in vascular lesion has steep or abrupt margin
    3. Field defect associated with neoplasms are often complete, gradually progressive and have sloping margin
    4. Absence of optokinetic nystagmus toward the hemianopic side is found in parietal lobe involvement
    5. The most common cause of the bitemporal hemianopia is a pituitary adenoma
Ans. All of the above are true
  1. About scotomas
    1. Positive scotomas are not regarded as true scotomas
    2. A central scotoma is characterized by blindness limited to the area of the visual field
    3. A caeco-central scotoma involves macular area and blind spot
    4. In annular or ring scotomas loss of vision surrounding the center of visual field
    5. None of the above
Ans. A to D69
  1. Ectopic ACTH production is seen with
    1. Small cell carcinoma of lung
    2. Thymoma
    3. Carcinoid tumor
    4. Pheochromocytoma
    5. Medullary thyroid carcinoma
Ans. All of the above are true
  1. Following are opioid peptides
    1. Dynorphin
    2. Alpha MSH
    3. Endorphin
    4. Glycine
    5. GABA
Ans. All of the above are true
  1. About Pituitary adenoma
    1. It comprises about 10–15% of all intracranial tumors
    2. Initially it produces bitemporal superior quadrantopia
    3. Mass effect is most likely due to non functioning tumor
    4. Among functioning tumors ACTH is likely to cause mass effect early
    5. Hyperprolactinemia may be present due to pressure on pituitary stalk
Ans. All of the above are true
  1. Cerebral vasodilatation is due to increase extracellular component EXCEPT
    1. KT
    2. HT
    3. NaT
    4. Adenosine
    5. PaCO2
Ans. C
  • Following result in cerebral vasodilation
    ↑Extracellular K+
    ↑Extracellular H+
    ↑Adenosine
    ↑PaCO2
    ↓PaO270
  1. VIth nerve is involved in
    1. Claude's syndrome
    2. Benedikt's syndrome
    3. Foville's syndrome
    4. Millard Gubler syndrome
    5. Gardenigo's syndrome
Ans. C to E
  • Nothnagel's syndrome – Injury to the superior cerebellar peduncle causes ipsilateral oculomotor palsy and contralateral cerebellar ataxia
  • Benedikt's syndrome – Injury to the red nucleus result in ipsilateral oculomotor palsy and contralateral tremor, chorea and athetosis
  • Claude's syndrome – Superior cerebellar peduncle and red nucleus are involved
  • Weber's syndrome – Injury to the cerebral peduncle causes ipsilatreal oculomotor palsy with contralateral hemiparesis
  • Foville's syndrome – Following dorsal pontine injury includes lateral gaze palsy, ipsilateral facial palsy and contralateral hemiparesis (AICA)
  • Millard Gubler syndrome – Ventral pontine injury. Involves Lateral rectus palsy ipsilateral only. No gaze palsy. Abducens fascicle is injured rather than nucleus
  • Gardenigo's syndrome – Pterous apex, mastoiditis can produce deafness, pain and ipsliateral abducen's palsy
  1. Downbeat nystagmus occur in
    1. Chiari malformation
    2. Basilar invagination
    3. Brainstem stroke
    4. Cerebellar stroke
    5. Lithium intoxication
Ans. All of the above are true
  • Also syringobulbia
  • Downbeat nystagmus occurs from lesion near the cranio cervical junction (or cervicomedullary junction)
  • Upbeat nystagmus is associated with damage to the pontine tegmentum71
  1. About papilledema
    1. Transient visual obscurations is a classic symptom
    2. Visual acuity is never affected by papilledema
    3. Visual field testing shows increased blind spot and peripheral constriction
    4. Papilledema can be present in spinal neurofibroma
    5. Papilledema also can be present in GB syndrome
Ans. A, C to E
  • Increased protein in CSF in neurofibroma and GB syndrome result in papilledema
  1. Following structures derived from the neural crest are
    1. Neuronal perikarya in the PNS
    2. Axons of ventral roots
    3. Axons of dorsal roots
    4. Autonomic ganglia
    5. Adrenal medulla
Ans. A, C to E
  • Much of the PNS drives from the neural crest. The crest derivative includes the dorsal root ganglia and their proximal and distal sensory processes, the sympathetic and parasympathetic ganglia and the adrenal medulla. The axons of the ventral root arise from the ventral motor neuron of the gray matter of the brainstem and spinal cord.
  1. According to the theory of nerve components, the white sympathetic rami contain
    1. GSE Axons
    2. GVE Axons
    3. GSA Axons
    4. GVA Axons
    5. All of the above
Ans. B
  1. The sublenticular and retrolenticular parts of the internal capsule conveys.
    1. Geniculo calcarine tract
    2. Temporal radiation
    3. Occipital radiation
    4. Posterior thalamic radiation
    5. Auditory radiation
72
Ans. All of the above are true. Sublentiform part consist of auditory radiation, Meyer's loop of optic radiation, Temporo-pontine and parietopontine fibres. Retropentiform part consist of optic radiation, posterior thalamic radiation, parietopontine and occipitopontine fibres.
  1. Match procedure or operation with the neurotransmmitter that it would deplete
    1. Destruction of spinal interneurons → GABA and glycine in the spinal cord
    2. Section of dorsal roots → substance P in the dorsal horns
    3. Section of the medial forebrain bundle → serotonin and catecholamine in the cerebral cortex
    4. Destruction of the anterior perforated substance → Acetyl choline in cerebral cortex
    5. Destruction of the medullary raphe → serotonin in the spinal cord
Ans. All of the above are true
  1. Cerebellar glomeruli contains
    1. Mossy fiber rosettes
    2. Glial capsule
    3. Dendrites of granule neurons
    4. Axons of Golgi neurons
    5. Dendrites of Golgi neurons
Ans. All of the above are true
  1. Reduction in pain responses can be produced by
    1. Stimulation of the periaqueductal gray matter
    2. Anterolateral cordotomy
    3. Section of the lateral division of the dorsal roots
    4. Section of the medial division of the dorsal roots
    5. Cingulotomy
Ans. A, B, C and E. D is false
  1. Section of both optic nerves have the following effects on calcarine cortex
    1. Decrease in capillarity
    2. Loss of the outer stripe of Ballinger
    3. Degeneration of cortical neurons
    4. Compensatory overgrowth of oligodendroglia
    5. All of the above
73
Ans. A to C.
  • Transection causes trans-synaptic degeneration of the geniculate bodies and the geniculocalcarine tract. Degeneration of this tract would deprive the calcarine cortex of its thalamic afferents and causes the outer line of Ballinger to disappear. Because of the loss of neurons, the capillarity decreases
  1. Neuronal perikarya assume the following pattern of organization
    1. Lamination
    2. Nuclear masses
    3. Reticular formation
    4. Ganglia
    5. None of the above
Ans. A to D
  • Neuronal perikarya arrange themselves in basically four patterns laminae, nuclear masses, reticular formation, ganglia. Each of these arrangement may show a wide variety of neuronal type
  1. Thalamic fasciculus conveys which of the following pathways
    1. Medial leminiscus
    2. Dentatothalamic tract
    3. Pallidothalamic tract
    4. Thalamo frontal radiation
    5. All of the above
Ans. A to C
  • Field of Forel H, or the thalamic fasciculus conveys the medial leminiscus, dentatothalamic tract and the pallidothalamic tract. It separates the ventral surface of the thalamus
  1. Major sites at which large number of neurons lodge or accumulate after migration including the
    1. Basis pontis
    2. Cerebellar cortex
    3. Olivary nuclei
    4. Neurohypophysis
    5. Diencephalic nuclei
    74
Ans. A to C
  • Neuroblasts that remain in the periventricular zone form the gray matter surrounding the cavities of the CNS—namely, the nuclei of the spinal cord, the somite nuclei of the brainstem, the diencephalic nuclei and the basal ganglia. The migratory nuclei forms the cortex in the cerebrum and cerebellum and also form the branchial nuclei and reticular formation that surrounds the periventricular nuclei core of the brainstem, the nuclei of the basis pontis and the inferior olivary nuclei.
  1. Clinical deficit with the expected lesion site
    1. Expressive (fluent) aphasia → Left posterior parasylvian area
    2. Loss of recent memory → Inferior medial quadrants of temporal lobes
    3. Gerstmann syndrome → Area 39 (angular gyrus) left side
    4. Prosopagnosia → Infer Medial Inferior Temporooccipital Region
    5. Astereognosis → Right posterior parasylvian area
Ans. A to D
  1. Statements regarding fissure or space are
    1. Invagination of the medial hemispheric wall with rolling in of the hippocampal formation creates the transverse cerebral fissure
    2. The transverse cerebral fissure is continued as the choroid fissure with the hippocampal formation forming its floor in the temporal lobe
    3. Corpus callosum constitute the immediate roof of the cavum veli inter positi
    4. The roof of the third ventricle is the floor of the cavum.
    5. Cavum septi pellucidi lateral walls are formed by the septum pellucidum
Ans. All of the above are true
  1. About weakness of muscle
    1. Spasticity is velocity dependent
    2. Spasticity predominantly affects antigravity muscles
    3. Rigidity affects flexor and extensor equally
    4. Paratonia result from the disease of frontal lobe
    5. Paratonia is present throughout the range of motion and affects flexor and extensor equally
Ans. All of the above are true75
  1. About localization of sensory loss
    1. Harlequin pattern of sensory disturbance localize lesion to the lateral medulla
    2. In tegmentum of pons and midbrain lesion cause pansensory loss on the contralateral body
    3. Hemisensory disturbance with tingling numbness from head to foot is thalamic in origin
    4. In parietal cortex lesion there is contralateral hemineglect, hemi-inattention and tendency not to use the hand and arm
    5. Anterior parietal infarction may present as a pseudothalamic syndrome with crossed hemilateral loss of primary sensation
Ans. All of the above are true
  1. Akinetic mutism occur in damage of
    1. Medial thalamic nuclei
    2. Pulvinar nuclei
    3. Orbitofrontal surface of frontal lobe
    4. Due to hydrocephalus
    5. All of the above
Ans. All of the above are true
  • Bilateral asterixis – signifies a metabolic encephalopathy hepatic failure
  • Myoclonic jerks or tremors are typical of uremic encephalopathy antipsychotic drugs such as lithium, phenothiazines or butyrophenones
  • Myoclonus with coma signify anoxic cerebral damage
  • Akinetic mutism refer to a partially or fully awake patient who is able to form impression and think but remain immobile and mute, particularly when unstimulated
  • The term abulia is used to describe a mental and physical slowness and lack of impulse to activity that is in essence, a mild form of akinetic mutism with the same anatomic origins
  • Catatonia→ Curious hypomobile and mute syndrome
  • Patient appear awake with eyes open but make no voluntary or responsive movement.
  • They blink spontaneously, swallow and may not appear distressed
  • Eyelid elevation is actively restricted
  • Blinking occur in response to visual threat and the eyes moves concomitantly with head rotation76
  • Waxy flexibility
  • Locked in state → Pseudocoma
  • Verical eye movement and lid elevation remain unimpaired
  • Infarction or hemorrhage of the ventral pons which transect all descending corticospinal or corticobutblar pathways is the usual cause
  • A similar awake but different state occur as a result of total paralysis of the musculature in severe cases of GB syndrome
  1. About staining
    1. Basic aniline dyes, such as hematoxylin demonstrates neuronal and glial perikarya but do not show dendrites, axons or myelin sheath
    2. Ehrlich homatoxylin and eosin stain is an all purpose stain
    3. Luxol fast blue and iron hematoxylin selectively stains myelin
    4. Osmium is deposited on cell membranes for electron microscopy and silver is deposited on cell membranes for light microscopy
    5. Astrocytes can be demonstrated by Hortega silver impregnation method.
Ans. All of the above are true
  • Normal axons – Bodian protagorol method Ramon Y cajal Neurofibillary impregnation Hortega neurofibrillary impregnation
  • Degenerating axons – Nauta–Gygax silver impregnation method
  • Astrocyles - Ramon Y cajal gold chloride sublimate Hortega silver impregnation method
  • Microglia/oligodendroglia – Hortega silver impregnation method
  • Entire individual neuron – Golgi silver impregnation method
  • Reticulin silver impregnation for reticulin
  • The Marchi method → osmic acid reacts with degenerating myelinated fibres so that they show up against unstained background, giving a positive trail of the degenerating tract
  • Iron – hematoxylin method shows the surrounding intact myelinated fibre while the tract in question stand out as an unstained void giving a negative trail of the tract77
  1. Statement about glia are
    1. Astrocytes provide a structural scaffolding for neurons
    2. Oligodendrocytes produce myelin sheath
    3. Microglia are one type of perineuronal satellite cell
    4. Ependymal cells are highly branched
    5. Glia serves in place of the fibrous connective tissue of other organs
Ans. All of the above are true
  1. About cells in the nervous system
    1. In Golgi type I neuron, the axonal volume may greatly exceed the volume of the perikaryon
    2. Neurons classified by axonal length are amacrine neurons, Golgi type I and Golgi type II neurons
    3. The named tracts of the CNS consist of Golgi type II axons
    4. Perineuronal satellite cell that nourish neurons and protect their surface are astrocyte, oligodendroglia and microglia
    5. Glial cells that have multiple branches are astrocytes, oligodendrocytes and microglia
Ans. A, B, D and E
  • Named tract contain Golgi type I axons
  1. The following structures develop by evagination
    1. Optic nerve
    2. Neurohypophysis
    3. Cerebral hemispheres
    4. Cerebellum
    5. Basis pontis
Ans. A to C
  1. Following structures have glial supporting tissues
    1. Optic nerve
    2. Neurohypophysis
    3. Olfactory tract
    4. Autonomic ganglia
    5. Dorsal root ganglia
Ans. A to C
  • All structures that develop as evagination from the wall of the forebrain tube have glial supporting tissues, since they are basically CNS tissues. These structures include the optic nerve, infundibulum, olfactory tract and pineal body78
  1. Statements about nerve sheaths are
    1. The nerve sheaths strongly resist compression
    2. Perineural fibres mainly orient circumferentially
    3. The perineural sheath divides the nerve into facicles
    4. Endoneurium forms the blood-nerve barrier
    5. Epineurium is continuous with the dura matter
Ans. All of the above are true
  • The sheaths of the peripheral nerves have different reactions to disease. The endoneurium consisting of connective tissue fibre, is essentially a longitudinal orientation that undergoes proliferation after loss of nerve fibres
  1. Branches or connection listed for the following major vessels
    1. Anterior cerebral artery/Heubner's artery of medial striate group
    2. Middle cerebral artery/lateral striate arteries
    3. Galenic vein/inferior longitudinal sinus
    4. Basilar artery/PICA
    5. Carotid artery/MCA
Ans. All of the above are true
  1. Field of Forel H1 (Thalamic fasciculus) convey pathways
    1. Medial leminiscus
    2. Pallidothalamic tract
    3. Dentatothalamic tract
    4. Lateral leminiscus
    5. All of the above
Ans. A to C
  • Numerous pathways run through the laminae of myelinated fibres designated as H Field of Forel. These include afferent to the thalamus from the pallidum, cerebellum and somatosensory systems. The fonix arches over the thalamus band run in the hypothalamus, ventral to the H Field of Forel.
  1. About anatomically defined areas and function are
    1. A perisylvian network for language
    2. A parieto frontal network for spatial orientation
    3. An occipito-temporal network for object recognition79
    4. A limbic network for retentive memory
    5. A prefrontal network for attention and comportment
Ans. All of the above are true
  1. About Wernicke's aphasia
    1. Due to lesion in posterior part of middle temporal gyrus
    2. An embolus to the inferior division of the middle cerebral artery and to the posterior temporal or angular branches is the most common
    3. A co-existing right hemi or superior quadrantanopia is common in patient with Wernicke's aphasia
    4. Mild right nasolabial flattening may be found
    5. All of the above are true
Ans. B to D.
  1. Other deficit associated with Broca's aphasia are
    1. Right facial weakness
    2. Hemiparesis
    3. Buccofacial aphaxia
    4. Occur due to occlusion of superior divison of the middle cerebral artery
    5. All of the above are true
Ans. E
  1. About pure alexia without agraphia
    1. Due to obstruction of left posterior cerebral artery
    2. The patient can understand and produce spoken language, name objects in the left visual hemifield, repeat and write
    3. Patient loses the ability to name colors
    4. Although they can match color
    5. Since the PCA also supplies medial temporal component of the limbic system, the patient with pure alexia may also experience an amnesia
Ans. All of the above are true
  1. In Balint's syndrome there is
    1. Oculomotor apraxia
    2. Optic ataxia
    3. Simultanagnosia
    4. The problem is central than peripheral
    5. Result from bilateral dorsal parietal lesions80
Ans. All are true
  • Balint's syndrome consists of psychic impairment of visual fixation and alteration in visual attention. The patient has inability to reach for objects using visual guidance despite normal visual acuity and intact visual field. Optic ataxia and inability to voluntarily direct gaze (optic apraxia). Seen in patients with B/L parietooccipital lesion
  1. Branches of cervical plexuses are
    1. Superior root (C1) and inferior root (C2 and C3) of the ansa cervicalis
    2. Lessor occipital nerve (C2)
    3. Greater auricular nerve (C2 and C3)
    4. Transverse cutaneous nerve of the neck (C2 and C3)
    5. Supraclavicular nerve (C3 and C4)
Ans. All of the above are true
  • Phrenic nerve (C3, C4, C5)
  1. Following statements about the denticulate ligament are
    1. A line drawn between the two ligaments bisects the spinal cord into right and left halves
    2. They consist of glia rather than collagen
    3. They attach directly to the vertebral bodies
    4. They travel along the dorsal roots
    5. All of the above are false
Ans. E.
  • The denticulate ligament attach to the lateral aspect of the spinal cord midway between the dorsal and ventral roots. They extend from the pia to the dura and are composed of fibrous connective tissues. Line bisects cord into dorsal and ventral half
  1. Lesion interfering with volitional control of micturition are
    1. Medial part of the motor area (area 4) in the interhemispheric fissure
    2. Transection of the ventral half of the spinal cord
    3. Pelvic splanchnic nerve
    4. Obturator nerve
    5. Pudendal nerve
Ans. A to C and E81
  • Efferent to the bladder travel in the dorsal portion of the lateral column of the spinal cord. The primary coordinating center for bladder function reside within the nucleus locus cerulus of the pons. Voluntarily cortical control primarily involves inhibition of pontine reflex and originate in the genu of the corpus callosum.
  1. Association between a nerve and its function is
    1. Median nerve/flexion of the wrist
    2. Ulnar nerve/abduction of the little finger
    3. Obturator nerve/thigh adduction
    4. Musculocutaneous nerve/elbow flexion by the biceps
    5. Common peroneal nerve/extension and flexion of the ankle
Ans. A to D
  1. Statements about spinal nerve are
    1. Each spinal nerve innervate one somite
    2. The nerve trunk is formed by the union of dorsal and ventral roots
    3. Each spinal nerves typically has one dorsal root ganglion
    4. Most spinal nerves contain parasympathetic efferent axons
    5. The spinal nerves convey axons to skeletal muscles
Ans. A to C and E
  • The spinal nerves typically contain four components—visceral and somatic efferent and visceral and somatic afferent. For most spinal nerves, the visceral efferent axons derives from the sympathetic nervous system. Only sacral nerves carry a large number of parasympathetic efferent axons.
  1. A patient complain of numbness and tingling in the thumb and index finger. A lesion responsible for this complaint are
    1. The radial nerve
    2. The lower trunk or medial cord of the brachial plexus
    3. The C6 dorsal root
    4. Nucleus ventralis posterior of the thalamus
    5. The post central gyrus
Ans. A, C to E82
  1. About Reticular formation
    1. It widely disperses axonal connections
    2. Its perikarya use a variety of different neurotransmitters
    3. It has heterogenous, multiple afferent connections
    4. Its output consists of a few discrete myelinated tract
    5. It extends from the cervicomedullary junction to the diencephalon
Ans. A to C and E
  1. Changes after destruction of both optic nerves are
    1. Pallor of the optic disks
    2. Dilated pupil non reactive to light
    3. Compensatory overgrowth of oligodendroglia in the retina
    4. Wallerian degeneration of the optic tract
    5. Transneuronal degeneration of the lateral geniculate bodies
Ans. All of the above are true
  • The retina normally does not contain myelinated axons and oligodendroglia
  • The myelin sheath of the optic nerve begin after the retinal axons pierce the lamina cribrosa
  • Degeneration of the geniculo calcarine tract deprive the calcarine cortex of its thalamic afferent and causes the outer line of Baillarger to disappear because it consists of myelinated fibres of thalamic origin
  1. About the Nodes of Ranvier
    1. A node marks the site of apposition of two adjacent schwann cell
    2. The theory of saltatory conduction presumes that the ionic flux of the nerve impulse occurs at the node
    3. The internodal distance corresponds to the length of an axon myelinated by one schwann cell
    4. The axons continue across the nodes without interruption
    5. Many more nodes occur along the smallest nerve fibres than the largest
Ans. A to D
  1. Dermatomal level matching the corresponding areas of the body
    1. C7/Middle finger
    2. T4/Nipple line
    3. T10/Umbilicus83
    4. L5/Bigtoe
    5. S1/Perianal region
Ans. A to D.
  • All somite derivative retain their original nerve root wherever they migrate
  1. Statements about the thalamic fasciculus (Field of Forel H1) are
    1. It conveys many myelinated axons
    2. It conveys thalamofrontal axons
    3. It conveys pallidothalamic axons
    4. It forms the dorsal boundary of the Zona incerta
    5. It conveys dentatothalamic axons
Ans. A to D.
  1. Statements about Apraxia
    1. Bilateral pyramidal tract interruption would preclude testing a patient for apraxia
    2. Language apraxia implies a left hemisphere lesion
    3. Lesion of the cortex and the thalaemo cortical circuits can cause apraxia
    4. Dyslexia is a form of aphasia apraxia
    5. Apraxia can affect common learned actions like dressing and walking
Ans. A to C and E.
  • Dyslexia is a defect in the language reception, thus it is not an apraxia which means the inability of a non-paralayzed patient to execute a volitional action
  1. About the corpus callosum
    1. Tapetum arises from the genu of the corpus callosum
    2. Its splenium connects cortex of the posterior part of the cerebral hemisphere
    3. The fornix and fornix commissure run along its ventral surface
    4. It conveys most of the projection fibres from the two cerebral hemispheres to lower centers
    5. Far fewer transcallosal axons connect the hand areas than the trunk areas of the sensorimotor cortex
Ans. B, C and E
  • Tapetum arise from the body of corpus callosum84
  1. About fissures
    1. Form earlier than sulci
    2. Are longer than sulci
    3. Are less numerous than sulci
    4. Are shallower than sulci
    5. Are not bordered by cortex
Ans. A to C and E.
  • Fissures form by evagination of the telencephalon of the hemisphere and of the temporal lobe
  • Longer and deeper than sulci
  • After evagination, sulcation occurs in a definite timetable sequence
  • The embryologic mechanism of sulcation are entirely different from fissuration
  • In lissencephaly, the fissure form but not the sulci
  • In holoprosencephaly, the fissure fail to form
  1. The following statement relating to the type of aphasia are true
    1. Motor aphasia/left posterior inferior frontal region
    2. Dyslexia/lateral surface of the left occipital lobe
    3. Global aphasia/left posterior parasylvian area
    4. Fluent aphasia/left posterior parasylvian area
    5. Auditory agnosia/left superior temporal gyrus
Ans. A, B, D and E
  1. Papez circuit includes
    1. Occipitofrontal fasciculus
    2. Fornix
    3. Ansa lenticularis
    4. Mammilothalamic tract
    5. Temporo ammonic tract
Ans. B to E
  • The amygdala connect with this circuit through the ansa lenticularis
  1. About Medial forebrain bundle
    1. It interconnects the basal telecephalic olfactory region with the hypothalamus and the brainstem
    2. It consists mainly of rapidly conducting myelinated axons
    3. It occupies the dorso lateral part of the hypothalamus85
    4. It conducts numerous ascending and descending sensory pathways
    5. It runs medial to the internal capsule
Ans. A, C to E.
  1. Following lesions reduce the afferent fibres to the deep cerebellar nuclei
    1. Transection of the inferior cerebellar peduncle
    2. Section of the auditory nerve
    3. Destruction of purkinje neurons
    4. Section of the superior cerebellar peduncle
    5. Destruction of the inferior olivary nuclei
Ans. A, C to E.
  1. Following are found in the molecular layer of the cerebellar cortex
    1. Dendrites of Purkinje neurons
    2. Dendrites of Golgi Type II neurons
    3. Cerebellar glomeruli
    4. Parallel and climbing fibers
    5. Stellate neurons
Ans. A, B, D and E
  1. Ocular actions mediated through the occipital lobes are
    1. Vergences
    2. Smooth pursuit
    3. Binocular fixation
    4. Saccadic kick backs
    5. Counter rolling of the eyes
Ans. A to C and E.
  1. Statements about the cortical efferent pathways for vertical eye movements are
    1. The vertical pathways loop down into the pons and back up to the nucleus of CN III in the midbrain
    2. The most axons of the vertical pathways end directly on the lower motorneurons of the optomotor nuclei
    3. The vertical pathways act unilaterally so that the left hemisphere pathways elevate the left eye, and the right hemisphere pathways the right eye86
    4. The vertical pathways run to all three optomotor nuclei, CN III, CN IV and CN VI
    5. The vertical pathways for upward movements run through the pretectum, separate from the pathway for downward movements
Ans. E.
  1. About Pia mater
    1. It is lined with squamous epithelium
    2. It belongs to the leptomeninges
    3. It dips into the crevices of the brain and spinal cord
    4. It intervenes between the arachnoid and the surface of the cerebral cortex
    5. It allows free permeability between the CSF and the CNS
Ans. A to D.
  1. Drugs used in Cushing's disease are
    1. Cyproheptadine
    2. Metapyrone
    3. Ketoconazole
    4. Mitotane
    5. Bromocriptine
Ans. A to D
  1. About Cerebello pontine angle
    1. Acoustic schwannoma are rare
    2. No vein is present in CP angle
    3. Posteromedially there is cerebellum
    4. Anteromedially there is pons
    5. Epidermoid cyst most common location
Ans. C to E.
  • Superior petrosal vein is present in CP angle
  1. Molecular agents inhibiting EGFR and being used to treat malignant glioma are
    1. Erlotinib
    2. Imatinib mesylate
    3. Cetuximab
    4. Bevacizumab
    5. Gefitinib
87
Ans. A, C and E Molecular agents targeting EGFR are ATP - binding site inhibitors Erlotinib, Gefitinib and lapatinib. Monoclonal antibodies include cetuxima VEGFR inhibitor include monoclonal antibody Bevacizuma PDGFR inhibitor include Imatinib mesylate and Dasatinib
  1. About Arachnoid cyst
    1. The most common site is interhemispheric fissure
    2. It is actually intra arachnoid cyst (between the arachnoid layers)
    3. It is associated with temporal lobe agenesis
    4. Bone expansion may be seen
    5. It can be followed by serial CT scan
Ans. B to E.
  • Most common site is sylvian fissure. Temporal lobe agenesis is now obsolete since brain volumes on each side are actually the same
  1. In primary optic atrophy
    1. Optic disc margins are blurred
    2. Optic disc margins are well defined
    3. Lamina cribrosa is well defined
    4. Disc is pale
    5. Disc is chalky white in colour
Ans. B, C and E
  1. In a 55 year old woman with a brain tumor all are false EXCEPT
    1. In the presence of a dressing apraxia, the lesion is likely to be in the dominant parietal lobe
    2. The presence of upper quadrant homonymous field defect suggests that the lesion is likely to be in the temporal lobe
    3. Sensory inattention is suggestive of dominant hemispheric lesion
    4. The presence of dyscalculia suggests a temporal lobe lesion
Ans. B
  • Constructional apraxias are localized to the non-dominant hemisphere. Sensory inattention and neglect are characteristic of non-dominant hemispheric lesions. Dyscalculia suggests dominant parietal lobe damage88
  1. Points distinguishing traumatic tap from SAH are
    1. Protein concentration will be greater in SAH than in traumatic tap
    2. SAH does not clot but traumatic tap clots
    3. Supertatene is clear in traumatic tap but xanthochromic in SAH
    4. Raio of WBC : RBC is elevated in SAH than in traumatic tap
    5. Opening pressure is usually elevated in SAH than in traumatic tap
Ans. All of the above are true
  • Fresh bleeding elevates CSF protein from normal by only = 1mg per 1000 RBC
  • In SAH blood breakdown product elevate this more than traumatic tap (measured protein exceeds the sum of normal protein + 1mg protein/1000 RBC)
  1. Pain sensitive structure in the vertebral body are
    1. Periosteum
    2. Dura
    3. Annulus fibrosus
    4. Epidural veins
    5. Posterior longitudinal ligament
Ans. A to C and E.
  • Also facet joints
  1. Drugs shown to reduce the risk of osteoporotic fracture are
    1. Biphosphonates
    2. Adendronate
    3. Tamoxifen
    4. Transdermal estrogen
    5. All of the above
Ans. A to D.
  1. About the brainstem
    1. The widest part of the brainstem is the midontine level
    2. The smallest cross sectional diameter is located at the caudal medullary level
    3. The roof plate of the caudal medulla consists of posterior medullary velum which is a thin membrane
    4. The narrowest site of communication between the ventricles is the aqueduct of the midbrain.89
    5. Largest indentation in the basis of the site of interpeduncular forsa is midbrain.
Ans. All of the above are true
  1. The central substrate of memory are
    1. Hippocampal gyri
    2. Ammon's horn
    3. Fornices
    4. Mammilothalamic tract
    5. Dorsomedial thalamic nuclei
Ans. All of the above are true
  • Bilateral inner circuit of the limbic system. Mammillary bodies, Anterior thalamic complexes cingulate gyri
  1. Flexion injuries are
    1. Wedge compression #
    2. Facet dislocation
    3. Hangman's #
    4. Atlas avulsion ring #
    5. Clay shover #
Ans. A, B and E.
  1. Gardenigo syndrome
    1. After suppurative otitis
    2. III, IV, V, VI Nerve palsy
    3. Cavernous sinus thrombosis
    4. Hearing Loss
    5. Sigmoid sinus thrombosis
Ans. A, C and E.
  1. Organism causing meningitis
    1. <4yr – H. influenzae
    2. Trauma → N. meningitides
    3. V P shunt → S. epidermidis
    4. CSF leak → S. pneumoniae
    5. <1month → Gram negative bacilli
Ans. A, C and E.90
  1. Subdural effusion in children
    1. Post VP shunt
    2. H. influenzae infection
    3. Trauma
    4. Arachnoid cyst rupture
    5. Congenital communication between ventricle and subdural space
Ans. All of the above are true
  1. Severe headache with 3rd nerve Palsy
    1. Cavernous sinus thrombosis
    2. Pituitary apoplexy
    3. Temporal abscess
    4. Ant. communicating artery aneurysm
    5. Intratumoral hemorrhage
Ans. All of the above are true
  1. Hormonal therapy is of proven benefit in
    1. Meningioma
    2. Pituitary adenoma
    3. Lymphoma
    4. Astrocytoma
    5. Hemangioblastoma
Ans. A and B.
  1. Cerebellar cortex
    1. Granule cell is inhibitory
    2. Golgi cell is excitatory
    3. Purkinje cell is excitatory
    4. Stellate cell is inihibitory
    5. Mossy and climbing fibres are excitatory
Ans. D and E
  1. Each of the following is true of decerebrate rigidity EXCEPT
    1. It results from tonic activity in the vestibulospinal and pontine reticulospinal neurons
    2. It is reduced by cutting dorsal roots
    3. It is reduced by destruction of the anterior lobe of cerebellum
    4. It occurs with transection between the colliculi
    5. There is increased gamma motor neuron activity
91
Ans. C
  • Destruction of the anterior lobe of the cerebellum releases cells of origin of the lateral vestibular tract from inhibition by Purkinje cells, thereby facilitating extensor motor neurons
  1. True about vagus nerve
    1. Supplies stylopharyngeus/Tensor veli palatani
    2. Somatosensory to esophagus
    3. Motor for parotid
    4. Supplies ext. auditory meatus
    5. Takes taste from epiglottic region
Ans. B, D and E.
  1. A patient present with LMN type facial paralysis with hyperacusis and loss of lacrimation, the lesion may be in
    1. Facial nucleus
    2. Stylomastoid foramen
    3. Geniculate ganglion
    4. CP angle
    5. Distal to N to stapedius
Ans. A, C and D
  1. Oligodendroglioma
    1. Never calcifies
    2. Common in parietal cortex
    3. May transform to glioblastoma multiform
    4. Very vascular
    5. It is an invasive tumor
Ans. C to E
  1. About acoustic schwannoma
    1. B/L tumors operated at the same time
    2. NF-1 with café au lait spots may be associated
    3. Autosomal dominant
    4. Not operated if hearing is intact
    5. Gamma knife can be an alternative
Ans. B, C and E.92
  1. Which of the following are intramedullary tumors?
    1. Spinal hemangioblastoma
    2. Ependymoma
    3. Meningioma
    4. Neurofibroma
    5. Astrocytoma
Ans. A, B and E.
  1. β adrenergic receptor function is
    1. Bronchoconstriction
    2. Vasodilation
    3. Increased myocardial contractility
    4. Lipolysis
    5. Glycogenolysis
Ans. All of the above are true
  1. The following have neural crest origin
    1. Dura
    2. Schwann cells
    3. Geniculate ganglion
    4. Spiral ganglion (Paravertebral)
    5. Visceral plexus
Ans. B to E.
  1. Communicating hydrocephalus is seen in
    1. SAH
    2. Arnold chiari malformation
    3. Choroid plexus papilloma
    4. Atresia of Foramen of Luschka and Magendie
    5. Anencephaly
Ans. A, B and E.
  1. About Diastatic fracture
    1. Linear fracture upto suture
    2. Depressed fracture
    3. Vascular line fracture
    4. Old age
    5. More common in younger children
Ans. E.
  • Diastatic fractures extend into and separate sutures.93
  1. Each of the following is true of Renshaw cells EXCEPT
    1. They are part of a negative feedback loop to the motor neurons.
    2. They facilitate Ia inhibitory interneurons that act on antagonist motor neurons
    3. They inhibit motor neurons that innervate synergist muscles
    4. They make divergent connections to motor neurons
    5. They receive input from decscending pathways
Ans. B
  • Renshaw cells inhibit 1a inhibitory interneurons that act on antagonist motor neurons
  1. About anesthetic agents
    1. Ketamine increases CBF and CRMO2
    2. Isoflurane induces seizure discharges
    3. Thiopental decreases CBF and CRMO2 and produces cardiovascular depression
    4. Etomidate decreases CBF and CRMO2 and suppresses adrenocortical repose to stress
    5. Enflurane is a volatile anesthetic and increases CBF the least
Ans. A, C and D.
  • Enflurane induces seizure dicharges. Isoflurane is a volatile anesthetic and increase CBF the least
  1. Progressive weakness distal to proximal paraparesis with no sphincter involvement with questionable tenderness at D6
    1. Ependymoma
    2. Neurofibroma
    3. Astrocytoma
    4. Meningioma
    5. Potts spine
Ans. B, D and E.
  1. Inclusion tumors with single tissue type
    1. Epidermoid cyst
    2. Embryoma
    3. Dermoid
    4. Lipoma
    5. Teratoma
Ans. A and D.94
  1. Posterior 3rd ventricular tumor with beta HCG in CSF are
    1. Chorio Ca
    2. Astrocytoma
    3. Embryonal Ca
    4. Yolk sac tumor
    5. PNET
Ans. A and C
  1. Most sensitive test to diagnose cavernous angioma is
    1. CT scan
    2. DSA
    3. T2WI MRI
    4. None of the above
Ans. C
  1. TIA of vertebral artery may present with
    1. Contralateral homonymous hemianopia
    2. Transient monocular blindness
    3. Hemisensory loss
    4. Ataxia
    5. Dysphagia
Ans. A, C to E.
  1. Early complication of intracranial aneurysm
    1. Thromboembolism
    2. Rebleeding
    3. Hydrocephalus
    4. Seizures
    5. Cisternal blockade
Ans. All of the above are true
  1. Brown Sequard syndrome is most common due to
    1. Intramedullary tumor
    2. Extradural tumor
    3. Intradural extramedullary tumor
    4. None of the above
Ans. C. i.e. Intradural extramedullary tumor95
  1. Hyperacusis will be present in the lesion in which of the following course or segment of the facial nerve
    1. Cisternal segment
    2. Meatal segment
    3. Labyrinthine segment
    4. Horizontal or Tympanic segment
Ans. D
  1. Two viruses which are genetically engineered to destroy tumor cells within the CNS and widely used in experimental study are
    1. Retro virus and Herpes virus
    2. Polio virus and Retro virus
    3. Parvo virus and Herpes virus
    4. None of the above
Ans. A
  1. Radio sensitizing agents are all EXCEPT
    1. BUdR
    2. IUdR
    3. 5-FU
    4. Interleukin-2
Ans. D.
  • Interleukin-2 is immune response modifier
  1. Xanthochromic CSF may be seen in
    1. Subarachnoid hemorrhage
    2. Some cases of SDH
    3. Gullian Barre syndrome
    4. Spinal block
    5. Acute pyogenic meningitis
Ans. All of the above are true
  1. About occipital eye field
    1. It is localized to a relatively small area
    2. It subserves pursuit eye movements that are largely voluntary
    3. Lesion in this area are associated with transient deviation of the eyes away from the side of the lesion96
    4. The threshold for excitation in this area is lower than it is in the frontal eye field
    5. With lesions in this area, the patient can direct the eyes to a particular location on command
Ans. E
  1. Fibres running in anterior white column are
    1. Anterior corticospinal tract
    2. Lateral corticospinal tract
    3. Vestibulospinal tract
    4. Ventral spinothalamic tract
    5. Fasciculus gracialis
Ans. A to D.
  1. About Finger flexion reflex
    1. Root value of reflex is C6 – T1
    2. Hoffman's reflex is due to extrapyramidal tract lesion
    3. Wartenberg's sign is positive when the thumb abduct and extends strongly
    4. Rossolimo's reflex significance is the same as the finger flexion reflex in the upper limb
    5. Wartenberg's sign and Hoffman's both are because of pyramidal tract disease
Ans. A, D and D
  • Hoffman's reflex is due to lesion in the pyramidal tract
  • In Wartenberg's reflex, thumb adduct and flex strongly (Pathological)
  1. Right sided homonymous superior quadrantopia occur in lesion of
    1. Right temporal lobe
    2. Left temporal lobe
    3. Right lingual gyrus
    4. Left cuneus gyrus
    5. Left lingual gyrus
Ans. B and E.97
  1. In carotid angiography under optimal conditions arteries upto what lumen diameter may be visualized
    1. .001 mm
    2. 0.1 mm
    3. 1.0 mm
    4. 2.00 mm
Ans. B.
  1. Statements regarding the eye movement due to stimulation of various areas are
    1. Stimulation of the caudal PPRF causes conjugate horizontal deviation to the same side
    2. Stimulation of the rostral PPRF causes conjugate horizontal deviation to the opposite side
    3. Stimulation of the superior colliculus causes conjugate horizontal deviation to the opposite side
    4. Stimulation of the middle frontal gyrus causes conjugate horizontal deviation to the opposite side
Ans. A, C and D
  • The center for horizontal gaze (the abducens nucleus) and the center for vertical gaze (the rostral interstitial nucleus of the medial longitudinal fasciculus, or RiMLF), are joined physiologically by the paramedian pontine reticular formation (PPRF), which lies rostral to the abducens nucleus. Stimulation of the caudal and rostral PPRF produces conjugate horizontal eye deviation and vertical eye movements respectively. Fibers from the caudal PPRF project to the ipsilateral abducens nucleus, while fibers from the rostral PPRF project uncrossed fibers to the RiMLF, which in turn projects to the ipsilateral oculomotor nuclear complex. Lesions of the caudal PPRF may cause paralysis of horizontal eye movements, while lesions of the rostral PPRF can cause paralysis of vertical eye movements. Stimulation of the frontal eye field, located in the caudal part of the middle frontal gyrus (area 8), usually results in conjugate deviation of the eyes to the opposite side. Stimulation of the superior colliculus results in contralateral conjugate deviation of the eyes98
  1. About Liliequists membrane (LM)
    1. Basically considered to separate the interpeduncular cistern from the chiasmatic cistern
    2. The superior leaflet of LM (diencephalic membrane) separates the interpeduncular cistern from the chiasmatic cistern medially and from the carotid cistern laterally
    3. The inferior leaflet (the mesencephalic membrane) separates the interpeduncular from the prepontine cistern
    4. The diencephalic membrane is thinner and is incompetent not effectively isolating chiasmatic cistern
    5. Blood in the carotid or prepontine cistern is compatible with a low–pressure pretruncal source of bleeding. However, blood in the chiasmatic cistern raise suspicion about aneurysmal rupture
Ans. A, C and E.
  • Diencephalic membrane is thicker and is more often competent, effectively isolating the chiasmatic cistern. The carotid cistern often communicate with the crural cistern and in turn with the interpeduncular cistern. The sellar segment of LM attaches inferiorly along the dorsum sella and superior diencephalic membrane attatches to mammillary body
  1. About different triangles related to approach to the intracavernous internal carotid artery
    1. Glasscocks triangle is posterior lateral triangle and this approach exposes posterior and lateral loop of ICA
    2. Kawases triangle is posteromedial and bounded laterally by GSPN and medially by Gasserian ganglion and base formed by pterous apex
    3. Parkinson's triangle approach exposes post genu of ICA and origin of meningohypophyseal trunk.
    4. Mullen's triangle is anterolateral triangle and exposes lateral loop of ICA
    5. Parkinson's triangle is also known as infra trochlear triangle
Ans. All of the above are true
  1. All of the following statements concerning sensory receptors are correct EXCEPT
    1. Nuclear bag fibers receive group IA primary afferent fibers (annulospiral endings) and static and dynamic gamma efferent fibers. They respond primarily to the rate of change of muscle length99
    2. Nuclear chain fibers receive group Ia primary and group II secondary afferent fibers (flower spray endings) and static gamma efferent fibers. They respond primarily to muscle length
    3. Pain and thermal receptors utilize group Ib fibers.
    4. Meissner corpuscles are found only in the dermal papillae of glabrous skin
    5. Pacinian corpuscles are acceleration detectors that respond to vibration sensation
Ans. A, B, D and E.
  • Pain and thermal receptors utilize group A delta (fast pain) and C (slow pain) fibers. Pain and thermal sensations are mediated by lateral spinothalamic tract
  1. In a 55-year-old woman with a brain tumor
    1. In the presence of addressing apraxia, the lesion is likely to be in the dominant parietal lobe
    2. The presence of upper quadrant homonymous field defect suggests that the lesion is likely to be in the temporal lobe
    3. Sensory inattention is suggestive of a dominant hemispheric lesion
    4. Altered personality and loss of initiative suggests that the lesion is likely to be in the frontal lobe
    5. The presence of dyscalculia suggests a temporal lobe lesion
Ans. B and D.
  • Constructional apraxia are localized to the non-dominant hemisphere. Temporal lobe damage causes visual field loss of the homonymous upper quadrant. Sensory inattention and neglect are characteristic of non-dominant hemisphere lesions. Personality changes are suggestive of frontal lobe damage. Dyscalculia suggests parietal damage
  1. A 66-year-old patient pesents with lower cranial nerve palsies
    1. A wasted and fasciculating tongue is suggestive of pseudobulbar palsy
    2. The jaw jerk is brisk in pseudobulbar palsy, but absent in bulbar palsy
    3. Motor neuron disease causes features of both bulbar and pseudobulbar palsy
    4. Speech is monotonous in bulbar palsy
    5. Emotional lability is suggestive of pseudobulbar palsy
100
Ans. C and E.
  • The gag reflex is absent in bulbar palsy. Speech is nasal in quality
  1. A 25-year-old man presented with a head injury
    1. Skull radiography is essential
    2. The risk of chronic subural hematoma is related to the severity of the head injury
    3. He is said to have had a concussion if only minor macroscopic brain damage has occurred
    4. Extradural hematoma is usually caused by a rupture of the sagittal or transverse sinuses
    5. The risk of post–traumatic epilepsy is increased if he develops an epileptic seizures in the first 24 hours after injury
Ans. E.
  • A CT scan or MRI may be more appropriate for severe head injury, mild head injury may not require investigation. Minor injuries can cause subdural hematomas. Concussion injuries show no macroscopic damage. Extradural hematoma is caused by tearing of the middle meningeal artery
  1. Best diagnostic method to diagnose post-traumatic CSF fistula
    1. Tomography
    2. Contrast enhanced CT
    3. Postitive ventriculography contrast
    4. Air ventriculography
    5. Intrathecal contrast ventriculography
Ans. E.
  1. Early sign of cerebral palsy is demonstration of
    1. Moro reflex at 2 months
    2. Moro reflex at 5 months
    3. Rooting reflex at 2 months
    4. Palmar grasp at age of 2 months
Ans. B.
  • Moro reflex at 5 months
  1. Each of the following has been associated with central pontine myelinolysis EXCEPT
    1. Alcoholism
    2. Rapid correction of hyponatremia101
    3. Serum hyper osmolaity
    4. Severe burns
    5. Vitamin A excess
Ans. E.
  • Vitamin A excess is not associated with central pontine myelinolysis
  1. About Action potential
    1. Inhibitory postsynaptic potentials are caused by increasing membrane permeability to cations e.g. Na
    2. Temporal summation occurs only when there is transmission at many synapses
    3. There are few Na channels at the axon hillock
    4. Conduction velocity m/s in myelinated axons is six times the diameter
    5. After demyelination, axonal membranes can store more charge
Ans. D and E.
  • Cation entry causes depolarization and an excitatory postsynaptic potential. Spatial summation occurs with simultaneous transmission from many synapses. There is a high concentration of Na channel at the axon hillock as this is here the action potential is generated. Axonal membranes have an increased capacitance after demyelination so can store more charge
  1. Regarding neurotransmission
    1. There is a single receptor identified for glutamate
    2. Muscarinic acetylcholine receptors are associated with a cation channel.
    3. Ionotrophic receptors are generally G-protein linked
    4. An inhibitory postsynaptic potential is created by increasing the membrane permeability to chloride ions
    5. Saltatory conduction is typically bidirectional
Ans. D.
  • There are several receptors for glutamate including NMDA, kainite and AMPA. Nicotonic receptors are associated with a cation channel. Metabotrophic receptors are generally G-protein linked. Inhibitory postsynaptic potential hyperpolarize the cell. Under physiological conditions, salutatory conduction is unidirectional102
  1. Parkinson's disease
    1. Is always an obvious diagnosis
    2. Should be treated immediately with L-dopa
    3. Is an upper motor neuron disorder and therefore associated with increased tendon reflexes
    4. Symptoms may respond to anticholinergic drugs
    5. Is associated with depression
Ans. D and E
  • Signs may be very subtle. The longer the patient can manage without drugs, the better as tolerance to L-dopa soon develops. Motor neurons are not affected. Anticholinergics inhibit striatal output cells. Depression may aggravate symptoms
  1. About primary and secondary GBM following are true EXCEPT
    1. Mutations in the p53 protein are found in more than 65% of secondary GBM
    2. Unlike secondary GBMs, primary GBMs infrequently display mutation in P53 (< 10%)
    3. More than 50% of primary GBMs show MDM2 protein over expression compared with only 10% in secondary GBM.
    4. P16ink 4a is deleted significantly in secondary GBM than primary GBM
Ans. D. P16ink 4a is deleted significantly more in primary GBMs than secondary GBMs (36% versus 4%).
  1. Surgical decompression of intracerebral hemorrhage is indicated if there is risk of all of the following EXCEPT
    1. Vital structures of the medulla at risk
    2. Declining level of consciousness
    3. Ventricular enlargement or herniation
    4. Hemorrhage in sites like putamen, thalamus
Ans. D.
  1. Physiological calcification may be seen in
    1. Basal ganglia
    2. Corpus callosum
    3. Occipital lobe
    4. Hypothalamus
    5. Choroid plexus
Ans. E.103
  1. Concerning memory and the limbic system
    1. Lesion of the hippocampus and surrounding area can produce amnesia
    2. The para hippocampal gyrus is continuous with the cingulate gyrus
    3. Primary effects are caused by long-term memory
    4. Procedural memory can be spared in amnesia
    5. Working memory has a capacity of 7 plus minus 2 units of information
Ans. All of the above are true
  • Hippocampus and surrounding areas are involved in memory. The parahippocampal gyrus is the inferior continuation of the cingulate gyrus. The first items in a list are remembered first by activating long term memory. Damage to the medial temporal lobe structure produces disruption of declarative memory, sparing procedural memory. There is a limited capacity for working memory
  1. About staining
    1. Osmium used in electron microscopy
    2. Silver impregnation method used for showing whole of neuron
    3. Marachis method used for staining myelin
    4. Iron – Hemotoxylin stains myelin
    5. Eosin stains rough endoplasmic reticulum + DNA
Ans. A to D.
  1. Structures arising from branchial arch are
    1. Carotid artery → 2nd arch
    2. Aortic arch → 3rd arch
    3. Greater cornu of hyoid → 3rd arch
    4. Stylopharyngeus → 3rd arch
    5. Recurrent laryngeal nerve → 6th arch
Ans. C and D
  • 3rd arch → Common and internal carotid
  • 4th arch → Aortic arch (L); Subclavian (Rt)
  • 6th arch → Pulmonary ductus arteriosus104
  1. About the muscles of hand
    1. Abductor pollicis brevis is the first to show weakness in the carpal tunnel syndrome
    2. Froment's sign is postitive in ulnar nerve injury
    3. Ape thumb hand is seen in median nerve injury
    4. Interossei and lumbricals causes flexion at MCP and extension at DIP and PIP
    5. Flexor diagitorum superficialis causes flexion in DIP and profundus in PIP joint
Ans. A to D.
  • Flexor digitorum profundus causes flexion in distal inter phalangeal joint
  • Flexor digitorum superficialis causes flexion in proximal interphalangeal joint
  1. About segmental innervation of spinal reflexes
    1. Cremastric is L1 – L2
    2. Adductor is L3 L 4 L5
    3. Anal is S4 – Co1
    4. Bulbocavernous is S3 4
    5. Abdominal reflex is T8 – T12
Ans. A, C to E.
  • Adductor is L2, L3, L4
  1. About cervical plexus
    1. Segmental origin is from C1 – C4
    2. Phrenic nerve originates from the cervical plexus
    3. Phrenic nerve mainly arise from C4
    4. Cervical plexus innervate the skin on the back of the head C2 via the greater and lesser occipital nerves
    5. Cape areas of the neck and shoulders by the supraclavicular nerves (C3 and C4)
Ans. All of the above are true
  1. About the basis of brainstem
    1. It transmits corticopontine, corticobulbar and corticospinal tracts
    2. Midbrain basis contains no nuclei
    3. Medullary basis is the smallest
    4. Pontine basis is the largest basis because it contains masses of nuclei upon which the corticopontine efferent synapse105
    5. The corticofugal tracts in the medullary basis have an oval or pyramidal shape on cross section, they are called the medullary pyramids and hence the pyramidal tracts
Ans. All of the above are true
  1. Important decussation occurring at the cervicomedullary transition zone are
    1. Reticulospinal pathways
    2. Pyramidal tract
    3. Trigeminal leminiscus
    4. Medial leminiscus
    5. Olivo cerebellar tract
Ans. All of the above are true
  • The reticulospinal respiratory pathways for automatic breathing just ventral to the obex of the fourth ventricle
  • The three decussation that form internal arcuate fibres and occurring at the cervicomedullary transition zone consist of the following
    1. Trigeminal leminiscus, from the spinal nucleus of CNV
    2. Medial leminiscus from the nuclei gracilis and cuneatus
    3. Olivocerebellar tracts from the inferior olivary nuclei
  1. Groin, anterior thigh sensory loss, loss of knee jerk, quadriceps weakness, loss of hip flexion the roots involved are
    1. L2
    2. L3
    3. L4
    4. L5
    5. S1
Ans. A to C
  1. Clinically important reflex mediated by trigeminal sensory nuclei are
    1. Jaw jerk which is a muscle stretch reflex [V-V reflex]
    2. Corneal reflex [V-VII reflex]
    3. Tearing [V-VII reflex]
    4. Sneezing [V-RF reflex via respiratory center]
    5. Many developmental reflex [Rooting, sucking and chewing]
Ans. All of the above are true106
  1. Muscles supplied by facial nerve are
    1. Platysma
    2. Stapedius
    3. Stylohyoid
    4. Posterior belly of digastric
    5. Orbicularis oculi
Ans. All of the above are true
  1. The Special Visceral Afferent Axons for taste of anterior 2/3rd of the tongue transverses four named peripheral segments in distal to proximal order
    1. Lingual branch of the trigeminal nerve
    2. Chorda tympani
    3. Trunk of CN VII
    4. Intermediate nerve of Wrisberg
    5. None of the above
Ans. A to D.
  1. Central pathway for taste
    1. The primary afferent for taste from CN VII, CN IX, CNX synapse in the nucleus solitarius
    2. Nucleus solitarius connect with the adjacent RF and dorsal motor nucleus (GVE) of CNX to mediate salivatory and lingual reflex
    3. Neurons of the nucleus solitarius also form part of the medullary respiratory center and project to the phrenic nucleus and thoracic segments of the cord involved in reflex such as coughing and vomiting
    4. One taste pathway descend through the central segmental tract to nucleus VPM of the thalamus, this is a non-leminiscal pathway to the thalamus
    5. A second taste pathway relays to the parabrachial nucleus to the RF. Related to the emotional aspect of the taste
Ans. All of the above are true
  • VPM of the thalamus relays taste impulse to the cerebral cortex, in the caudal orbito frontal region, and to the rostral insular and opercular cortex107
  1. Central nervous system disorders causing SIADH are
    1. Basal skull fracture
    2. Acute encephalitis
    3. Subarachnoid hemorrhage
    4. Cerebral vascular thrombosis
    5. Tuberculous meningitis
Ans. All of the above are true
  • CNS disorders—Cerebral atrophy, acute encephalitis, tuberculous meningitis, purulent meningitis, GB syndrome, lupus erythematosus, acute intermittent porphyria
  • Drugs—Chlorpropamide, vincristine, vinblastine, cyclophosphamide, carbamazepine, oxytocin, narcotics, tricyclic antidepressant
  • Ectopic AVP production and release from neoplastic tissue small cell carcinoma of lung, pancreatic carcinoma, lymphoma, Hodgkins disease thymoma, carcinoma of duodenum, inflammatory lung disease, tuberculosis, lung abscess, pneumonia
  1. About pathogenesis of SIADH
    1. Neoplastic cell obtained from the tumor of patient with SIADH can synthesize, store and release AVP
    2. Vasopression also has been demonstrated in tuberculous lung tissue
    3. Intracranial lesion (meningitis, encephalitis, trauma, vascular accidents) probably stimulate release of AVP from the neurohypophysis acting through cytokine – mediated mechanism
    4. Vincristine, cyclophosphamide, carbamazepine stimulate release of AVP from the neurohypophyseal system
    5. Chlorpropamide and NSAID potentiate the ADH action of secreted AVP
Ans. All of the above are true
  1. In SIADH there is
    1. Urine osmolality over 300 mmol/kg
    2. Subnormal plasma osmolality
    3. Subnormal serum sodium concentration
    4. Sodium excretion is usually above 20 mmol/L
    5. Blood urea nitrogen and uric acid tend to fall because of plasma dilution
Ans. All of the above are true108
  • Sodium excretion in the urine is maintained above 20 mmol/L by hypervolemia, suppression of the rennin angiotensin aldosterone system
  • Because of the hypervolemia, blood pressure shows no orthostatic fall, but in spite of hypervolemia, there is no recumbent hypertension and no peripheral edema. The extracellular hypotonicity leads to intracellular swelling and severe symptoms may result from cerebral edema
  1. About clinical mainifestation of SIADH
    1. The rate of fall in serum sodium concentration is more important in producing the neurologic features of SIADH
    2. When SIADH is mild or with serum sodium concentration of 130 to 135 mmol/L the symptoms may be absent or limited to anorexia, nausea or vomiting
    3. When hyponatremia is severe or acute in onset, body weight increases symptoms of cerebral edema predominant including restlessness, irritability, confusion, coma, convulsions
    4. Pitting edema is almost always absent
Ans. All of the above are true
  • SIADH should be suspected in patients who have hyponatremia, concentrated urine (osmolality > 300 mmol/kg) associated with lethargy and in the absence of edema, orthostatic hypotension and features of dehydration
  • The diagnosis is supported by finding of
    1. BUN ↓
    2. Serum uric acid ↓
    3. Serum cretinine ↓
    4. Albumin ↓
  • Condition to differentiate from SIADH are -
  • Dilutional hyponatremia → orthostatic hypotension, tachycardia, elevated BUN characteristic
  • Edematous states → hypothyroidism, congestive heart failure with hyponatremia
  • Hypertensive states → associated with hyponatremia caused by renovascular stenosis or diuretic therapy
  • Pseudohyponatremia → associated with excessive plasma glucose, triglyceride or protein concentration
  • A positive diagnosis can usually be made with water load test. It should not be performed unless the serum sodium concentration has been elevated to a safe level (>125 mmol/L) by restriction of water intake, or by saline administration109
  • Patient is asked to drink the water load (20 ml/kg of body weight upto 1500 ml) 10 to 20 min
  • At least 65% of the water load should be excreted in 4 hrs or 80% in 5hrs and lowest urine osmolality usually reached in the second hour should be below 100 mmol/kg
  • Failure to excrete the water load may occur in adrenal insufficiency or renal insufficiency as well as in SIADH
    Treatment-
    Restriction of fluid intake to 800 to 1000 ml daily
    – To verify the effectiveness of fluid restriction by documenting the change in weight and serum Na+ concentration daily until serum Na+ > 135 mnol/L
    In addition to restriction of fluid intake 200 ml to 300 ml 3% or 5% Nacl should be infused IV over 3 to 4 hours in patient with severe confusion, coma, convulsion
    –To identify the cause and correct the case
  • Administration of water retaining drug should be stopped
  • Meningitis or CNS infection should be treated, phenytoin inhibit AVP release
  • Domecloclycline (900 to 1200 mg/dl) is the most potent inhibitor of AVP. Lithium interfere with action of AVP on renal tubule
  • Prognosis depend on the cause. If drug induced → rapidly and completely corrected
  1. Match of reflexes with their segmental innervation
    1. Cremastric reflex – L1, L2
    2. Superficial anal reflex – S3, 45
    3. Bulbocavernous reflex – S3, S4
    4. Plantar reflex – L5 – S2
    5. Finger flexion reflex – C7, T1
Ans. All of the above are true
  1. Frontal lobe release phenomenon are
    1. Glabellar reflex
    2. Snout reflex
    3. Grasp reflex
    4. Sucking reflex
    5. Groping reflex
Ans. A to E.110
  1. High steppage gait is present in
    1. Diabetic neuropathy
    2. Progressive spinal atrophy
    3. Common peroneal nerve palsy
    4. All of the above
Ans. D
  1. About cortical surface anatomy
    1. The central sulcus joins the sylvian fissure in only 2% of the cases
    2. Superior temporal sulcus terminates in the angular gyrus
    3. Sylvian fissure terminates in the supramarginal gyrus
    4. Inferior temporal gyrus contain the pars opercularis, pars triangularis and pars orbitalis
    5. In 98% of cases there is sub central gyrus
Ans. A to C and E
  • Inferior frontal gyrus contain the pars opercularis, pars triangularis, pars orbitalis
  1. About Craniometric points
    1. Pterion is the region where there is junction of frontal, parietal, temporal and great wing of sphenoid
    2. Asterion is the region where there is junction of lambdoid, parietomastoid and occipito mastoid suture
    3. Stephanion is the junction of the coronal suture and superior temporal line
    4. Asterion overlies the transverse and sigmoid sinus
    5. Bregma is the junction of coronal and superior sagittal suture
Ans. A to C and E.
  1. About blood supply to spinal cord
    1. Posterior spinal artery are paired and usually arise from PICA and are fed from 10–23 radicular arteries
    2. Anterior “corticomedullary” or radicular artery usually arise at C3, C6, C8 level
    3. Artery of Adamkiewicz usually arise between T9-L2 on left side from 2nd lumber artery and supply from T8-conus
    4. Cephalic branch of artery of Adamkeiwicz give a characteristic hair pin appearance on arteriography
    5. Vein from the spinal cord drain into 2 plexiform longitudinal channel than to coronary venous plexus on the cord surface111
Ans. A to C
  • The caudal branch and not cephalic branch of artery of Adamkeiwicz give a characteristic hair pin appearance on arteriography
  • Vein from the spinal cord drain into 6 plexiform longitudinal channels
  1. About cerebral arterial anatomy
    1. A balanced configuration of the Circle of Willis is present in only 18% of the population
    2. 15–35% of patients supply their posterior cerebral artery on one or both sides from the carotid (via P-comm) instead of via the vertebrobasilar system (Foetal circulation)
    3. Hypoplasia of one or both P-communicating arteries occurs in 22–32%, absent or hypoplastic A1 segment occurs in 25%
    4. [P-3] segment of the posterior cerebral artery transverses the ambient cistern
    5. Recurrent artery of Heubner arises commonly from A2 segment of anterior cerebral artery
Ans. A to C and E
  • P-2 segment of the posterior cerebral artery transverses the ambient cistern. P-3 transverses the quadrigeminal cistern and give rise to terminal branches
  1. About Dentate ligament
    1. It is formed from arachnoid
    2. Help in stabilizing spinal cord
    3. Consist of 21 teeth on each side
    4. It is a landmark (posterior most) limit during anterolateral cordotomy
    5. Spinal accessory nerve lies dorsal to the dentate ligament and dentate ligament separates dorsal root from vental root
Ans. B to E
  • Dentate or denticulate ligament arise from Pia and attaches to spinal dura and consist of 21 teeth on each side
  1. About Bladder neurophysiology
    1. Primary co-ordinating center for bladder function resides within the nucleus locus ceruleus of the Pons. Center synchronizes bladder contraction with relaxation of the urethral spincter during voiding112
    2. Voluntary cortical control primarily involves inhibition of the pontine reflex and originate in the anteromedial portion of the frontal lobe and the genu of the corpus callosum
    3. Sensory information is carried through sympathetic T10 – L2 segment and the motor outflow is from parasympathetic S2-S4
    4. Detrusor hyper-reflexia can result from interruption of efferent anywhere from cortex to sacral cord
    5. During spinal shock, the bladder is a contractile and areflexic (detrusor areflexia) sphincter tone usually persists and urinary retention is the rule.
Ans. All of the above are true
  1. Failure disorder of Neuronal migration will result in
    1. Lissencephaly
    2. Schizencephaly
    3. Heterotopia
    4. Porencephaly
    5. Holoprosencephaly
Ans. A to C
  • Holoprosencephaly also known as archinencephaly results due to the failure of the telencephalic vesicle to cleave into two cerebral hemispheres. The degree of cleavage failure range from the severe a lobar to semilobar and lobar anomalies
  1. About Chiari malformations
    1. Beaking of midbrain tectum and cephalad course of upper cervical nerves is characteristic of chiari III malformation
    2. Massa intermedia is present in chiari II malformation
    3. A menigomyelocele is virtually always present in Chiari I malformation
    4. Occipital or high cervical encephalocele is present in Chiari IV malformation
    5. Cerebellar hypoplasia with no herniation is present in Chiari III malformation
Ans. B
  • Beaking of midbrain tectum or tectal fusion and cephalad course of upper cervical nerves is characteristic of [chiari II] malformation113
  • Massa intermedia is the interthalamic adhesion due to absorption of intervening septum pellucidum due to hydrocephalus
  • A meningomyelocele is virtually present in about 100% cases of Chiari II malformation
  • Occipital or high cervical encephalocele is present in Chiari III malformation
  • Cerebellar hypoplasia with no herniation is present in Chiari type IV malformation
  1. Branches of the Meningohypophyseal trunk are
    1. Superior hypophyseal artery
    2. Inferior hypophyseal artery
    3. Capsular artery of Mc Conell
    4. Tentorial artery of Bernasconi and Cassanari
    5. Dorsal meningeal artery
Ans. B, D and E
  1. Following cisterns are parts of mesencephalic cistern
    1. Crural cistern
    2. Ambient cistern
    3. Quadrigeminal cistern
    4. Cisterna magna
    5. Infundibular recess
Ans. A to C
  • Mesencephalic cistern has four parts. Anteriorly there is interpeduncular cistern. Anterolaterally there is crural cistern. Posterolaterally there is ambient cistern. Posteriorly there is quadrigeminal cistern
  1. Regarding immunoglobulin
    1. Molecular weight is around 1,50,000 and IgM has the highest molecular weight
    2. Carboxy terminus of immunoglobulin is responsible for binding of antigen and aminoterminus is responsible for complement activation and various biologic functions
    3. All immunoglobulin classes can activate the complement cascade
    4. J Chain is present in the IgD and IgE
    5. Mature B cell first express both IgM and IgD following stimulation and there is switch from IgM and IgD to other classes114
Ans. A and E.
  • An individual antibody molecule always consists of identical H and L chains
Properties of Human Immunoglobulin
Sl.
Property
IgG
IgA
IgM
IgD
IgE
1.
Percentage of total
75%
15%
9%
0.2%
0.04%
2.
Serum concentration
Highest
Lowest
3.
Molecular wt
Lowest
Highest
4.
Structure
Monomer
Monomer Dimer
Pentomer
Monomer
Monomer
5.
H chain
Y
α
μ
δ
6.
Placental passage
+
7.
Allergic response
+
8.
Found in secretions
+
9.
Opsonizations
+
+
10.
Antigen receptor on B cells
+
+
11.
J chain
+/–
+
• IgG [IgG1, 2,3] can activate the complement cascade
• J chain is present in the IgA and IgM
  1. Match the following antibody in paraneoplastic syndrome
    1. Anti Hu → Limbic encephalitis
    2. Anti – Yo → Pancerebellar degeneration
    3. Anti – VGCC → Lambert Eaton myasthenia syndrome
    4. Ani – GAD → Moersch Woltmann syndrome (Stiff man syndrome)
    5. Ant – Ri → Opsoclonus Myoclonus
Ans. A to E
  1. About shunt in hydrocephalus
    1. Third ventriculostomy is usually not considered in congenital hydrocephalus
    2. Trokildsen shunt is indicated in adult acquired obstructive hydrocephalus
    3. VA shunt has less siphoning effect than VP shunt
    4. Lumboperitoneal shunt is often indicated in communicating hydrocephalus in adult
    5. Acute overdrainage can be prevented by applying anti siphon device
Ans. All of the above are true115
  1. About antibiotics use in neurosurgery
    1. β-lactam antibiotics reduce seizures threshold and are GABA antagonists
    2. Steroid decreases blood brain barrier penetration
    3. Inflammation increases BBB penetration
    4. Combination of third generation cephalosporin + metronidazole is preferable for temporal lobe abscess than for frontal lobe abscess
    5. Frontal lobe abscess is usually secondary to frontal and ethmoid sinus infection and is caused by streptococcus milleri and combination of penicillin G + Metranidazole is used
Ans. A to E
  1. About EEG
    1. Bilateral periodic lateral epileptiform discharge from temporal lobe is diagnostic of Herpes simplex I encephalitis
    2. Bilateral sharp waves [.5–2/sec] with myolconus support diagnosis of C-J disease
    3. Petit mal epilepsy is characterized by a dome and spike pattern is three per second (3/S)
    4. In subacute sclerosing encephalitis, there is periodic high voltage discharge separated e.g. 4–15 seconds associated with body jerks
    5. May be important for differentiating hydrencephaly from severe hydrocephalus
Ans. All of the above are true
  • In hydrencephaly, EEG will be silent and there will be no cortical activity. But in maximal hydrocephalus, there will be presence of background activity
  1. About regulation of respiration
    1. Apneustic center is located in the upper medulla
    2. Destruction of the upper pons can lead to apneusis
    3. Increased number of impulses in the vagal afferent will lead to fast and shallow breathing
    4. Destruction of the medullary respiratory center will result in Biot's breathing (Ataxic breathing)
    5. Cheyne stokes breathing is due to increased sensitivity of respiratory center to PaCo2 and can be seen in diencephalic damage or bilateral cerebral cortex damage
    116
Ans. B to E
  • Apneustic center is present in the lower pons + middle pons.
  • Pneumotaxic center present in upper pons influences the rate of breathing. Inhibits the inspiratory centre and tends to shorten the respiratory cycle.
  1. Paintbrush appearance, meniscus sign, wide splaying of the cord are the findings on myelogram is due to following pathology
    1. Intradural extramedullary, Intramedullary, Extradural respectively
    2. Extradural, Intramedullary, Intradural extramedullary respectively
    3. Intramedullary, Extradural, Intradural extramedullary respectively
    4. Extradural, Intradural extramedullary, Intramedullary respectively
Ans.D
  1. About tuberculous spondylitis or tuberculous vertebral osteomyelitis or Pott's disease are
    1. Usually asymptomatic for many months and affect lower lumber level
    2. Has a predilection for the vertebral body, sparing the posterior elements
    3. Psoas abscess is common because psoas major muscle attaches to the bodies and intervertebral disc from T12 to L5
    4. Neurological deficit develop in 10–47% of patients may be due to medullary and radicular inflammation in most cases, infection rarely extend into spinal cord
    5. 90% of cases can be managed non surgically with antibiotics and immobilization depending on neruraxis compression
Ans. B to E
  • It is usually symptomatic for many months and affects the lower thoracic and upper lumber level
  1. About CNS infection
    1. HIV infection leads to tropical spastic paraplegia and vacuolar myelopathy
    2. Subdural effusion or empyema after meningitis in an infant most commonly develops with H. influenza117
    3. Meningitis occurring within 72 hours after a basilar skull fracture is most commonly secondary to Strep pneumoniae
    4. Late post-operative ventriculo-peritoneal shunt infections are almost only by coagulase negative staphylococci and in neonates by E. coli and streptococcus hemolyticus
    5. Most common cause of neonatal meningitis is group B or D streptococci (Streptococcus aglactiae)
Ans. All of the above are true
  • Subdural effusion is also due to (Streptococcus pneumoniae)
  1. Most common organism causing post-operative discitis is
    1. Staph. Albus
    2. Staph. Epidermidis
    3. Staph. Aureus
    4. H. Influenza
    5. Pseudomonas Aeruginosa
Ans. B.
  • Staph. aureus is the most common organism isolated from discitis but S. epidermidis is the most common pathogen in postoperative discitis. Pseudomonas aeruginosa may be more common in IV drug abusers. H. influenza is common in juvenile discitis
  1. Loss of consciousness is present in all EXCEPT
    1. Grandmal epilepsy
    2. Petit mal epilepsy (Absence seizure)
    3. Complex partial or psychomotor epilepsy
    4. Simple partial epilepsy
Ans. D.
  1. About side effect of Antiepileptic drug
    1. With valproic acid, a rare but serious adverse effect is fulminant hepatitis which occur only in children especially below 3 years
    2. At therapeutic levels phenytoin causes gum hypertrophy (20%), hirsutism, pseudolymphoma, megaloblastic anemia, osteomalacia, hyperglycemia
    3. Carbamazepine causes water retention and hyponatremia in elderly people118
    4. Fatal bone marrow depression can occur with the use of Ethosuximide
    5. Hemeralopia (inability to tolerate bright light) is a side effect of Trimethadione
Ans. All of the above are true
  1. About drug of choice in epileptic disorder
    1. Carbamazepine is the drug of choice for complex partial seizure
    2. Ethosuximide is the drug of choice for absence seizures
    3. Valproic acid is used for grand mal, partial seizures but not for absence seizures
    4. Post traumatic epilepsy is best prevented by carbamazepine and valproic acid because they have effect on kindling but phenytoin has no effect
    5. Carbamazepine in the lowest possible dose as long as controlling seizure is recommended first choice of therapy in pregnant woman
Ans. A, D and E.
  • Nowadays valproic acid has become the drug of choice in absence seizures ethosuximide used to be in past. Valproic acid is additionally helpful in myoclonic and akinetic seizures
  • Phenytoin has no effect on kindling so it is not effective in post traumatic epilepsy
  1. About mechanism of action of antiepileptic drug
    1. Ethosuximide, trimethadione inhibit T (transient) type Ca2+ current exhibited by thalamic neurons
    2. Valproic acid prolong inactivation of Na+ channel, facilitate GABA mediated Cl channel opening and inhibit T type Ca2+ current
    3. Carbamazepine prolong in activation of Na+ channel and also phenytoin
    4. Vigabatrin inhibit GABA transaminase thus inhibiting degradation of GABA
    5. Gabapentin increases the release of GABA
Ans. All of the above are true
  • Thalamocortical system is involved in the generation of absence seizures. Lamotrigine causes inactivation of Na+ channel and inhibit presynaptic release of glutamate119
  1. About interaction of antiepileptic drug
    1. Carbamazepine is an enzyme inducer and its metabolism is induced by phenobarbitone, phenytoin and vice versa
    2. Erythromycin inhibit metabolism of carbamazepine and thus increases its level
    3. Valproate increases plasma level of phenobarbitone by inhibiting its metabolism and displaces phenytoin from protein binding site and decreases its metabolism (phenytoin toxicity)
    4. Concurrent administration of diazepam and valproate is contraindicated because absence status may be precipitated
    5. Chloramphenicol, isoniazide, cimetidine and warfarin inhibit phenytoin metabolism can precipitate its toxicity
Ans. All of the above are true
  1. About antiepileptic drug
    1. Phenobarbitone has the longest half life
    2. Valproic acid has the shortest half life
    3. Primidone metabolism give rise to phenobarbitone as an active product
    4. Alopecia, curling of hair and increase ammonia can be due to side effect of valproic acid
    5. At toxic level of phenytoin, cerebellar and vestibular disturbance can occur like ataxia, nystagmus, vertigo
Ans. All of the above are true
  1. About Thoracolumbar spine fractures
    1. Burst fractures are the most common
    2. Fracture dislocation may involve all three columns
    3. Seat belt type injuries are stable injuries
    4. Wedge compression fractures are generally unstable
    5. Wedge compression fractures involve the middle column
Ans. B
  • Wedge compression fractures are the most common fractures in thoracolumbar region
  • Seat belt injuries involve middle and posterior column and is thus unstable
  • Wedge compression fractures involve only anterior column and hence stable120
  1. Factors that lower the seizure threshold
    1. Sleep deprivation
    2. Hyperventilation
    3. Hypoglycemia
    4. Alkalosis
    5. Cerebral ischaemia
Ans. All of the above are true
  1. About Prolactin levels after seizures
    1. Transient elevation in human serum prolactin level occur following 80% generalized motor, 45% of complex partial and only 15% of simple partial seizures
    2. A serum prolactin level shortly after a questionable seizure may be needful in differentiating non epileptic seizure (which may have elevated cortisol levels but normal serum prolactin levels)
    3. Repetitive seizures are associated with progresively smaller serum prolactin elevation
    4. No rise follows absence seizures or status epilepticus
    5. Greater than two fold serum prolactin elevation consistently follow seizures that produce intense widespread high frequency mesial temporal lobe discharges
Ans. All of the above are true
  • The presence of serum prolactin peaks may be strongly indicative of true seizures
  1. Features of non-epileptic seizures
    1. Most non-epileptic seizure are psychogenic
    2. Out of phase clonic upper extremity movement
    3. Out of phase clonic lower extremity movement
    4. No vocalization or vocalization at start of event
    5. Affect only 20% males as compared to 70% in epileptic seizures
Ans. All of the above are true
  • In non-epileptic seizure also there is violent, high amplitude side to side head turning
  1. About Febrile seizures
    1. Febrile convulsions are the most common types of seizures
    2. The febrile seizures prevalence is 27%. The risk of developing epilepsy after simple febrile seizure is 1%121
    3. The risk of developing epilepsy after complex febrile seizure is 6%
    4. Carbamazepine and phenytion are very effective in controlling febrile seizures
    5. The best treatment of febrile convulsion is rectal diazepam 0.5mg/kg given at the onset of convulsion
Ans. A, B, C and E
  • They are focal seizures not generalized
  • Incidence commonest between 6 months – 5 years
  • Complex febrile seizures → a convulsion that lasts longer than 15 minutes, is focal or multiple (more than one convulsion per episode of fever)
  1. About Status Epilepticus
    1. Most common cause in an adult is with previous seizure and subtherapeutic level of antiepileptic drug
    2. Mean duration of status epilepticus in patient without neurological sequelae is 1.5 hrs (therefore proceed to pentobarbital anaesthesia before 1 hour of status epilepticus)
    3. Recent mortality is <10–12%. Mortality is lowest among children and highest mortality occurs in elderly patients and those with status epilepticus resulting from anoxia or CVA
    4. CNS infection in children more commonly with H. influenzae and S pneumoniae cause status epilepticus
    5. Electrolyte imbalance hyponatremia (most common in children), hypoglycemia, hypocalcemia, ureamia are the causes
Ans. All of the above are true
  • Although status epilepticus is defined for seizures lasting >30 minutes aggressive anticonvulsant therapy is indicated for any seizure lasting >10 mins
  1. About treatment of Status Epileticus
    1. If seizures continue >20 mins, intubate and begin “general anesthesia” with phenobarbital
    2. Using both phenobarbital and a benezodiazepine (i.e., diazepam) is discouraged because of increased risk of respiratory depression
    3. Medications to be avoided in status epilepticus are enflurane, narcotics, phenothiazines including promethazine, neuromuscular blocking agents without antiepileptic therapy122
    4. Among benzodiazepines, lorazepam is preferred because diazepam redistributes rapidly in fatty tissues
    5. Valproic acid is choice for myoclonic status and absence status epilepticus always respond to diazepam
Ans. All of the above are true
  1. About antiepileptic drug
    1. Therapeutic level of free phenytoin is 10–20 ug/ml
    2. At therapeutic level, the elimination of phenytoin becomes zero order kinetic so any change in dosage will lead to a sudden increment in plasma level
    3. Carbamazepine can be started at high dose from starting
    4. Fluoxetine and valproic acid result in elevated phenytoin level
    5. Carbamazepine has more effect on cognitive function than phenytoin
Ans. B and D.
  • Therapeutic level of phenytoin is 10–20 ug/ml and of free phenytoin is (1%) i.e. 1–2 ug/ml. 90% of drug is bounded to plasma protein carbamazepine dose range from [600–2000mg/d]. It is started at low dose and increased slowly. Phenytoin has more effect on cognitive function than carbamazepine
  1. Match the therapeutic level of following antiepileptic drug
    A.
    Phenytoin
    10–20ug/ml
    B.
    Carbamazepine
    6–12ug/ml
    C.
    Valproic acid
    50–100ug/ml
    D.
    Phenobarbital
    15–30ug/ml
    E.
    Ethosuximide
    40–100ug/ml
    • Note: Therapeutic level of carbamazepine may be misleading since the active metabolite carbamazepine –10, 11 epoxide may cause toxicity and must be assayed separately
Ans. All of the above are true
  1. Following antiepileptic drugs are inducer of liver cytochrome P450 enzyme
    1. Primidone
    2. Phenytoin
    3. Lamotrigine
    4. Valproic acid
    5. Carbamazepine
123
Ans. A, B and E.
  • Felbamate is a potent metabolic inhibitor, thus it is necessary to reduce the dose of phenytoin, valproate or carbamazepine when used with felbamate
  1. Which of the following antiepileptic drug is GABA uptake inhibitor?
    1. Gabapentin
    2. Phenobarbitone
    3. Vigabatrin
    4. Tiagabine
Ans. D
  1. Which of the following antiepileptic drugs lead to withdral seizure even status and has to be withdrawn over 3–6 months?
    1. Phenytoin
    2. Valproic acid
    3. Phenobarbitone
    4. Clonazepam
Ans. D
Recommended AED withdrawal times AED
Recommended withdrawal period
Phenytoin, Valproic, arbamazepine
2–4 weeks
Phenobarbital
6–8 weeks (25% per week)
Clonazepam
3–6 months
  1. Which of the following antiepileptic drug level increases during pregnancy?
    1. Carbamazepine
    2. Phenobarbital
    3. Phenytoin
    4. Valproic acid
Ans. D
Changes in free AED levels during pregnancy
Drug
Change
Carbamazepine
↓ 11%
Phenobarbital
↓ 50%
Phenytoin
↓ 31%
Valproic acid
increase 25%
124
  1. About the side effect of antiepileptic drug in pregnancy:
    1. Carbamazepine increases the risk of minor malformation but not of major malformation and may increase the risk of NTD so never given in combination with valproic acid
    2. Phenytoin may lead to hypoplastic phalanges, microcephaly, cleft plate cleft lip
    3. Phenobarbital produced the highest incidence of major malformation (9–1%) and associated with most of the increase in foetal death
    4. Valproate causes the highest incidence of neural tube defect (1–2%), thrice dosing may reduce the risk of neural tube defect
    5. Benzodiazepine given shortly before delivery can produce the “floppy infant syndrome”
Ans. All of the above are true
  1. Which of the following tumor is associated with medically intractable seizures?
    1. Central neurocytoma
    2. Gliomatosis cerebri
    3. Pleomorphic xanthoastrocytoma
    4. Dysembryoplastic neuroepithelial tumor (DNT)
Ans. D
  1. Corpus callosotomy is most favourable for which form of epilepsy?
    1. Atonic
    2. GTC (Generalized tonic clonic)
    3. SPE (simple partial epilepsy)
    4. CPE (complex partial epilepsy)
Ans. A
  • Section of corpus callosum when drop attacks are the most disabling seizure type or for multiple bilateral foci, there is 70% reduction with callostomy
  1. Disconnection syndrome after corpus callostomy in a patient with a dominant left hemisphere consists of
    1. Left tactile anomia
    2. Pseudohemianopsia
    3. Right sided anomia for smell125
    4. Impaired spatial synthesis of right hand resulting in difficulty copying complex figures
    5. Left sided dyspraxia may resemble hemiparesis
Ans. All of the above are true
  • Inability to match object present in right and left hand. Inability to match object shown in right and left half of visual field. Right sided anomia for smell is because representation for olfaction is asymmetric in prefrontal cortex, it is (Right > Left) more common with larger surgical sections of the corpus callosum. Risk is less if the anterior commisure is spared. Patients usually adapt after 2–3 months, with final function normal for most daily activities
  1. Each of the following cell groups is derived from the alar plate EXCEPT
    1. Nucleus ambigus
    2. Principal sensory nucleus of V
    3. Solitary nucleus
    4. Spinal trigeminal nucleus
    5. Vestibular nucleus
Ans. A
Nucleus ambigus is derived from basal plate
  1. Following test recommended for assessing acute low back problem
    1. Needle EMG
    2. H-reflex + SEP
    3. Nerve conduction velocity
    4. Surface EMG
    5. F-wave
Ans. A to C
  • Surface EMG and F-wave response are not recommended for assessing acute low back problems
  1. How many % asymptomatic patient show presence of herniated lumbar disc and spinal stenosis respectively on MRI?
    1. 24%, 4% respectively
    2. 28%, 8% respectively
    3. 38%, 12% respectively
    4. 12%, 2% respectively
126
Ans. A.
  • Asymptomatic patient having herniated lumbar disc is about 24% and spinal stenosis about 4%
  • In elderly asymptomatic patient, herniated lumbar disc is about 36% and spinal stenosis 21%
  • Diagnostic radiology is of limited benefit as the initial evaluation in the majority of spinal disorders
  1. About Lumbar disc
    1. Disk bulge is broad based occurring due to focal weakness in Posteror longitudinal ligament
    2. Protrusion of the disc implies ruptured annulus but Posterior longitudinal ligament remains intact
    3. In extrusion, it is mandatory that Posterior longitudinal ligament is ruptured
    4. S1 invovement can occur in central disc herniation at L4-L5 level
    5. Extreme lateral disc herniation at L5-S1 involves L5
Ans. B, D and E
Note: The bulge is concentric smooth circumferential expansion beyond the confines of end plates. It is broad based short radius of extension < 3 mm with no weakness in annulus or PLL. In protrusion there is focally weakend or ruptured annulus but PLL is intact. In extrusion, disc material is connected with only an isthmus with ruptured annulus or intact PLL or ruptured annulus along with ruptured PLL. Not mandatory for extrusion to have ruptured PLL
  1. About diagnostic radiological procedure evaluating vertebral column
    1. MRI and myelography both provide information in sagittal plane unlike plain CT scan
    2. MRI and myelography both evaluate cauda equine unlike routine CT
    3. MRI provide information regarding tissue outside the spinal canal (e.g. extreme lateral disc herniation tumors possibly better than CT)
    4. Advantage of CT over MRI is that it has faster scanning, less expensive, less claustrophobic, excellent bony detail and of studying blood early
    5. In cases of scoliosis, there is difficulty to interpret in MRI. Myelogram/CT may be superior
    127
Ans. All are true
  • Note: Myelography provide [functional] information about degree of stenosis. In cases of scoliosis coronal plane scan give better information than sagittal scan in MRI
  1. Regarding artificial disc system all are true EXCEPT
    1. Bryan artificial disc system is for lumbar disc
    2. Prestige cervical disc system is FDA approved as class III device
    3. ChariteTM artificial disc is for lumbar disc.
    4. Criteria for artificial disc replacement is disc degeneration in the only one disc
Ans. A. Bryan artificial disc system is for cervical disc. Link MB charite III disc prosthesis is for lumbar spine not FDA approved. ChariteTM artificial disc is FDA approved
  1. Regarding straight leg rising test
    1. Crossed SLR is more sensitive for the radiculopathy than ipsilateral SLR (Lasegue's sign)
    2. Forced foot dorsiflexion or Bragard's test causes a positive SLR test to become positive at a lower angle of inclination.
    3. A positive femoral stretch test or reverse straight leg raising test suggest L5 or S1 radiculopathy
    4. Severe pain with straight leg raising at 90 degrees is less sensitive for radiculopathy than positive response at a lower angle
    5. In case of SLR test tension in nerve increases little above 60° angle
Ans. B, D and E
  • [Terminology]
    Lasegue's sign → Straight leg rising test
    Fajersztajn's sign → Crossed straight leg rising test
    Femoral stretch → Reverse SLR test
    Bragard test → Forced foot dorsiflexion + SLR test
  • SLR primarily tenses L5 and S1. L4 less so and more proximal root very little
  • Nerve root compression produces a positive Lasegue's sign in 83% of cases, more likely to be positive in patient <30yr of age with HLD. May be positive in lumbar plexopathy
Note: Flexing both thighs with the knee extended may be tolerated further than flexing the single symptomatic side alone128
  • Crossed straight leg rising test → SLR on the painless leg causes contralateral limb pain (a greater degree of elevation is usually required than the painful side. More specific but less senstive than SLR (97% of patients undergoing surgery with this sign have confirmed Herniated lumbar disc, may correlate with a more central disc herniation
  • Reverse SLR → Positive with L2, L3, L4 nerve rest compression (e.g., in upper lumber disc herniation) or with extreme lumber disc herniation (may also be position in diabetic femoral neuropathy or psoas hematoma). In these situations SLR (Lasegue's sign) is frequently negative (since L5 and S1 are not involved).
  1. Herniated lumber disc is most common at which level
    1. L2 – L3 level
    2. L3 – L4 level
    3. L4 – L5 level
    4. L5 – S1 level
Ans. D.
  • L3-L4 level → [3–10%]
  • L4-L5 level → [40–45%]
  • L5-S1 level → [45–50%]
  1. About Oculomotor nucleus
    1. Lateral nucleus is for superior rectus
    2. Intermediate nucleus is for inferior rectus
    3. Medial nucleus is for medial rectus and inferior oblique
    4. Caudal subnucleus for perlia for accommodation and convergence
    5. Cranial nucleus for perlia is for levator palpebrae superioris
Ans. C
  • Lateral nucleus is for Inferior rectus and has ipsilateral innervation
  • Intermediate nucleus is for Superior rectus and has contralateral innervation
  • Medial nucleus is for Inferior oblique + Ipsilateral medial rectus
  • Caudal sub nucleus of perlia is for Bilateral levator palpebrae superioris. Cranial nucleus of perlia is for accommodation and convergence
  • E – W nuclei is parasympathetic nuclei and is for pupil constriction129
  1. Statement about nerves
    1. C1 passes between C1 and C2 vertebrae
    2. Epineurium, perineurium and endoneurium is the continuation of the dura, subarachnoid and pia respectively
    3. Endoneural capillaries and their cell form the nerve blood barrier
    4. The epineurium may sometime continue as the capsule of Meissner's corpuscles
    5. The perineurium continue as the capsule of some specialized nerve ending such as pacinian corpuscles, muscle spindles and Golgi tendon organs
Ans. B to E
  • C1 passes between the occiput and the atlas (C1) vertebrae
  1. Percentage of injury of the recurrent laryngeal nerve in Anterior cervical disectomy with fusion (ACDF) is
    1. 11% temporary, 4% permanent paresis more common on left side
    2. 11% temporary, 4% permanent paresis, more common on right side
    3. 22% temporary, 8% permanent paresis, more common on right side
    4. 22% temporary, 8% permanent paresis, more common on left side
Ans. B.
  • More common on right side approaches where the recurrent laryngeal nerve is more variable
  1. Cervical sympathetic plexus lies within which muscle
    1. Rectus capitis lateralis
    2. Obliquis capitis inferior
    3. Longus colli
    4. Splenius capitis
Ans. C
  1. The oxyhaemoglobin dissociation curve is shifted to the right by
    1. Acidosis
    2. Decreased 2,3 diphosphoglyceric acid (2,3 –DPG)
    3. Fever
    4. Banked blood
    5. All of the above
130
Ans. A and C.
  • The curve is shifted to the right by acidosis, fever, increased 2,3 –DPG and hypoxemia and to the left by alkalosis, hypothermia, banked blood and decreased 2,3 –DPG
  1. About Ossification of the spine
    1. At birth, most vertebrae have three primary and five secondary ossification centers connected by hyaline
    2. Exception to typical ossification occur at C1,C2,C7, lumbar vertebrae, sacrum and coccyx
    3. Atlas C1 vertebrae has no secondary ossification centers
    4. Lumbar vertebrae has three primary ossification centers per vertebrae and seven secondary ossification centers per vertebrae
    5. Sacrum has five ossification centers per vertebrae and four secondary ossification centers
Ans. All options are true. Centrum ossification starts at lower thoracic/upper lumbar spine of fetus. Moves in both cranial, and caudal direction. Neural arch ossification begins at cervicothoracic level and then upper cervical region and lastly thoracolumbar region. Atlas has two to five (three most common) primary ossification centers and no secondary ossification centers. Axis has five primary ossification centers and two secondary ossification centers. C3–6 has three primary ossification centers per each vertebrae and five secondary ossification centers per each vertebrae Co1 has three primary ossification centers. Co2–Co4 have one primary ossification center each and no secondary ossification center
  1. “Standard” laminectomy for cervical spondolytic myelopathy is
    1. C3-C7
    2. C4-C7
    3. C2-C7
    4. C1-C7
Ans. A.
  • A C3–7 laminectomy is often considered a “standard” laminectomy. An “extended laminectomy” includes C2 and sometime C7131
  1. Recognised causes of potassium depletion include
    1. Chronic steroid therapy
    2. Cardiac failure
    3. Hypertension
    4. Renal tubular acidosis
    5. Triamterene therapy
Ans. A to D
  • Triamterene is a K+ sparing diuretic
  1. About subdural hematoma
    1. Acute subdural hematoma mostly occur in association with diffuse axonal injury and is more common with head rotation or angular acceleration in coronal plane than in sagittal plane
    2. Subdural hematoma occur commonly in the elderly due to some cerebral atrophy and in children less than 3 years because dura is attached more firmly to the skull
    3. In chronic subdural hematoma, the size of hematoma increases due to repetitive bleeding into the hematoma from the fragile capillaries from both inner and outer membrane.
    4. Both inner and outer membranes covering the hematoma are adhered firmly to the arachnoid and dura respectively
    5. During the removal of haematoma adhesion on the outer membrane is not touched due to neocapillaries present which can lead to repetitive bleeding
Ans. A, B and E
  • Outer membrane that is derived from dura is very vascular and consist of fragile capillaries. Inner membrane is thin and fibrous
  • One characteristic pathological feature of chronic subdural hematoma is that there is adhesion with the dura. Where the contact of the inner membrane with the arachnoid is smooth there is no adhesion
  1. In Paget's disease of the skull, most common cranial nerve to be involved is
    1. V
    2. VIII
    3. VII
    4. IX
    5. X
132
Ans. B
  • 8th nerve is the most common to be involved leading to deafness and ataxia
  1. Which of the following drugs is used in the treatment of Hypercalcemia due to bony metastases?
    1. Plicamycin or Mithramycin
    2. Etidronate
    3. Pamidronate
    4. Salmon calcitonin
Ans. A.
  • Plicamycin or mithramycin-a cytotoxic antibiotic that inhibit RNA synthesis with preferential toxicity for osteoclasts. Not approved for treating paget's disease in any country
  • Etidronate a biphosphonate [Pyrophosphate analogue that bind to hydroxypatite crystal and inhibit reabsorption. Except Etidronate other biphosphonate used for paget's disease do not reduce normal bone mineralization
  1. Regarding ossification of posterior longitudinal ligament [OPLL]
    1. Also known as Japanese disease because of highest prevalence in Japan (2–3.5%)
    2. It begins with hypervascular fibrosis in the PLL
    3. Changes within the spinal cord involve the white matter more than the posterolateral gray matter.
    4. There is average involvement of 2.7 to 4 level and involve cervical region (70–75%) of cases of OPLL which typically begins at C3-C4 and proceeds distally but usually sparing C6-C7
    5. Myelography with post myelographic CT is probably best at demonstrating and accurately diagnosing OPLL
Ans. A, B, D and E.
  • Involves posterolateral gray matter more than white matter probably ischaemic basis
  1. About Spinal AVM's
    1. Type I spinal AVM is a dural AVM and is most common type in adults usually traumatic in aetiology with low flow-high pressure dynamics133
    2. Type II AVM also called Glomus is intramedullary common in lumbar region with high flow-high pressure dynamics and worse prognosis than dural AVM
    3. Type III AVM also called juvenile AVM is intradural extramedullary in position with high flow-high pressure dynamics
    4. Type IV AVM is intradural and perimedullary in position occur in younger patients than type I and may present catastrophically with hemorrhage into the subarachnoid space also called as Arteriovenous fistulae
    5. Regarding treatment dural AVM fed by dural artery usually require surgery, intradural AVM may be amenable to interventional neuroradiological procedure including embolization
Ans. A, C, D and E
  • Intramedullary glomus AVM arising from the medullary artery usually present in the cervical region
  • Thoracolumbar is the commonost site (-65%). It may be extra or intradural. The main feeding vessels are often dural arteries of the spinal nerve root (Intramedullary lesion) at this site are less common
  • Low-flow and high pressure dynamics is present in Type I and Type IV AVM
  • High flow and high pressure dynamics is present in Type II, Type III, Type IV but not in Type I
  • Type IV and Type II typically present with progressively worsening symptoms without significant clinical improvement
  • Coup de poignard of Michon = onset of SAH with sudden excruciating backpain (clinical evidence of SpinalAVM). Foix-Alajouanine syndrome → acute or subacute neurologic deterioration in a patient with a Spinal AVM without evidence of hemorrhage
  1. About spinal meningeal cyst(SMC)
    1. Type I SMC is also called Tarlov's cyst or perineural cyst
    2. Type II SMC is most prominent and symptomatic in sacrum
    3. Type III SMC is intradural
    4. Type I and Type II are extradural
    5. Type II SMC often asymptomatic, but sacral lesion may cause sciatica or sphincter disturbance.134
Ans. B to E
Type
Description
Type I
Extradural meningeal cysts without spinal nerve root fibers
IA
Extradural meninigeal/arachnoid cyst
IB
Occult “sacral meningocele”
Type II
Extradural meningeal cysts with spinal nerve root fibers/Tarlov cyst
Type III
Spinal intradural meningeal cysts (Intradural arachnoid cyst)
  1. Causes of communicating syringomyelia are
    1. Dandy Walker malformation
    2. Arnold chiari malformation Type I or Type II
    3. Cerebellar ectopia
    4. Basilar adhesive arachnoiditis
    5. Basilar impression (with constriction of the foramina)
Ans. All of the above are true
  1. About Spinal epidural hematoma
    1. Following trauma occur almost exclusively in patient who is anticoagulated, thrombocytopenia or has bleeding diasthesis
    2. Thoracic level is most common
    3. Most often located posterior to spinal cord, facilitating removal
    4. Immediate decompressive laminectomy is indicated in those patients who can tolerate surgery
    5. In high risk patient consider use of high dose methylprednisolone to minimize cord injury or percutaneous needle aspiration may be considered
Ans. All of the above are true
  1. Regarding surgical treatment of Parkinson's disease
    1. Historically anterior choroidal artery was ligated by Irwin Cooper for Parkinsons’ disease
    2. Neural transplantation in patient with Parkinson's disease was first suggested by Bjorklund and Perlow et al
    3. Primary indication for pallidotomy in cases with levopopa induced dyskinesia.
    4. Tremors mainly respond to VOP/VIM (Ventro lateral nucleus) thalamotomy
    5. Patient with secondary parkinsonism not idiopathic usually do not respond to surgery
Ans. All of the above are true135
  1. Match the following Microvascular compression syndromes
    1. Hemifacial spasm – Anterior inferior cerebellar artery
    2. Trigeminal neuralagia – superior cerebellar artery
    3. Glossopharyngeal neuralagia – Posterior inferior cerebellar artery
    4. Torticollis – Vertebral artery
Ans. All of the above are true
  1. Surgical procedures utilized in treatment of spasmodic forticollis are
    1. Dorsal cord stimulation
    2. Selective rhizotomy and spinal accessory nerve section
    3. Stereotactic electrocoagulation of Forel's H field
    4. Microvascular decompression of the 11th nerve
    5. Local injection of botulinum toxin work for retrocollis, poor for lateral torticollis and totally ineffective for anterocollis
Ans. All of the above are true
  1. About Torticollis
    1. Spasmodic torticollis is a specific subtype of torticollis that is idiopathic and shortened sternocleidomastoid is usually in spasm
    2. Torticollis from atlanto axial rotatory subluxation the elongated sternocleidomastoid is in the spasm
    3. Pseudotorticollis develop as an unconscious correction to diplopia which may occur in trochlear nerve palsy
    4. Extrapyramidal lesion [including degenerative] may cause torticollis which is often alleviated by lying down
    5. Torticollis due to microvascular compression is usually a horizontal type which is exacerbated when supine
Ans. All of the above are true
  1. Match the following glycogen storage disease
    Name of the disease
    Enzyme deficiency
    A. Von Gierke's disease
    Glucose 6 phosphatase
    B. Pompe's disease
    alpha (,4) glucosidase
    C. Cori's disease
    Amylo 1,6- glucosidase, i.e. debranching enzyme
    D. Andersen's disease
    1,4 → 1,6 transglucosylase, i.e. branching enzyme
    E. McArdle's disease
    Muscle glycogen phosphorylase
    136
Ans. All of the above are true
  • Glycogen storage disease name is based on the name of the patient first diagnosed of that disease
  • Hers’ disease is due to deficiency of Liver phosphorylase
  1. About Brain metabolism
    1. Ketones (fat derivative acetoacetate and hydroxybutyrate) normally make up approx. 30% of the fuel for the brain in adults
    2. Brain consumes 20% of the body oxygen and 60% of its glucose
    3. In infants ketone transport is seven times as high as in adults
    4. Glycogen content of the brain is greater than muscle and liver
    5. Amount of CO2 and glutamine produced by brain per minute is about 49 ml and 8.4 mg respectively
Ans. A, B, C and E
  • RQ (Respiratory quotient) of cerebral tissue is (95–99) in normal individual
  • In a 70 kg man the amount of glycogen in liver will be 60mg
  • In muscles total glycogen will be 150 mg
  • Brain contains very little glycogen, major energy source is glucose
  • Glucose is utilized at a rate of 5.5 mg/100 g/min or total 77 mg/min
  1. Match the following rate of utilization or production for brain tissue in 100gm/min
    1. Mean blood flow → 54 ml/100 gm/min
    2. O2 utilization → 3.3–3.5 ml/100 gm/min
    3. Glucose utilization → 5.5 gm/100 gm/min
    4. Glutamate utilization → 0.4 mg/100 gm/min
    5. Glutamine production → 0.6 mg/100 gm/min
Ans. All of the above are true
  1. Secondary Hyperhidrosis is due to
    1. Hyperthyroidism
    2. Diabetes Mellitus
    3. Syringomyelia
    4. Parkinsonism
    5. Hypothalamic tumor
    137
Ans. All of the above are true
  • Essential (Primary or Idiopathic) Hyperhidrosis Secondary Aetiologies
    1. Pheochromocytoma
    2. Acromegaly
    3. CNS trauma
    4. Menopause
    Sweat is a hypotonic secretion with saline as the primary constituent most eccrine or merocrine sweat gland serve as thermo regulatory function, however, those on the palms and soles respond primarily to emotional stress
  1. Procedures for treating medically refractive pain are
    1. Cingulotomy
    2. Stereotactic mesencephalotomy 5mm lateral to the sylvian aqueduct at the level of inferior colliculus
    3. Anterior thalamotomy
    4. Dorsal rhizotomy
    5. Anterolateral cordotomy
Ans. A, B, D and E
  • It is medial thalamotomy not anterior thalamotomy
  1. Regarding complex regional pain syndrome or causalgia
    1. The edema and trophic changes are due to over activity of sympathetic outflow
    2. Regional sympathetic blockade is effective for complex regional pain
    3. Sympathetic outflow to the affected region is increased in most patients
    4. Spread of the distribution of pain and sensory change is common and does not implicate a psychogenic etiology.
    5. CRPS studies have had an unusually high placebo response rate. Medical therapy is usually ineffective.
Ans. D and E.
  • Causalgia term was given by Weir Mitchell in (1864)
  1. About Pain relieving procedure
    1. Medial thalamotomy can be used in phantom limb pain
    2. Morphine intraventricular infusion for nociceptive pain above C5138
    3. DREZ rhizotomy can be used for post herpetic neuralgia but is best for deafferentiation pain in brachial plexus avulsion
    4. Sympathectomy can be used for causalgia
    5. Younger patients, without sensory complication having trigeminal neuralgia of first division microvascular decompression is the procedure of choice
Ans. All of the above are true
  1. Which of the following antiepileptic used in case of Trigeminal neuralgia?
    1. Carbamazepine
    2. Vigabatrin
    3. Valproic acid
    4. Phenytoin
    5. Gabapentin
Ans. A, D and E
  • Clonazepam is also used
  1. Preganglionic parasympathetic fibres from Edinger westphal nucleus reach to cilliary ganglion via
    1. Nerve to superior rectus
    2. Nerve to medial rectus
    3. Nerve to inferior rectus
    4. Nerve to inferior oblique
Ans. D.
  1. Regarding contribution of the following in Neurosurgery
    1. Victor Horsley first attempted retrogasserian neurectomy for tic douloureux
    2. Simpson first pointed out the importance of extent of resection of meningioma as an important determinant in recurrence
    3. Cushing first proposed deliberate hypotension to provide bloodless field during surgery
    4. Walter Dandy first started choroid plexectomy for hydrocephalus
    5. Egas Moniz received nobel prize in 1949 for the initiation of psychosurgery
Ans. All of the above are true139
  1. Glossopharyngeal neuralgia consist of
    1. Lancinating pain radiating to throat and base of tongue
    2. Hypotension
    3. Syncope
    4. Often Cardiac arrest
    5. Convulsion
Ans. All of the above are true
  1. Prosopalgia is seen in
    1. Geniculate neuralgia (Hunt's neuralgia)
    2. Trigeminal neuralgia
    3. Hemifacial spasm
    4. Glossopharyngeal neuralgia
Ans. A.
  • Prosopalgia (pain referred to deep facial structures, including orbit Posterior nasal and palatal regions, seen in Geniculate neuralagia
  1. About Post Herpetic Neuralgia
    1. Caused by Herpes simplex type I virus
    2. Occur in peripheral nerve distribution
    3. Post herpetic neuralgia is rare in age less than 40 years and usually occurs in age >60 and in those with diabetes mellitus
    4. Post herpetic neuralgia is more likely after ophthalmic herpes zoster than after spinal segment involvement
    5. Most drug useful for trigeminal neuralagia are less effective for post herpetic neuralgia
Ans. C to E.
  • It is caused by Herpes varicella zoster (chickenpox)
  • Herpes zoster (Shingles) indicate painful vesicle eruption
  • Infection without vesicle [zoster sine herpete]
  • In 20% it involves ophthalmic nerve
  • In 85% thorax (dermatomal distribution)
  • Vesicle eruption is always in the dermatomal distribution not in the peripheral nerve distribution
  1. Regarding statement about rigidity
    1. Decorticate rigidity is greater than decerebrate rigidity
    2. Righting reflex is present in the decerebrate animal
    3. Visual righting reflex is present in a thalamic animal140
    4. Decorticate rigidity is seen only when the animal is at rest
    5. Decerebrate rigidity is popularly called as alpha-rigidity
Ans. C to E
  • Decerebrate rigidity is greater than decorticate rigidity
  • Righting reflex is integrated in midbrain and hence absent in the decerebrate animal
  • Visual righting reflex is integrated in cerebral cortex hence present in thalamic animal
  • Decerebrate rigidity is popularly called gamma-rigidity. It is due to release of spinal gamma-motor neuron from an inhibitory extrapyramidal discharge therefore called gamma-rigidity
  1. Regarding representation of somatosensory projection areas in the cerebellar cortex and thalamus
    1. In cerebellum, there is an ipsilateral projection and is inverted in the anterior lobe
    2. In cerebellum, there is a bilateral representation and is erect in the posterior lobe
    3. In cerebellum, trunk is represented medially and the extremities laterally
    4. In the thalamus, the extremities are represented dorsally and the back ventrally
    5. In the thalamus, the head is represented medially and the caudal thorax laterally
Ans. A, B, C and E.
  • In thalamus, the extremities are represented ventrally and the back dorsally
  1. Which of the following familial syndromes associated with Medulloblastoma and Glioblastoma?
    1. Von Hippel-Lindau
    2. Tuberous Sclerosis
    3. Neurofibromatosis type II
    4. Li-Fraumeni syndrome
    5. Turcot syndrome
Ans. E.
  • Von Hippel Lindau consist of Hemangioblastoma
  • In Tuberous sclerosis, there is Subependymal giant cell astrocytoma
  • Li-Fraumeni syndrome include Astrocytoma and PNET
  • Turcot syndrome include Medulloblastoma and Glioblastoma141
  1. Tumor amenable to surgical resection and have most favourable prognosis
    1. Juvenile pilocytic astrocytoma
    2. Pleomorphic xanthoastrocytomas
    3. Ganglioglioma
    4. Dysembryoplastic neuroepithelial tumor [DNT]
    5. Oligodendroglioma
Ans. A to D
  1. Ratio of relative frequency of Glioblastoma : Anaplastic astrocytoma : Low grade astrocytoma is
    1. 5:3:2
    2. 6:2:2
    3. 7:2:1
    4. 8:1:1
Ans. A
  • Peak age for Low grade astrocytoma is 34 years, for Anaplastic astrocytoma is 41 years and for Glioblastoma, it is 53 years
  1. Match the mechanism of the following anti-cancer drug
    1. Nitrosoureas → DNA alkylation
    2. Temozolamide → DNA crosslinks carbamoylation of aminogroup
    3. Vinca Alkaloid → Microtubule function inhibitors
    4. Carboplatin or cisplatin → chelation via intrastrand cross links
    5. Tamoxifen → Protein Kinase c inhibitor
Ans. C to E
  • Chemotherapeutic agent used for CNs tumors
  • Nitrosoureas, Carmustine, Lomustine and Nimustine causes DNA cross links, carbamolyation of amino groups
  • Alkylating agent Procarbazine, Temozolomide causes DNA alkylation and interfere with protein synthesis
  • Carboplatin and Cisplatin causes chelation via intrastrand crosslinks
  • Nitrogen mustards –Cyclophosphamide, isofamide, Cytoxan causes DNA alkylation, carbonium ion formation.
  • Vinca alkaloids –Vincristine, Vinblastine, Paclitaxel causes microtubule function inhibition. Tamoxifen is a protein kinase with inhibitor. Epidophyllotoxins result in topoisomerase II inhibition142
  1. About Reflexes
    1. In tonic labyrinthine reflex when the patient is supine maximum tone is in the antigravity muscle
    2. In a decorticate patient when the head is moved to one side there is ipsilateral extension of the limbs
    3. In tonic labyrinthine reflex when the patient is prone there is minimal tone in the extensor group of muscles
    4. Cerebral cortex, basal ganglia, cerebellum has facilitatory effect on the stretch reflex
    5. In tonic labyrinthine reflex when the head is flexed there is flexion of upper limb with extension of one hind limb
Ans. A to C.
  • Cerebral cortex, basal ganglia, cerebellum has inhibitory effect on the stretch reflex. Reticular facilitatory area and vestibular area facilitilate the stretch reflex whcih is a basic postural reflex
    Option E. is a typical tonic neck reflex
  1. Which of the following righting reflex is integrated in midbrain?
    1. Optic righting reflex
    2. Placing reaction
    3. Labyrinthine righting reflex
    4. Neck righting reflex
    5. Body on head righting reflex
Ans. C to E
  • Option A. and B. are integrated in cerebral cortex also hopping reaction
  1. A patient has undergone surgery for a brain tumor which has been present for 5 years with seizure, has a benign course and was removed completely with a cure. The most likely histological feature will be
    1. Sheet of polygonal cell with psammomatous body
    2. “Fried egg” appearance and chicken wire vasculature
    3. Necrotic areas with pseudopalisading
    4. Section showing Homer Wright Rossetes
Ans. A.
  • It's a typical presentation of meningioma, a benign tumor and there is cure if it is completely removed. In meningiotheliomatous AKA syncytical type have sheet of polygonal cells143
  1. About Acoustic neuroma
    1. Commonly arise from the superior vestibular division at the Obersteiner Redlich zone
    2. Consist of Antoni A and Antoni B area and displace the nerve fibre peripherally
    3. High pitch tinnitus is usually present in early cases followed by progressive hearing loss
    4. Rate of growth is [1–10mm/yr]
    5. If tumor is very small say, <1cm and is lateral to IAC then middlefossa (extradural subtemporal) approach is preferred
Ans. All of the above are true
  1. Common tumor with ectopic ACTH secretion
    1. Small-cell carcinoma of the lung
    2. Thymoma
    3. Carcinoid tumors
    4. Phaeochromocytomas
    5. Medullary thyroid carcinoma
Ans. All of the above are true
  1. Which statement is true about Meningioma?
    1. Are malignant in 50% of cases
    2. Occur predominantly in men
    3. Are treated primarily by surgical excision
    4. Are cured, when properly treated in nearly 95% of cases
    5. Arise from the dura
Ans. C.
  • It is basically a benign tumor, malignant only in 1.7% cases
  • Occur predominatly in 65% Female: Male ratio – [1.18:1]
  • 5 year survival rate is 91.3% and 15 years survival rate is 68%
  • Arise from the arachnoid cap cell
  • Meningioma constitute 14.3–19% of the intracranial tumor
  1. Which of the following pituitary tumor is least likely to produce mass effect?
    1. Null cell adenoma
    2. Oncocytoma
    3. Prolactinoma
    4. ACTH secreting adenoma
144
Ans. D.
  • Null cell adenoma and oncocytoma constitute the bulk of endocrine inactive adenomas of functional tumor, prolactinoma is most likely to become large enough to cause mass effect (ACTH tumor is least likely)
  • Mass effect is usually (but not exclusively) seen with nonfunctioning tumors
  1. In which of the following conditions there is increased Prolactin?
    1. Prolactinoma
    2. Phenothiazine
    3. Stalk effect
    4. Primary hypothyroidism
    5. Secondary hypothyroidism
Ans. A to D.
  • Phenothiazine is a dopamine antagonist hence lead to increase in Prolactin. In primary hypothyroidism there is increase in TRH which increases release of prolactin
  1. Regarding Acid-Base disturbances
    1. Primary aldosteronism and Cushing's disease causes metabolic alkalosis
    2. Myasthenia gravis will cause respiratory acidosis
    3. Addison's disease will cause normal anion gap metabolic acidosis
    4. Metabolic alkalosis is often accompanied with hypokalemia
    5. Salicylate (overdose) early will cause respiratory alkalosis and ethylene glycol overdose will cause increased anion gap metabolic acidosis
Ans. A to E.
  1. Regarding Endocrinologic test to differentiate Cushing's disease, Ectopic ACTH production and Adrenal tumors
    1. Low dose dexamethasone test suppresses cortisol in ectopic ACTH production and adrenal tumor
    2. High dose dexamethasone test suppresses cortisol in ectopic ACTH production and adrenal tumor
    3. Serum ACTH will be low in adrenal tumor
    4. In metapyrone test which suppres cortisol synthesis in most patients with Cushing's disease will show a rise in 17-OHCS in urine of 70% above baseline145
    5. CRH stimulation test positive in Cushing's disease and negative in ectopic ACTH production and adrenal tumor.
Ans. C to E.
  • Petrosal sinus sampling is the best way to distinguish pituitary ACTH producing tumor from an ectopic ACTH producing tumor
  • Low dose dexamethasone do not suppress cortisol in Cushing disease or ectopic ACTH production
  • High dose dexamethasone do not suppress cortisol in ectopic ACTH secretion but suppress in Cushing's disease
  1. Differential diagnosis of Pituitary stalk thickening is all EXCEPT
    1. Lymphoma
    2. Lymphocytic hypophysitis
    3. Granulomatous disease
    4. Hypothalmic glioma
    5. Pituitary adenoma
Ans. E
  • Normal thickness of pituitary stalk is approximately equal to basilar artery diameter
  • If there is selective decrease in one pituitary hormone and thickened pituitary stalk, the diagnosis is lymphocytic hypophysitis
  1. Regarding treatment of pituitary adenoma
    1. Usually medical treatment is treatment of choice for prolactinoma in patients having level >500 ng/ml
    2. Surgery is the treatment of choice in GH and ACTH secreting adenoma
    3. Ketoconazole, Metapyrone, Mitotane, Cyproheptadine all can be used to treat Cushing's disease
    4. Somatostatin analogue octreotide is used for treatment of GH adenoma
    5. Bromocriptine can be used for GH secreting adenoma
Ans. All of the above are true
  • Usually Transphenoidal surgery is done except in certain circumstances146
  1. Epstein-Barr virus is associated with how many % cases of primary CNS lymphoma
    1. 30%
    2. 50%
    3. 80%
    4. 100%
Ans. D.
  • Epstein-Barr virus is detectable in 30–50% of systemic lymphomas however, it has been associated with 100% of PCNSL
  1. Which of the following tumors arise from remnant of notochord?
    1. Chordoma
    2. Chondroma
    3. Chondrosarcoma
    4. All of the above
Ans. A
  • Chordoma arise from remnant of notochord and have characteristic physaliphorous cells
  1. Match the following grading
    A. Simpson grading
    Extent of removal of glioma
    B. House and Brackmann grading
    Assesment of function of VIII gradingnerve
    C. Hess and Hunt scale
    Subarachnoid hemorrhage
    D. Modified Gardener and Robertson scale
    Related to VII nerve
    E. Modified Jackson classificationtumor
    Relate to Glomus jugulare
Ans. C and E
  • Simpson grading is related with the extent of removal of meningioma
  • House and Brackmann grading is for assessment of the function of facial nerve [VII nerve]
  • Modified Gardener and Robertson scale is related to testing of cochlear function of VIII nerve
  1. The following primitive neuroectodermal tumor (PNET) are
    1. Medulloblastoma
    2. Pinealoblastoma
    3. Ependymoblastoma147
    4. Neuroblastoma
    5. Esthesioneuroblastoma
Ans. A to E.
  • Also Retinoblastoma, and Polar Spongioblastoma
  1. About Dermoid and Epidermoid cyst
    1. Growth rate is exponential rather than linear
    2. Epidermoid constitute 5-1.5% of intracranial tumor and epidermoid constitute 3% of intracranial tumor
    3. Epidermoid tumors causes bacterial meningitis, always occur in the midline and associated with congenital malformation in 50% of cases.
    4. Dermoid tumor causes aseptic meningitis, occur laterally and not associated with congenital formation
    5. Both can be treated by surgery
Ans. E.
  • Growth is linear like the skin. It is not exponential which is in malignant tumor
  • Dermoid constitute 3% of intracranial tumor and epidermoid constitute 5-1.5% of intracranial tumor
  • Epidermoid tumor causes aseptic meningitis, always occur laterally and is not associated with congenital malformation
  • Dermoid tumor causes bacterial meningitis (commonly associated with dermal sinus), occur in midline and associated with congenital malformation in 50% of cases. Radiotherapy is not given. Surgical excision is the treatment of choice
  1. Tumor arising in pineal region are
    1. Pineocytomas and pineoblastomas
    2. Astrocytoma
    3. Meningioma
    4. Ependymoma and chemodectoma
    5. Geminoma and choriocarcinoma
Ans. All of the above are true
Substrate in pineal region
Tumor that may arise
Pineal glandular tissue
Pineocytomas and pineoblastomas
Glial cells
Astrocytomas (including pilocytic) oligodendroglioma, glial cyst
148
Arachnoid cell
Meningiomas, arachnoid cyst
Ependymal cell
Ependymoma
Sympathetic nerve
Chemodectomas
Rests of germ cells
Germ cell tumors, choriocarcinoma,
  1. Poppen/Jamiesson approach for surgery of pineal tumor is
    1. Infratentonial supracerebellar approach
    2. Paramedian transtentorial approach
    3. Occipital transtentorial approach
    4. Transventricular approach
Ans. C.
  • Krause/Stein approach is infratentorial supracerebellar approach
    Important surgical consideration
  • Deep cerebral veins are a major obstacle to operation in this region
  • The base of the pineal gland is the posterior wall of the 3rd ventricle. The splenium of the corpus callosum lies above and the thalamus surrounds both sides. The pineal projects posteriorly and inferiorly into the quadrigeminal cistern
  1. In Central nervous system melanocytes are concentrated in the
    1. Choroid plexus
    2. Red nuclei
    3. Region of the amygdala
    4. Septum pellucidum
    5. Ventral medulla
Ans. E
  • Melanocytes present in the pia-mater are concentrated in the ventral medulla and upper spinal cord
  • Primary CNS melanoma arise from melanocytes in the leptomeninges, and spread through CSF. Peak age of this tumor is fourth decade
  1. An infant evaluated for hydrocephalus has dilated ventricles with a thin cortical mantle and normal fontanelle. The management will be
    1. Regular measurement of head circumference
    2. Immediately ventriculoperitoneal shunt or CSF diversion
    3. Start Acetazolamide
    4. Immediately lumboperitoneal shunt
    5. Angiography and EEG may help in evaluation
149
Ans. A, C amd E.
  • This condition is likely to be cerebral atrophy (early) stage with thin cortical mantle. So shunt has to be done but not immediately as fontanelle are normal. Lumboperitoneal shunt is avoided in children. Acetazolamide is usually started when the patient can be stabilized before going expectant surgery. To differentiate hydranencephaly from maximal hydrocephalus, EEG is used which will show no change in hydranencephaly
  1. Match the following plain X-ray radiological finding with the causative factor
    1. Hemangioma → Honey comb appearance common
    2. Osteoma → Discrete high density lesion with smooth contour
    3. Epidermoid cyst → Scalloping border with sclerotic rim
    4. Eosinophilic granuloma → Discrete radiolucent area
    5. Multiple myeloma → Multiple discrete round lesions
Ans. All of the abvoe are true
  • Eosinophilic granuloma involving cervical vertebrae are called vertebrae plana
  1. The most common primary CNS tumor responsible for extracranial spread (systemic) is
    1. Medulloblastoma (cerebellar PNET)
    2. Meningiomas
    3. Malignant astrocytomas
    4. Ependymomas
    5. Pinealoblastoma
Ans. A.
  • Medulloblastoma is the most common primary CNS tumor for extraneural spread
  1. About Radiotherapy in CNS tumors
    1. The most common external beam radiation therapy regimen for brain metastases is 30Gy in 10 fractions over 2 weeks
    2. The most appropriate radiation protocol for gliomas is 6000 cGy in 200 cGy daily fractions
    3. Stereotactic radiosurgery has not been proven to be more effective for treatment of malignant gliomas
    4. Cerebral PNET's are usually localized and require local radiation after surgery
    5. Germinomas are very sensitive to radiotherapy
150
Ans. A, B, C and E.
  • Cerebral PNET (most common medulloblastoma) has high propensity for metastases. Therefore craniospinal irradiation is given prophylactically
  1. Failure of a mitotic pupil to dilate after instilling 2–10% cocaine followed by 1% hydroxyamphetamine indicates a
    1. First–order Horner's syndrome
    2. Second–order Horner's syndrome
    3. Third–order Horner's syndrome
    4. First–or second–order Horner's syndrome
    5. Second-or third–order Horner's syndrome
Ans. C.
  • Horner's syndrome can be confirmed by the failure of the mitotic pupil to dilate in response to 2–10% cocaine drops. If the later application of the adrenergic mydriatic hydroxyamphetamine has no effect, then the lesion localizes to the third–order neuron (postganglionic part). A first or second-order lesion is indicated by a failure of the miotic pupil to dilate to cocaine drops followed by dilation (after 24 hours) with 1% hydroxyamphetamine.
  1. Radiosensitive cerebral metastases are
    1. Small cell lung Ca
    2. Large cell lung Ca
    3. Lymphoma
    4. Germ cell tumors
    5. Malignant melanoma
Ans. A, C and D
  • Leukemia and Multiple myeloma are also sensitive
  1. A male patient aged 50 years is having multiple cerebral metastases. He is posted for emergency surgery due to presence of life threatening lesion. These lesions include
    1. Posterior fossa lesion
    2. Frontal lobe lesion
    3. Occipital lobe lesion
    4. Parietal lobe lesion
    5. Large temporal lobe lesion
151
Ans. A and E.
  • Posterior fossa lesion leads to compression of vital centers early. Large temporal lobe lesion leads to early transtentorial herniation
  1. Regarding Idiopathic intracranial hypertension (IIH) or pseudotumor cereberi or benign intracranial hypertension
    1. Elevated ICP and papilledema (10%) without intracranial mass
    2. CSF has normal cell count and sugar level but elevated protein level
    3. Abducens nerve palsy (20%), enlarged blind spot (66%) and visual field defect (9%) is present
    4. Visual loss in Idiopathic Intracranial Hypertension may occur early or late, may be sudden or gradually progressive and is not reliably correlated to duration of symptoms
    5. There is conspicuous absence of altered level of consciousness in spite of high ICP
Ans. A, C to E.
  • In IIH or pseudotumor cerebri, the protein is normal or even low <20% but never high
  1. An elderly patient present with proximal muscle stiffness with paroxysmal lancinating pain in temporal area present to you with increased ESR, the management will be
    1. Steriod should be started immediately
    2. CT scan or MRI scan should be done to rule out intracranial pathology
    3. Full investigation should be done before starting treatment
    4. Symptomatic treatment should be given and patient must be observed for any developing focal deficit
Ans. A.
  • This patient is having temporal arteritis. Fifty percent of patients with temporal arteritis have polymyalgia rheumatica which causes increase proximal muscle stiffness. Often there is risk of blindness. In case of slightest suspicion, steroid has to be started immediately152
  1. About Empty Sella Syndrome
    1. In primary empty sella syndrome, most of these patients are obese women
    2. The frequency of empty sella syndrome (primary) is higher in patients with pituitary tumors and in those with increased ICP for any reason
    3. Most patients usually present with symptoms that do not suggest an intrasellar abnormality including headache, dizziness, seizures, etc
    4. Clinically evident endocrine disturbances are rare with primary empty sella syndrome
    5. Surgery is usually not indicated except in case of CSF rhinorrhoea
Ans. All of the above are true
  1. A single CAT scan of the head produces how much radiation exposure
    1. 1–40 REM
    2. 10–40 REM
    3. 18–40 REM
    4. 10–20 REM
Ans. C.
  • I Sievert = 100 REM
  • 1 REM is estimated to cause = 300 additional cases of cancer per million persons. The average annual exposure to radiation is 360 MREM. A CXR produces about 10–40 MREM of exposure. A CAT scan of the head = 18–40 REM (1.25 REM/slice) a cerebral arteriogram = 10–20 REM (including fluoroscopy)
  1. Nerves arising from posterior cord are
    1. Thoracodorsal nerve
    2. Dorsal scapular nerve
    3. Supra scapular nerve
    4. Upper sub-subscapular
    5. Lower sub-subscapular
Ans. A, D and E.
  • Other branches of posterior cord are axillary and radial nerve153
  1. Following statement about brachial plexus lesion with the description are
    1. In middle trunk lesion, the median sensory responses from the index and middle finger are low in amplitude, but motor conduction velocities of the hand muscles are normal
    2. In lower trunk lesion, the ulnar sensory response from the little finger is abnormal and electromyographic exam of the extensor indicis or indicus proprius and abductor pollicis longus is abnormal
    3. In medial cord lesion, the ulnar sensory response from the little finger is abnormal but normal response from the extensor indicis proprius
    4. In upper trunk lesion, the action potentials from the deltoid and biceps are of low amplitude
    5. In lateral cord lesion, there is abnormal median sensory responses and denervation in the biceps and flexor carpi radialis and normal response from the adductor pollicis brevis
Ans. All of the above are true
  1. Match the following
    A. Supraspinatus
    Subscapular nerve
    B. Pollicis brevis (deep head)
    Median nerve
    C. Palmaris longus
    Ulnar nerve
    D. Adductor pollicis
    Ulnar nerve
    E. Brachioradialis
    Radial nerve
Ans. D and E.
  • Supraspinatus is supplied by suprascapular nerve (branch of upper trunk)
  • Flexor pollicis brevis (deep head) is supplied by ulnar nerve and superficial head is supplied by median nerve
  • Palmaris longus is supplied by median nerve but palmaris brevis is supplied by ulnar nerve
  1. Peripheral neuropathy is seen in
    1. Zidovudine
    2. Lamivudine
    3. Didanosine
    4. Stavudine
    5. Zalcitabine
154
Ans. C to E.
  • Didanosine causes a painful dose related neuropathy
  • Zalcitabine dose related neuropathy is severe and persistent
  • Ritonavir (protease inhibitor) can cause peripheral paresthesias
  • Amprenavir can cause perioral paresthesias. Zidovudine causes toxic mitochondrial myopathy
  • Distal sensory polyneuropathy is the most common neuropathy in patients with AIDS
  1. Match the following
    1. Greater occipital nerve/Nerve of Arnold
    2. Nervus intermedius/Nerve of Wrisberg
    3. Long thoracic nerve/Nerve of Bell
    4. All the above are false
Ans. A to C.
  1. Anterior interosseous nerve supplies
    1. Abductor pollicis brevis
    2. Flexor pollicis brevis
    3. Pronator teres
    4. Flexor pollicis longus
    5. Pronator quadratus
Ans. D and E.
  • Anterior interosseous nerve involvement result in Kiloh-Nervin syndrome
  • Abductor pollicis brevis, flexor pollicis brevis, pronator teres are supplied by median nerve
  • Anterior interosseous nerve which is pure motor branch of median nerve, supplies flexor digitorum profundus I and II, pronator quadratus, flexor pollicis longus
  1. All of the following can be seen in ulnar nerve entrapment at the wrist
    1. Motor deficits in the adductor pollicis
    2. Motor deficits in the deep head of the flexor pollicis brevis.
    3. Motor deficit in the third and fourth lumbricals
    4. Sensory deficit in the dorsum of hand
    5. Sensory deficit in the palmar surface of the hypothenar muscle
Ans. A, B, C and E155
  1. Muscles supplied by Posterior interosseous nerve
    1. Triceps brachii
    2. Supinator
    3. Extensor carpi radialis longus
    4. Brachioradialis
    5. Extensor carpi ulnaris
Ans. E.
  • Posterior interosseous nerve supplies extensor carpi ulnaris, extensor digitorum, extensor digiti minimi, extensor pollicis brevis and longus, abductor pollicis longus, and extensor indicus
  1. If the injury to the radial nerve occur in the radial groove, the muscles which are involved are
    1. Long head of triceps
    2. Lateral head of triceps
    3. Medial head of triceps
    4. Extensor carpi radialis longus
    5. Extensor carpi radialis brevis
Ans. B, D and E
  • Branches to long head of triceps and medial head of triceps given before entering into the radial groove
  1. Match the following nerves with the structure causing compression
    1. Median nerve/TCL (Transverse carpal ligament)
    2. Suprascapular nerve/TSL (Transverse scapular ligament)
    3. Median nerve/Struther's ligament
    4. Posterior interosseus nerve/arcade of Frohse
    5. Ulnar nerve/Arcade of Struther
Ans. All of the above are true
  • Guyon's canal contain ulnar nerve
  • Cubital tunnel syndrome also involves ulnar nerve
  • Supinator tunnel syndrome involves radial nerve
  1. In Kiloh-Nervin syndrome associated with nerve entrapment neuropathy
    1. There is no sensory loss
    2. Both sensory and motor deficit
    3. Paralysis of pronator teres muscle156
    4. Paralysis of fexor digitorum profundus [I and II]
    5. Paralysis of flexor pollicis longus
Ans. A. D and E.
  • Kiloh-Nervin syndrome is due to anterior interosseous nerve entrapment, pure motor branch of Median nerve supplying Flexor digitorum profundus [I and II], Flexor pollicis longus and pronator Quadratus
  1. Function of Palmar interosseous muscle
    1. Flexion at MCP joint
    2. Extension of MCP joint
    3. Adduction at MCP joint
    4. Extension at PIP joint
    5. Extension at DIP joint
Ans. A, C and D.
  1. Muscles having dual nerve supply are
    1. Bicepbrachii
    2. Brachialis
    3. Flexor digitorum profundus
    4. Bicep femoris
    5. Flexor pollicis brevis
Ans. B to E.
  • Bicepbrachii is supplied by musculocutaneous nerve. Brachialis is supplied mainly by musculocutaneous nerve but lateral part is supplied by radial nerve also
  • Flexor Digitorum Profundus – Medial part supplied by ulnar nerve
  • FDP Lateral part supplied by Median nerve (anterior interosseous branch)
  • Adductor Magnus supplied by Obturator, Sciatic
  • Bicepfemoris (Long head) supplied by Sciatic
  • Bicep femoris (Short head) supplied by Common peroneal nerve
  • Flexor pollicis brevis (Superficial head) supplied by Median nerve
  • Flexor pollicis brevis (Deep head) supplied by ulnar nerve
  • Pectineus also has double supply by Femoral nerve and; Obturator nerve157
  1. About Trendelenburg sign
    1. Due to paralysis of contralateral gluteus medius/gluteus minimus muscle
    2. Due to paralysis of ipsilateral gluteus medius/gluteus minimus muscle
    3. Involvement of inferior gluteal nerve (Ipsilateral)
    4. Involvement of superior gluteal nerve (Ipsilateral)
    5. May present in L4 – L5 disc herniation
Ans. B, D and E.
  • Trendelenburg sign → occurs when the pelvis tilts down toward the side of the lifted leg indicating weakness of the contralateral thigh abductors (primarily L5 innervated) or ipsilateral thigh abductor Gluteus medius and minimus causes abduction and medial rotation
  1. Statement regarding the waves in the somatosensory evoked potential
    1. N9 wave absence or delay implies peripheral nerve disease
    2. N11 wave absence orderly implies cervical cord disease
    3. N13/P13 wave absence or delay implies a lesion in the lower medulla
    4. N18 wave absence or delay implies a lesion in the upper pons or midbrain
    5. N19 wave absence or delay implies a lesion in the thalamo sensory cortex
Ans. All of the above are true
  1. About motor potential during EMG
    1. Decreased voltage and decreased duration is found in myopathy
    2. Prolonged, low amplitude and polyphasic motor unit potentials are typical in early reinnervation
    3. Prolonged, high amplitude and polyphasic motor unit potential are typical in late reinnervation
    4. Fibrillation potential are following denervation of a muscle where individual muscle fibers begin firing independently and is not visible through skin.
    5. In plexopathy, there is paraspinal muscle fibrillation with normal sensory nerve action potential but in radiculopathy, there is no paraspinal muscle fibrillation but there is reduced sensory nerve action poiential (SNAP)158
Ans. A to D.
  • In plexopathy, there is reduced sensory nerve action potential and no paraspinal muscle fibrillation
  • In radiculopathy, there is normal sensory nerve action potential and paraspinal muscle fibrillation present
  • In myotonic dystrophy classic EMG finding “dive bomber” sound due to myotonic discharges
  • Following disectomy for radiculopathy motor potential return if first nerve injury were complete, it would take a month to return. Lost sensory potentials return last or may not return. Paraspinal muscle potential may no longer be useful because the muscles are cut during surgery
  1. Which of the following water soluble contrast agent is used both IV and intrathecally and is most preferably used in contrast studies?
    1. Iohexol (omnipaque)
    2. Diatrizoate meglumine (Reno-60)
    3. Ioversol (optiray)
    4. Metrizamide (Anaipaque)
    5. Pantopaque
Ans. A.
  • Diatrizoate meglumine and Ioversol are not for intrathecal use. Metrizamide is superseeded by iohexal and has higher risk of seizures and significant nausea and vomiting
  • Pantopaqe is non soluble contrast
  1. At how much haematocrit does an acute SDH is isodense to brain on CAT scan?
    1. <23%
    2. <32%
    3. <13%
    4. <23%
Ans. A
  • Hct <23% will cause an acute SDH to be isodense with brain
  1. Match the following Hounsfield unit
    1. Air - − 1000 (Hu)
    2. Gray matter - 30 to 40 Hu
    3. White matter - 20 to 35 Hu
    4. CSF - +5 Hu
    5. Fresh SAH - 75 to 80 Hu
159
Ans. A to E.
  • Water – 0 Hu
  • Densebone - + 1000 Hu
  • Cerebral edema – [10 to 14] Hu
  • Bone - + 600 Hu
  • Fat – [30 – 40] Hu
  • Disc material [55 – 70] Hu
  • Thecal sac – [20 –30] Hu
  1. Which is the most common primitive persistent Carotid – Basilar anastomoses?
    1. Primitive trigeminal artery
    2. Primitive otic artery
    3. Primitive hypoglossal artery
    4. Proatlantal intersegmental artery
Ans. A.
  • Primitive trigeminal artery is the most common of the persistent anastomoses. It anastomose proximal cavernous internal carotid artery to basilar artery
  • Primitive otic artery connects petrous internal carotid artery to basilar artery
  • Persistent primitive trigeminal artery is seen in 0.6% of cerebral angiogram
  1. Which of the following is also called as “Italian artery”?
    1. Artery of Mc Conell AKA capsular artery
    2. Inferior hypophyseal artery
    3. Superior hypophyseal artery
    4. Artery of Bernasconi and cassinari AKA artery of tentorium
Ans. D.
  • Artery of Bernasconi and cassinari AKA Italian, artery. This artery is enlarged in tentorial meningiomas
  1. Infundibulum which is funnel shaped initial segment of an artery has to be distinguished from aneurysm, is most commonly found at the origin of
    1. Anterior cerebral artery
    2. Anterior choroidal artery
    3. Posterior communicating artery
    4. Middle cerebral artery
160
Ans. C.
  • Criteria of an infundibulum (1) Triangular in shape (2) Mouth (widest portion) <3 mm) (3) Vessel at apex
  1. Regarding magnetic resonance imaging the following statements are
    1. Fat is high signal in T2WI and low signal in T1WI
    2. Bone is low signal on both T2WI and T1WI
    3. CSF is high signal on T2WI but low signal on T1WI
    4. Gray matter is low signal on T1WI but high signal on T2WI and for white matter it is just reverse
    5. T1WI show pathological image and T2WI shows anatomical image
Ans. B to D.
  • Proton rich tissue (eg H2O) has long T1 and T2
  • In T1WI short T1 produce high signal
  • In T2WI long T2 produce high signal
  • Paramagnetic substance like methaemoglobin when intracellular produce short T1 and short T2
  • When extracellular produce short T1 and long T2
  1. Which of the following aneurysmal clip is not MRI compatible?
    1. Mayfield
    2. Yasargil (Phynox)
    3. Sugita (Elgiloy)
    4. Silver clip
Ans. A.
  • Stainless steel is classified as martensitic (ferromagnetic) or austenitic (non-ferromagnetic). Cobalt- based super alloys are non ferromagnetic and include Elgiloy (Sugita clip), Phynox (Yasargil) and Vari angle are MRI compatible
  1. Cervical spinal stenosis is present when on AP diameter on a plain lateral C-spin x-ray the AP diameter is
    1. <12mm
    2. <14mm
    3. <15mm
    4. <17mm
161
Ans. A.
  • Normal canal diameter on lateral C-spin X-ray (from spinolaminar line (SLL) to posterior vertebral body is [17± 5mm] (12–22mm)
  1. Two neurocentral synchondrosis on atlas fuse at about
    1. 8 years
    2. 7 years
    3. 10 years
    4. 12 years
Ans. B.
  • Secondary ossification center
  1. Secondary ossification center which appears at summit of dens between age 3–6 years fuses with dens by age
    1. 10 years
    2. 12 years
    3. 14 years
    4. 16 years
Ans. B
  1. The lumbar disc space with the greatest vertical height is
    1. L1 – L2
    2. L2 – L3
    3. L3 – L4
    4. L4 – L5
Ans. D.
  1. Which of the following radiological measurements is the most sensitive for basilar impression (BI)?
    1. McRae's line
    2. McGregor's baseline
    3. Chamberlain's line
    4. Weckenheim Clivus Canal Line
Ans. B
  • McRae's line is drawn across foramen magnum (tip of clivus) (basion) to (opisthion) and no part of odontoid should be above this line. It is most specific
  • McGregor's baseline is drawn from posterior margin of hard palate to most caudal point of occiput. No more than 4.5 mm of dens should be above this. It is most sensitive162
  1. Oscillopsia is present in
    1. Arnold Chiari Malformation 1
    2. Multiple sclerosis
    3. Aminoglycoside ototoxicity
    4. Bilateral vestibular neurectomies
    5. Dandy's syndrome
Ans. All of the above are true
  • Oscillopsia – visual sensation that stationary objects are swaying side to side or vibrating
  1. Statements regarding audiometric testing are
    1. In conductive loss, there is equal threshold elevation for each frequency
    2. In early Meniere's disease, threshold are elevated more in lower than in higher frequency
    3. In noise induced hearing loss, threshold is greater in lower than in high frequency
    4. Sensory neural deafness tend to have greater threshold elevation at each higher frequency
    5. In acoustic schwannoma, loudness recruitment is characteristic
Ans. A to D.
  • Loudness recruitment is characterstic of cochlear lesion and not retrocochlear lesion
  1. Facial diplegia (Bilateral facial palsy) is present in
    1. Moebius syndrome
    2. Myotonic dystrophy
    3. Neurosarcoidosis
    4. Lyme's disease
    5. Gullian Barre syndrome
Ans. All of the above are true
  • In neurosarcoidosis, VII is the most commonly affected cranial nerve. In Lyme disease, facial diplegia is a hallmark
  • In Moebius syndrome, it affects upper face more than lower face
  • In tuberculous meningitis, VI is the most common nerve to be involved163
  1. About Bell's Palsy
    1. Most cases probably represent a viral inflammatory demyelinating polyneuritis due to the herpes simplex virus
    2. Usually exhibit proximal to distal progression
    3. 100% of cases recover completely
    4. Per cases with focal injury to the facial nerve (e.g., trauma, injury during operation) dynamic reconstruction by nerve anastamoses are usually considered superior to static method and without focal cause, e.g. Bell's palsy only “static” method applicable
    5. If paralysis is complete at onset, 50% will have incomplete recovery
Ans. A, D and E.
  • It usually exhibits distal to proximal progression
  • 75 – 80% of cases recover completely
  1. Extracranial facial nerve anastomosis can be done by
    1. Hypoglossal nerve (Cr. N-XII)
    2. Spinal accessory nerve
    3. Phrenic neve
    4. Glossopharyngeal
    5. Crossface grafting (VII – VII)
Ans. All of the above are true
  1. Head injury is most frequent among which age group in India
    1. Children less than 5 years
    2. 20 – 40 years
    3. 40 – 60 years
    4. > 60 years
Ans. B.
Age Group
% of all Head Injuries
• Children below 20 years
30
• Young adult 20 – 40 years
60
• Elderly above 60 years
5
• Not surprisingly incidence of head injury is lowest in extremes of age, below five years and above 60 years and the rate is 125 – 150/100,000 population
164
  1. Statements regarding head injury and its mechanism are
    1. Males had higher incidence of head injury as compared to females also in all age groups except infants and elderly patients above 65 years of age
    2. Angular acceleration is most injurious
    3. Angular acceleration is a combination of both translational and rotational component. Higher the center of angulation, greater the rotational component and at lower center of angulation, the translational component increases
    4. Structural damage to superficial vascular tissue (bridging veins and pial vessels) occur in short acceleration duration with large acceleration magnitude whereas brain tissue injury occurs in longer duration and lesser acceleration magnitude
    5. Skull fractures, epidural hematoma coup contusions are due to result of contact injuries while countercoup contusions are due to head acceleration (inertial effect)
Ans. All of the above are true
  • Centre of angulation is usually mid cervical to lower cervical region
  1. Which statements about mechanism are correct?
    1. SDH (subdural hematoma) result from head acceleration that produce short duration, high strain ratio loading
    2. Concussion refers to an immediate but transient loss of consciousness associated with a short period of amnesia caused due to transient electrophysiologic dysfunction of reticular activating system in the upper midbrain.
    3. Almost all cases of severe diffuse axonal injury, arise from vehicular accident in which acceleration is long
    4. Sagittal acceleration occasionally produce Diffuse Axonal Injury of grade I. Angular deacceleration in coronal plane has a high incidence of severe form of Diffuse Axonal injury
    5. Linear skull fracture follows local skull bending occurring at the site of impact which exceeds the strain limit for the bone.
Ans. All of the above are true
  • Remember, in concussion most of the strain is insufficient to cause structural damage but biochemical and ultrastructural changes such as mitochondrial ATP depletion and local disruptions BBB occur165
  1. Acceleration injury will result in
    1. Coup contusion
    2. Concussion
    3. Countercoup contusion
    4. Extradural hematoma
    5. Subdural hematoma
Ans. B, C and E
  • DAI is also following acceleration injury
  1. Ca2+ pump pumps out how many Ca2+ ion out of cell per ATP
    1. 1
    2. 2
    3. 3
    4. 4
Ans. B.
  1. Head trauma results in
    1. ↑ extracellular K+
    2. ↑ intracellular Ca2+
    3. ↑ extracellular glutamate
    4. ↑ adenosine
    5. ↓ glutamine intracellulary
Ans. All of the above are true
  • Note: Adenosine increases CBF and has neuro protective role as it inhibits glutamate release
  1. Which statements are correct?
    1. Intracranial volume of an adult is approx 1500 ml which is about 2% of total body weight
    2. Total CSF volume in normal condition is about 120ml to 140 ml, ventricular volume is about 40 ml, spinal subarachnoid space contains about 30ml and the remaining CSF occupies the cranial subarachnoid space and cistern
    3. The total cerebral blood volume is approx. 150 ml
    4. Mean CBF is 54 ml/100 gm/min with gray matter and white matter flow being 70 and 20 ml/100 gm/min respectively
    5. Cerebral autoregulation fails below 70 mmHg and above 150 mm/Hg
    166
Ans. A to D
  • Cerebral autoregulation is maintained between 50–150 mmHg. It has to drop below 40 mHg if the blood supply to brain is to be hampered.
  1. Following statement regarding compliance and elastance of brain are?
    1. Cerebral compliance of an adult brain is greater than that of infant
    2. Cerebral elastance of an infant brain is greater than that of adult brain
    3. Cerebral capacitance in case of extradural dural hematoma is greater than that of subdural hematoma
    4. Cerebral capacitance in case of meningioma is lower than that of glioblastoma multiform
    5. Slope of pressure volume curve is elastance
Ans. E.
  • Compliance – Adult skull ↓;Infant skull ↑;dv/dp
  • Elastance – Adult skull ↑;Infant skull ↓;dp/dv
  • Cerebral capacitance represents the rate at which the brain can accommodate change in intracranial volume and is determined by time dependent derivative of the same variables determining cerebral compliance
  1. A patient is having diffuse axonal injury, histologically there are axonal swelling (retraction ball) in cerebral white matter, corpus callosum, dorsolateral quadrant upper brainstem and superior cerebellar peduncle and a focal lesion in the corpus callosum. He is having
    1. Grade I DAI
    2. Grade II DAI
    3. Grade III DAI
    4. None of the above
    5. All of the above
Ans. B.
  • Diffuse axonal injury was first described by Stritch in 1956
  • Grade I diffuse axonal injury less common in cerebellum without focal lesion
  • Grade II diffuse axonal injury with focal lesion in corpus callosun
  • Grade III diffuse axonal injury with focal lesion in corpus callosum and dorsolateral quadrant of brainstem167
  1. About contusions
    1. Overlying pia mater always remains intact
    2. Coup contusion and countercoup contusion has the same mechanism of formation
    3. Plaque Jaune are common with countercoup lesion and are found over inferior frontal lobe and temporal tip and occipital pole
    4. It is wedge shape with base toward the surface.
    5. Herniation contusion are most frequently located along the margin of the falx cerebri, tentorium or foramen magnum
Ans. A, C to E
  • In contusion overlying pia mater always remain intact but in laceration it is breached
  1. Blood brain barrier is formed by
    1. Choroidepithelial cell
    2. Endothelial cell
    3. Foot process of astrocytes
    4. Process of oligodendroglia
    5. Pia mater
Ans. B and C.
  • Tight junction between choroids epithelial cell form blood-CSF barrier (Arachnoid cells) which also have barrier function
  1. Following are indicated in brain edema of head injury
    1. Mild hypothermia (33 – 34°C)
    2. Barbiturate
    3. Phencyclidine
    4. Selenium
    5. Corticosteroids
Ans. A,B and D
  • Vit. E, selenium, superoxide dismutase reduce the free radical and stop lipid peroxidation
  • Phencyclidine dextromethorphan, Ketamine, MK-801 are NMDA antagonist and have been used in experimental traumatic model. They have side effect which limit their use except dextromethorphan which have role in future168
  1. Regarding vasospasm in subarachnoid hemorrhage
    1. Onset is almost never before day 3 post SAH
    2. Maximal frequency of onset during day 6 – 8 post SAH
    3. Vasospasm leading to clinical deterioration between 5th and 9th day
    4. Clinical cerbrovascular spasm almost always resolved by day 12 post SAH and is radiographically demonstrated over 3 – 4 weeks
    5. Onset is usually insidious but 10% have an abrupt and severe deterioration
Ans. All of the above are true
  • Hypoxia and Hypotension are the two most important causes which lead to ischaemic brain damage
  1. Statement regarding Neurentic Cyst are
    1. Most common alternate term is enterogenous cyst
    2. Occur due to persistence of foramen of kovalavescky
    3. Most are simple cyst lined by cuboidal columnar epithelium and mucin secreting globlet cell
    4. Recurrent meningitis can occur
    5. Most commonly present during the second decade of life
Ans. A to D
  • Most commonly present during the first decade of life. Pain or myelopathy are the most common presentation in older children and adult. Neonate and young children may present with cardiorespiratory compromise from an intrathoracic mass or cervical spinal cord compression
  1. Which of the following drug raises the cellular pH and combat acidosis and is used widely in head injury patients?
    1. Phencyclidine
    2. Dextromethorphan
    3. MK - 801
    4. Tris Hydroxyaminothane (THAM)
Ans. D
  • Rest of the drugs are NMDA antagonist and are being used in traumatic experimental study169
  1. One and half syndrome is due to destruction of
    1. Both PPRF (Bilateral)
    2. Both PPRF and MLF (Bilateral)
    3. Both MLF (Bilateral)
    4. Both PPRF and MLF (Unilateral)
Ans. D
  1. At birth, the brain is about what % of its adult weight
    1. 25%
    2. 40%
    3. 60%
    4. 75%
Ans. A.
  1. About Investigation
    1. Non haemorrhagic lesion are better evaluated by MRI than CT scan
    2. CT is better for acute SDH than MRI
    3. MRI is more sensitive than CT in identifying isodense SDH
    4. Subarachnoid hemorrhage is seen better in CT than MRI
    5. 50 – 40% linear skull fracture can be missed on axial projection in CT
Ans. All of the above are true
  • For diagnosing DAI, MRI is superior to CT
  1. About lesions in diffuse axonal injury (DAI)
    1. All are haemorrhagic
    2. 80% are haemorrhagic and 20% are non haemorrhagic
    3. 20% are haemorrhagic and 80% are non haemorrhagic
    4. All are non haemorrhagic
Ans. C.
  • The vast majority (80%) of DAI lesions are non haemorrhagic and only about 20% of lesions contain small central areas of petechial hemorrhage
  1. About position of aneurysm
    1. Traumatic aneurysms are usually more peripherally situated and typically do not occur at branching site
    2. Traumatic aneurysms located at the distal middle cerebral artery, distal ACA and basal ICA are less 1% of then reported aneurysm170
    3. Mycotic aneurysms occur in 3 – 15% of the patients with infective endocarditis and are located peripherally in the middle cerebral artery and fusiform
    4. Fungal infection causing aneurysm tends to involve large cranial vessel than bacterial infection
    5. Berry aneurysms are more common in anterior (internal carotid artery) circulation and are saccular in shape rather than most posterior circulation aneurysms which are fusiform in shape
Ans. All of the above are true
  1. The intracranial hypertension after trauma is maximum between
    1. 0 – 24 hr
    2. 24 – 48 hr
    3. 48 – 72 hr
    4. 72 – 96 hr
Ans. C
  • In a few patients, secondary raised ICP can occur between 7 – 10 days after treatment
  1. Among all the factors in GCS the most PROGNOSTIC is
    1. Verbal response
    2. Eye opening
    3. Motor response
    4. All of the above
Ans. C.
  • Amongst the all factors motor scoring is probably the most prognostic
  1. Regarding mannitol use in head injury for reduction of brain oedema
    1. Mannitol is used at a dose of .25gm – 1gm/kg
    2. Serum osmolality should not exceed 320mosmol/L
    3. Repeated long term use result in dilutional hyponatremia, acute renal failure and disruption of BBB
    4. It has a neuroprotective role as it scavanges free radical which produce lipid peroxidation
    5. They can be used sometime conjunctive to chemotherapy for the Glioma to increase BBB penetrance
Ans. All of the above are true171
  1. About factors influencing outcome in head injury patients
    1. Incidence of surgically treatable hematomas increases with age
    2. In children, mortality is highest at 1–2 years and decreases till adolscence being minimum at 12 years
    3. Motor component alone has been used as single important predictor. Flaccidity has worse outcome (-100%) mortality
    4. Patients with normal oculovestibular reflex were generally free of significant deficit 2 years after the injury
    5. Patients with GCS ≤8 are comatose and those ≥9 are not
Ans. All of the above are true
  1. The triad of head injury with lucid interval, mydriasis on the side of a hematoma and contralateral paresis occur in how many % of cases of EDH
    1. 18%
    2. 28%
    3. 38%
    4. 48%
Ans. A.
  • i.e., 18% that too in temporoparietal region
  • Clinical cause of presentation of EDH - 5 pattern
  • Conscious throughout (8–24%)
  • Unconscious throughout (23–24%)
  • Initially unconscious and subsequently recovered (20–28%)
  • Initially conscious followed by loss of consciousness (14–21%)
  • Initially unconscious followed by recovery, further followed by a second loss of consciousness (12–34%)
  1. Which anatomical landmark correspond to transverse sinus?
    1. Inferior nuchal line
    2. Superior nuchal line
    3. Asterion
    4. Pterion
    5. Highest nuchal line
Ans. B.
  • Superior nuchal line correspond to transverse sinus
  • Asterion at junction of lambdoid occipito mastoid, parietomastoid suture marks the junction of transverse and sigmoid sinus172
  1. Burst lobe is often referred to as the combination of
    1. Acute SDH + extensive lobar contusion
    2. Extensive lobar contusion + intracerebral hemorrhage
    3. Acute SDH + extensive intracerebral hemorrhage
    4. Acute SDH + extensive lobar contusion + intracerebral hemorrhage
Ans. D.
  1. Following statements are true
    1. Extradural Hematoma crosses both midline and suture line
    2. Subdural Hematoma crosses both midline and suture line
    3. Extradural Hematoma crosses midline but does not cross suture line
    4. Subdural Hematoma crosses midline but does not crosses suture line
    5. Subdural Hematoma crosses suture line but does not cross midline
Ans. C and E.
  1. A patient is brought to the emergency who has suffered head injury. CT scan appear normal and no visceral injury. The patient dies soon. Which of the following statements are true regarding the case?
    1. On histopathology cluster of microglia (microglial star) can be found
    2. Axonal retraction ball may be seen
    3. Antibody against B-amyloid precusor protein can be demonstrated
    4. All of the above
Ans. D.
  • These all are indicative of diffuse axonal injury
  1. Factors predisposing to chronic subdural hematomas
    1. Low intracranial pressure
    2. Cerebral atrophy
    3. All coagulopathies
    4. Anticoagulant
    5. Alcoholics
    173
Ans. All of the above are true
  • Low ICP → overdrainage of CSF, VP shunt, dehydration, spinal anaesthesia
  • Alcholics → More prone to develop SDH ↑ rate of accident secondary coagulation disturbance vascular fragility as a result of avitaminosis
  1. About post-traumatic epilepsy
    1. Seizure presenting after 24hr is more common than within 24hr
    2. AED do not reduce the frequency of late PE but have effect on the early PE
    3. Fe (ferritin) and Hb are epileptogenic as they decrease the release of inhibitory neurotransmitter
    4. Cerebral contusion, peneterating SDH injury depressed skull fracture have more incidence of late PTE
    5. Hereditary factor probably does not play any role in PTE
Ans. B to E.
  • In adults in case of moderate, and severe head injury there is relation between early and late PTE
  • After trauma most seizures occur within <24hr
  1. Incidence of CSF rhinorrhea following transphenoidal surgery is
    1. 3% to 6%
    2. 13% to 16%
    3. 23% to 26%
    4. 1% to 3%
Ans. A
  1. About CSF leak
    1. Post traumatic CSF rhinorrhea occur in about 2% of closed head injuries and in 9% patients with penetrating head injuries
    2. In majority of patients, the CSF leak occur soon after the injury or within 48hr of trauma
    3. In 50% of cases, the leak abolishes at the end of first week and almost all stop by 6 months
    4. The risk of meningitis is 3% to 11% in patients who have post traumatic CSF rhinorrhea
    5. In non-traumatic CSF rhinorrhea, conservative treatment has no role to play174
Ans. All of the above are true
  • Prophylactic antibiotics have not been shown to be effective in the prevention of meningitis and are no longer given for closed post-traumatic leaks
  • The roof of ethmoid, the cribiform plate are the most frequent site of CSF rhinorrhea
  1. Indication for early treatment of carotico-cavernous fistula are
    1. Progressive visual failure
    2. Early filling of cortical veins
    3. Extension into air sinus
    4. Disfiguring progressive proptosis
    5. Subarachnoid hemorrhage
Ans. All of the above are true
  • Also intracerebral bleed
  • ICA ligations today have almost no role in CCF (traumatic) management and traping of ICA leads to opening of collateral and recurrence of Carotid Cavernous Fistula
  • Endovascular treatment is the safest mode of treating these cases
  1. Which statements about optic nerve are correct?
    1. It contains about 1 million fibres
    2. The intracanalicular portion is relatively avascular and fixed to periosteum and supplied by centripetal branches of pia and is 4–10 mm in length
    3. During head injury, the most frequent field defect is temporal field defect
    4. The introocular portion is 30mm in length
Ans. A to C.
  • Optic nerve is around 5 cm in length. 4 parts
  • Intraocular → 1–2 mm
  • Intraorbital → 25–30 mm [longest]
  • Intracanalicular → 4–10 mm
  • Intracranial → 10–15 mm
  • Develop from optic stalk
Note: Temporal field defect is due to relative involvement of nasal fibres as the bone on the medial side of the orbit is thin and is likely to fracture frequently175
  1. Which of the part of the optic nerve is the longest?
    1. Intraocular
    2. Intraorbital
    3. Intracanalicular
    4. Intracranial
Ans. B.
  1. Secondary brainstem hemorrhage is more frequent in
    1. Tectum of midbrain and pons
    2. Tegmentum of midbrain and tectum of pons
    3. Tectum of midbrain and tegmentum of pons
    4. Tegmentum of midbrain and pons
Ans. D.
  • Hemorrhage is bilateral and paramedian in position. Primary lesions are less common in tegmentum
  1. Which of the following modalities is being used recently in patients of vegetative state due to head injury?
    1. Periaqueductal gray matter stimulation
    2. Periventricular gray matter of stimulation
    3. Anterior column stimulation
    4. Dorsal column stimulation
Ans. D.
  • Treatment of persistent vegetative state is an unsolved problem. Recently role of dorsal column stimulation (DCS) has been shown to have some role of the management of intractable pain or spasticity and in vegetative states
  1. Which of the following segments of the optic nerve is most common one damaged with closed head injury?
    1. Intraocular
    2. Intraorbital
    3. Intracanalicular
    4. Intracranial
Ans. C.176
  1. How many percent of patients with closed head injury and having normal CT scan will have intracranial hypertension?
    1. 13%
    2. 3%
    3. 23%
    4. 33%
Ans. A.
  • 60% of patients with closed head injury and an abnormal CT will have ICHTN
  • Patients with a normal CT and 2 or more risk factors will have 60% risk of Ic-HTN
  • Risk factor for Intracranial-Hypertension with a normal CT
  • Age > 40 year
  • SBP < 90mmHg
  • Decerebrate or Decorticate posturing on motor examination (unilateral or bilateral)
  • If only 1 or none are present, ICP will be increased in only 4%
  1. Neurofibrillary tangles are found in
    1. Progressive supranuclear palsy
    2. Post encephalitic parkinsonism
    3. ALS – parkinsonism/dementis complex of guam
    4. Chronic traumatic encephalopathy
    5. Idiopathic parkinsonism
Ans. A to D.
  • In idiopathic parkinsonism, lewy bodies (intracytoplasmic eosinophitic bodies) are found. They are however characteristic of Alzhiemer's disease
  1. Histologically, which of the following is almost invariable accompaniment of Alzheimer's disease?
    1. Neuritic plaque
    2. Neurofibrillary tangles
    3. Amyloid angiopathy
    4. Hiranobody [granulovascular degeneration]
    5. Lewy body
Ans. C.
  • Amyloid angiopathy gives rise to lobar hemorrhage177
  1. Among which of the following incomplete spinal cord injury has the best prognosis?
    1. Anterior cord syndrome
    2. Posterior cord syndrome
    3. Central cord syndrome
    4. Brown-Sequard syndrome
Ans. D
  • B-S syndrome has the best prognosis of any of the incomplete spinal cord injuries = 90% of patients with this condition will regain the ability to ambulate independently as well as anal and urinary spincter control
  • In central cord syndrome (cord contusion without hematomyelia), = 50% will recover enough lower extremity strength and sensation to ambulate independently although typically with significant spasticity
  1. About ligaments of the occipito atlanto-axial complex
    1. Posterior atlanto-occipital membrane is a continuation of posterior longitudinal ligament
    2. Apical ligament is formed by proatlas or fourth occipital scleretome
    3. Tectorial ligament is an extension of anterior longitudinal ligament and along with alar ligament help in occipito atlanto stability
    4. Transverse ligament is the key ligament in maintaining altanto-axial stability
    5. In atlanto-occipital dislocation traction is usually avoided and posterior occipito cervical fusion recommended
Ans. B, D and E.
  • Tectorial ligament is an upward extension of the posterior longitudinal ligament. The most important structures in maintaining atlanto-occipital stability are the tectorial membrane and the alar ligaments
  1. How much will be the sum total overhang of both C1 lateral masses on C2 that will indicate that transverse ligament is probably disrupted?
    1. ≥ 4mm
    2. ≥ 5mm
    3. ≥ 6mm
    4. ≥ 7mm
178
Ans. D.
  • >7mm, it is also called Rule of Spence
  1. About Jefferson fracture
    1. Unstable but does not have neurological deficit
    2. Mechanism is hyperextension and distraction
    3. About 41% of cases are associated with C2 fracture, among which type II odontoid is most common (40%)
    4. Type II odontoid # ≥ 6mm displacement with Jefferson fracture is treated by open reduction internal fixation
    5. As such Jefferson is treated by collar or SOMI brace
Ans. A, C to E.
  • Mechanism is axial loading with neck in neutral position
  • Hangman fracture (Bilateral Pedicle # of C2 through pars interarticularis). two mechanisms are present in Judicial hanging—hyperextension and distraction. In vehicular accident there is hyperextension and axial loading
  1. Which of the following fractures are unstable?
    1. Jefferson fracture
    2. Type I and II hangman's #
    3. Type III hangman's #
    4. Type II odontoid fracture
    5. Type III odontoid fracture
Ans. A, C and D
  • Jefferson # → unstable
  • Type I Hangman's # → stable
  • Type III Hangman's # → unstable
  • Type I odontoid → unstable (controversial)
  • Type II odontoid → unstable
  • Type IIA odontoid → unstable
  • Type III odontoid → stable
  • Wedge compression lumbar # → stable
  • Burst lumber # → unstable
  • Seat belt # → unstable
  • Fracture dislocation → unstable
  • True tear drop # → unstable (often require stabilization)179
  1. Cerebrovascular resistance is affected by PaCO2 such that there is a linear increase in CBF with increasing PaCO2 within the range of
    1. 40 – 70mmHg
    2. 30 – 80mmHg
    3. 20 – 80mmHg
    4. 20 – 60mmHg
Ans. C
  1. The coupling ratio (CBF / CMRO2) in normal quiescent brain is
    1. 10 – 14
    2. 14 – 18
    3. 24 – 28
    4. 34 – 38
Ans. B.
  • With focal cortial activity, local CBF increases = 30% while CMRO2 increases 5%
  • CMRO2 = 3.0 – 3.8ml/100gm tissue/min
  1. Lacunar strokes is diagnosed by the presence of the following
    1. Aphasia
    2. Sensorimotor CVA
    3. Monoplegia
    4. Homonymous hemionopia
    5. Pure sensory CVA
Ans. E.
  • Lacunar strokes diagnosis virtually excluded by Aphasia, apractagnosia, sensorimotor CVA, monoplegia homonymous hemianopia, severe impairment, stupor, coma, LOC or seizures
  • Pure sensory CVA (most common manifestation)
  • Pure motor hemiparesis (2nd most common)
  • Typical location for lacunar strokes in descending frequency is Putamen > caudate > thalamus > Pons > IC > convolutional white matter
  1. Anterior choroidal artery syndrome result in
    1. Hemihyperpathia
    2. Hemihyperalgesia
    3. Hemihypesthesia
    4. Homonymous hemianopia
    5. Contralateral hemiplegia
Ans. D and E.180
  1. Refractory cases of cluster headache can be treated by radiofrequency ablation of
    1. Nodose ganglion
    2. Pterosal ganglion
    3. Otic ganglion
    4. Spiral ganglion
    5. Sphenopalatine ganglion
Ans. E.
  1. An elderly patient present with sudden headache and unilateral pupilary dilatation, the differential diagnosis will be
    1. Superior cerebellar artery aneurysm
    2. Posterior communicating artery aneurysm
    3. Posterior cerebral artery aneurysm
    4. Putaminal hemorrhage
    5. Cerebellar hemorrhage
Ans. A to D. E is false
  1. Bitemporal hemianopia can be present in the following situation
    1. Pituitary adenoma
    2. Craniopharyngioma
    3. Superior hypophyseal artery aneurysm
    4. Basilar bifurcation aneurysm or Basilar tip aneurysm
    5. Ophthalmic artery aneurysm
Ans. A to D.
  • Pituitary adenoma causes bitemporal hemianopia, compression comes from below
  • Craniopharyngioma causes bitemporal hemianopia, compression comes from above
  • Superior hypophyseal artery aneurysm only suprasellar variant not paraclinoid, compress pituitary stalk and cause hypopituitarism and classic chiasmal visual symptoms (bilateral temporal hemianopsia)
  • Enlargement of the basilar artery aneurysm may rarely compress the optic chiasm leading to bitemporal field cut (mimicking pituitary tumor or occasionally may compress the third nerve as it exits from the interpeduncular forsa causing occulomotor nerve palsy)181
  1. Conditions causing unilateral compressive occulomotor nerve palsy
    1. Post-comm artery aneurysm
    2. Basilar tip aneurysm or Basilar bifurcation aneurysm
    3. Superior cerebellar artery aneursym
    4. Posterior cerebral artery aneurysm
    5. Transtentorial herniation or uncinate herniation
Ans. A to E
  1. Post traumatic carotid dissection is due to which mechanism of injury
    1. Hyperflexion
    2. Hyperextension
    3. Hyperflexion with lateral rotation
    4. Hyperextension with lateral rotation
Ans. D.
  • Hyperextension with lateral rotation
  1. Following statement regarding CT in SAH
    1. CT performed 72 hours after SAH is more sensitive than CT within 24 hours
    2. Modern CT is able to detect SAH in a least 90% of patients within 24 hours
    3. Modern CT is sufficiently sensitive than LP, it is seldom necessary if the CT is normal
    4. If CT suggest SAH, LP is done for confirmation
    5. All of the above are true
Ans. B.
  • Modern CT is able to detect 90–95% of subarachnoid hemorrhages but this sensitivity declines by 72 hours of CT. If CT is positive, there is little diagnostic utility of LP, so it usually does not need to be performed. However, if CT is negative in patient with appropriate clinical history, LP should be performed to look for blood
  1. The content of interpeduncular cistern are all EXCEPT
    1. IIIrd nerve
    2. VIth nerve
    3. P2 of PCA
    4. P1 of PCA
    182
Ans. C.
  • P2 of PCA is present in ambient cistern. Interpeduncular cistern also consist of basal vein of Rosenthal and basilar artery tip. The cerebellomedullary cistern which is continued posteriorly as the cisterna magna contain IX, X, XI, XII, PICA and vertebral artery. Prepontine cistern contain basilar artery. The cerebellopentine cistern contains VII, VIII and branches of AICA. Crural cistern contain posterior choroidal artery
  1. Predisposition to intracranial aneurysm occur with which of the following conditions?
    1. Autosomal dominant polycystic kidney disease [ADPKD]
    2. Ehlers-Danlos Type IV (deficient collagen type III)
    3. First degree relative with aneurysmal SAH
    4. Neurofibromatosis type I
    5. Coarctation of aorta
Ans. All of the above are true
  • Also AVM including Moya Moya disease
  • Marfan's syndrome pseudoxanthoma elasticum
  • Osler–Weber-Rendu syndrome
  • Atherosclerosis
  • Bacterial endocarditis (3–20%)
  • Cigarette smoking
  1. Statements regarding meningioma
    1. Acoustic schwannoma and meningiomas both are associated with abnormalities of chromosome 22
    2. Posterior fossa is the most common location for meningiomas in adults
    3. Multiple meningiomas suggest neurofibromatosis type 2
    4. Progesterone receptor is found in 50% and estrogen receptor in about 20% of meningiomas
    5. Intraventricular position is quite common in children than in adults
Ans. A, C to E. B is false
  • Most common location is parasaggital or Falcine (24%) and cerebral convexity (18%)
  1. In which age group infratentorial pediatric tumor is commonest?
    1. 0–6 month
    2. 6–12 month
    3. 12–24 month
    4. 2–16 year
183
Ans. C.
  • 0–6 month 27%
  • 6–12 month 53%
  • 12–24 month 74%
  • 2–16 year 42%
  1. About tumors following statements are
    1. Nitrosoureas have good blood brain barrier peneterance which make them attractive as chemotherapy for brain tumors
    2. Cyclical vomiting and occipital headache suggest a cerebellar tumor
    3. Most primary brain tumors are benign
    4. Cerebral PNET's are usually localized and require local radiation after surgery
    5. Surgical resection of vestibular schwamomas result in facial paralysis and hearing loss in virtually all patients
Ans. A and B.
  • Nitrosoureas such as BCNU and CCNU have better penetrance through the blood brain barrier than most other agents, which makes them used widely for patients with malignant brain tumors (These are lipophilic)
  • Cerebellar tumors commonly produce vomiting and occipital headache, although these symptoms are not specific, the majority of primary brain tumors are malignant
  • Cerebral PNET, have a propensity to metastasize within the CSF, so total neuraxis radiation is indicated following surgery with current state of the art techniques, facial nerve is preserved in 36% of patients and cochlear function is preserved in 26% of patients
  1. Match the following selection of antibiotics to be used in meningitis with following organism
    1. Gram +ve cocci → Cephalosporin + Vancomycin
    2. Gram +ve bacilli → Ampicillin + Aminoglycoside
    3. Gram –ve bacilli → Cephalosporin + Aminoglycoside
    4. Gram –ve cocci → Penicillin G
Ans. All of the above are true
  • Drug of choice
  • S. Pnumoniae [PCNG] 2nd choice chloramphenicol
  • N. Meningitides : [PCNG] 2nd choice chloramphenicol184
  • H. Influenzae (Gram +ve bacilli) (A) Non penicillinase producing → ampicillin (B) Penicillnase producing → chloramphenecol
  • Group B streptococus → ampicillin
  • L Monocytogens → ampicillin
  • S-aureus (A) If MRSA, coagulase negative S-aureus vancomycin + Po Rifampicin + Po Trimethoprim (B) Not MRSA Infant <7d – Methicillin; all others – Nafcillin; PCN allergy – vancomycin or (cefazolin both IV)
  • Gram negative rods (A) Ceftriaxone, Cefotaxime,
  • P. aeruginosa → ceftozidime
  1. Match the following
    1. Post herpetic neuralgia→ Varicella Zoster virus
    2. Neonatal meningitis→ HSV-II
    3. Hearing loss beginning in childhood→ CMV (cytomegalo virus)
    4. Subacute sclerosing panencphalitis→ Measles [Paramyxo virus]
    5. PML (Progressive multifocal leukoencephelopathy) → JC virus SV – 40 virus
Ans. All of the above are true
  • Ganciclovir is used for CMV
  1. Regarding infections following statement
    1. Toxoplasma gondii is carried in birds and cats
    2. Cysticercosis is caused by pork tapeworm
    3. Echinococcus is usually acquired in the household by close contact with dogs
    4. Neurologic complication of trichinosis are usually due to direct muscle innervation
Ans. All of the above are true
  • T. gondii is carried by birds which has given the organism a world wide distribution. In the household cat, harbor the organism in the gut
  • Patient with trichinosis typically present with fever, periorbital edema and myalgia and may progress to muscle invasive phase with pain swelling and weakness which begin proximally and in extra ocular muscle and become more extensive185
  1. Following statement regarding EEG are
    1. Either 12 hour of observation without a confirmatory test or 6 hours with an isocletric EEG are sufficient to establish brain death when the cause is known and it is not anoxia
    2. There are currently no established criteria for determination of brain death in children younger than 7 days
    3. Either 24 hour of observation or 12 hour with a confirmatory test are required for determination of brain death when the cause is anoxia
    4. Burst-suppression pattern, periodic triphasic waves, alpha coma suggest a poor prognosis for good neurologic recovery assuming the absence of a sedative
Ans. All of the above are true
  1. Following statements regarding brain oedema are
    1. Vasogenic oedema is pericellular edema and is caused by disruption of BBB
    2. Vasogenic oedema due to tumor respond mainly to corticosteroids
    3. Corticosteroids are usually not indicated in head trauma where there is mainly cytotoxic edema
    4. Interstitial edema is transependymal edema seen in hydrocephalus
    5. In post anoxic edema which is due to brain necrosis only hyperventilation can reduce intracranial vascular volume, temporarily reducing ICP and osmotic agents are of no value because they will leak out of the vessel into damagd brain and have little effect
Ans. All of the above are true
  1. Regarding Ca2+ disorder
    1. Presence of Ca2+ on nerve membrane may play a significant role in keeping Na+ gate closed
    2. A decrease in extracellular concentration of Ca2+ causes decreased membrane stability
    3. Hyperparathyroidism is the most common cause of hyper calcemia
    4. Papilledema can develop as a complication of hypocalcemia with increased intracranial pressure
    5. Hypercalcemia produces confusion and weakness but hypocalcemia can produce seizure and ↑ ICP also extrapyramidal signs which have a parkinsonian appearance
    186
Ans. All of the above are true
  • ↓ Extracellular concentration of Ca2+ caused Na+ gates to open and membrane protential decrease (depolarization occur)
  1. TRUE Statement about the blinking of the eye is
    1. Blinking occurs irregularly at a rate of 12–20 times a minute
    2. Increased blink frequency ooccurs with sensitization of trigeminal nerve ending
    3. Reduced frequency of blinking is less than 10 per minute is characteristic of progressive nuclear palsy
    4. Myerson sign is absent in parkinsonism
Ans. D
  • In progressive supranuclear palsy and Parkinson's disease, adaptation to repeated supraorbital tapping at a rate of about 1 per second is impaired, therefore the patient continue to blink with each tap on the forehead or glabella. The failure to inhibit this response is reffered to as glabellar or Myerson sign
  1. Following statements regarding Edema and CSF circulation are
    1. Tamoxifen may produce intracranial hypertension
    2. Patient with Diabetic Keto Acidosis are at greatest risk for cerebral edema during the phase of glucose increase
    3. Cerebral edema is a greater problem for acute liver failure than chronic liver failure
    4. Acetazolamide reduces CSF production
    5. Tight junctions are characteristic of brain capillaries but not peripheral vessel
Ans. A, C to E
  1. Statements regarding Myelination of brain pathways
    1. The corpus callosum does not complete myelinate until mid-adolscence
    2. Most cranial nerves are myelinated before the third trimester
    3. Myelination commence in the fourth foetal month in the spinal cord motor roots caudal to rostral direction
    4. Axon which will myelinate do not function electrically until the myelination is complete
    5. Hypothyroidism causes slowed myelination
    187
Ans. A to C and E
  • Myelination of corticospinal tract continue upto end of the second postnatal period
  • Myelination of association neocortex extend into the third decade
  1. Match the following congenital anomalies with their time of occurrence in utero
    1. Lissencephaly→ 8 – 20 weeks
    2. Anencephaly → 1 – 4 weeks
    3. Myelomeningocele → 1 – 4 weeks
    4. Agenesis of the corpus callosum → 4 – 8 weeks
Ans. All of the above are true
  • Lissencephaly is due to failure of neuronal migration from the germinal matrix to cortex during 8 – 20 weeks
  • Anencephaly is due to failure of closure of the anterior neuropore during 24th day of gestation
  • Meningomyelocele is due to failure of closure of the posterior neuropore during 26th day of gestation
  1. About Mucopolysaccharidosis statements are
    1. Hurler's syndrome (MPSIH) is caused by deficiency of alpha-1-idurodinase
    2. Morquio's syndrome (MPS IV) is characterized by severe skeletal deformities and ligamentous laxity
    3. Heparan sulfate only is excreted in urine in Sanfilippo's syndrome
    4. Hunter's syndrome is characterized by iduronate sulfatase, pebbling of the skin may occur, X linked recessive
    5. Morquio's and Scheies syndrome is characterized by normal intelligence
Ans. All of the above are true
  • Heparan-N-Sulfatase is deficient in Sanfilippo's syndrome
  • L-Iduronidase deficient in Hurler's syndrome
  • Iduronate –2-Sulfatase deficient in Hunter syndrome
  1. Following statements regarding Dementia
    1. Progressive supranuclear palsy cause subcortical dementia
    2. Vascular dementia can be both cortical and subcortical dementia
    3. Alzheimer's disease causes cortical dementia188
    4. Apraxia is a common feature of cortical dementia
    5. Psychomotor slowing is a feature of subcortical dementia
Ans. All of the above are true
  1. Statement regarding post traumatic epilepsy
    1. Post-traumatic epilepsy is more common with depressed skull # than closed head injuries
    2. Prophylactic phenytoin decreases the frequency of late onset post-traumatic seizures
    3. Subdural hematoma predisposes to development of post-traumatic seizures
    4. Children are less likely to develop early post-traumatic epilepsy than adult
    5. In children, hereditary history of seizure has no role in early traumatic seizure but has role in late traumatic seizure
Ans. A, C and E.
  • Prophylactic phenytoin decreases the chance of early post-traumatic seizure but not late seizures, so long term prophylaxis is seldom indicated, children are more likely than adults to have early post-traumatic seizures
  1. Following statements regarding Herpes-zoster are
    1. Acyclovir prevents postherpetic neuralagia if given at the onset of the eruption
    2. Corticosteroids may reduce the frequency of post herpetic neuralgia
    3. Zoster of the geniculate ganglion has a high risk of stroke from carotid involvement
    4. Post herpetic neuralgia develops in about 10% of patients with herpes zoster opthalmicus
    5. Incidence of thoracic herpetic neuralgia is greater than ophthalmic post herpetic neuralgia
Ans. B and D.
  • Acyclovir does not prevent the development of PHN but it reduces acute pain and speeds healing of the lesions. Zoster of the geniculate ganglion does not increase the risk for stroke from carotid involvement. Incidence of ophthalmic PHN is greater than thoracic herpetic neuralgia189
  1. Hemifacial spasm is a focal movement disorder which may be difficult to treat. Following statements regarding HFS
    1. May be treated by botulinum toxin injection
    2. May be treated by microvascular decompression
    3. May follow Bell's palsy
    4. There may be bilateral involvement
    5. Hemifacial spasm and Palatal myoclonus are the only involuntary movements present during sleep
Ans. All of the above are true
  1. Regarding Wilson's disease
    1. Elevated serum ceruloplasmin
    2. Copper deposition in the cornea
    3. Seizures are common
    4. Fine action tremor is common
    5. It is autosomal recessive
Ans. B, D and E.
  • Decreased serum ceruloplasmin is present copper is deposited in the descemet's membrane. Seizures are uncommon. Gene on chromosome 13
  1. Statements regarding drug induced Parkinsonism
    1. 15% of patients on neuroleptics develop parkinsonian features
    2. Careful examination can distinguish drug induced from idiopathic parkinsonism in most patients
    3. Tremor is less prominent in drug induced parkinsonism than in idiopathic parkinsonism
    4. Parkinsonism typically perists for months or years following discontinuation of the neuroleptic
    5. Clozapine an atypical neuroleptic do not produce parkinsonism
Ans. A, C and E.
  1. Myelomeningocele may be associated with which of the following malformation
    1. Polymicrogyria
    2. Hydrocephalus
    3. Chilari Malformation
    4. Syringomyelia
    5. All may be associated
190
Ans. E
  • About 100% of patients with myelomeningocele has MRI evidence of Arnold chiari malformation type II
  1. Which of the following clinical features would not be expected in patients with Amyotrophic lateral sclerosis?
    1. Head droop
    2. Muscle fasciculation
    3. Muscle cramps
    4. Unsteady gait
    5. Sphincter dysfunction
Ans. E.
  • Head droop can also occur with polymyositis or myasthenia gravis
  1. Which is not expected EMG finding in patients with Amyotrophic Lateral Sclerosis?
    1. Active dennervation
    2. Chronic reinnervation
    3. Fasciculations
    4. Early recruitment of motor units
Ans. D.
  • Reduced number of motor units is expected in patient with ALS. Early recruitment would be expected in myopathic disorders. Fasciculations are not specific for ALS, but are usually identified in patients with ALS. Active denervation and polyphasic motor units indicative of chronic reinnervation are typical
  1. Statements pertaining to the type of neuromuscular condition and relevant clinical finding
    1. Mild weakness with total areflexia is seen in demyelinating neuropathy such as Guillain-Barre's syndrome
    2. Proximal weakness with preservation of distal muscle strength seen in myopathy
    3. Dysproteinemias have produced both axonal and demyelinating neuropathies
    4. Most toxins produce an axonal neuropathy
Ans. All of the above are true191
  1. Match the following
    1. Abnormal unpleasant sensation in response to an ordinary non-noxious sensory stimulus → Dysesthesia
    2. Abnormal unpleasant spontaneous sensation with a tingling or pricking character → Paresthesias
    3. Painful sensation in response to a non-noxious stimulus → Allodynia
    4. Exaggerated pain from a noxious stimulus → Hyperalgesia
    5. Exaggerated pain from a non-noxious stimulus (emotional continence) → Hyperpathia
Ans. All of the above are true
  1. About Entrapment neuropathy
    1. Median nerve compression at the wrist is the most common entrapment neuropathy
    2. Sensory latency is the most sensitive electrodiagnostic test for carpal turnnel syndrome
    3. Pregnancy, AVshunt for hemodialysis obesity all have shown to predispose to carpal tunnel syndrome
    4. Extensor carpi radialis longus and brevis, supinator, and extensor carpt ulnaris will be denervated in a patient with lesion of the posterior interosseus nerve
    5. Common peroneal nerve (Fibular) is the most common entrapment neuropathy in the leg
Ans. A to C and E.
  • Extensor carpi radialis longus and brevis, supinator are supplied by radial nerve. Extensor carpi ulnaris is supplied by posterior interosseus nerve (a pure motor nerve)
  1. About Hereditary Motor Sensory neuropathy
    1. HMSN-3 is Dejerine – Sottas disease
    2. HMSN 4 is usually autosomal recessive
    3. HMSN-2 is dominantly inherited axonal neuropathy
    4. HMSN-3 has latest onset after age 30
    5. Nerve conduction velocity is normal in HMSN-3
Ans. A to C.192
  1. Which of the following interventions for Gullerin–Barre syndrome are felt to have documented beneficial effects?
    1. Corticosteroids
    2. Intravenous immunoglobulin
    3. Azathioprine
    4. Plasma exchange
    5. Cyclophos phanide
Ans. D.
  • Corticosteroid, Azathioprine and cyclophosphamide have no beneficial effect
  1. Following statements regarding multifocal motor neuropathy and ALS are
    1. ALS has weakness most prominent in the arms while MMN has weakness most prominent distally in the legs
    2. Muscle cramps and fasciculation are common in ALS but uncommon in MMN
    3. Weakness may be profound in muscles of normal bulk in ALS while patients with MMN have weakness proportional to atrophy
    4. Depressed tendon reflexes are common in MMN while hyper reflexia is expected in ALS
    5. Uppor motor neuron finding in lower limb and lower motor neuron finding in upper limb is seen in ALS
Ans. D and E.
  • Both ALS and MMN have weakness which is most prominent in the arms. Muscle cramps and fasciculation are prominent in both conditions. MMN quite typically has muscles which may be profoundly weak despite normal bulk. However, differences in reflexes can be a helpful differentiating tool
  1. Features characteristic of critical illness polyneuropathy
    1. Flaccid weakness and hyporeflexia
    2. Setting of sepsis and multi organ failure
    3. Gradual recovery over 3–6 months
    4. Prominent distal pain and paraesthesias
Ans. A to C.
  • Pain and other sensory symptoms are uncommon in patients with critical illness polyneuropathy193
  1. Sweating changes are common in patients with traumatic myelopathy, which of the following indicates the most common sweating disorder with myelopathy
    1. Hyperhydrosis above the lesion and anhydrosis below the lesion
    2. Anhydrosis above the lesion and hyperhydrosis below the lesion
    3. Normal sweating above the lesion and anhydrosis below the lesion
    4. Normal sweating above the lesion and hyperhydrosis below the lesion
Ans. A.
  • Hyperhydrosis is above the lesion and anhydrosis below the lesion
  1. Following statements regarding the sympathetic skin response are
    1. The sympathetic skin response is usually absent in axonal neuropathies
    2. The SSR is usually preserved in demyelinating neuropathies
    3. The SSR is usually absent in autonomic neuropathies
    4. Prolonged SSR latency or reduced amplitude indicates incipient autonomic failure regardless of cause
Ans. A to C.
  1. Which of the following findings is least likely in patients with generalized myasthenia gravis?
    1. Normal Combined Motor Action Potential amplitude
    2. Decremental response to repetitive stimulation more prominent in proximal muscle than distal muscles
    3. Increased fitter on single fiber EMG
    4. Fibrillation potentiale and positive sharp waves in affected muscle
Ans. D.
  • Fibrillation potentials and positive sharp waves are a hallmark of muscle fibre excitability and can be seen in myopathies and neuropathies194
  1. Following statements regarding Myasthenia gravis
    1. Almost all patients with thymoma and myasthenia have elevated AchR-binding antibodies
    2. The best response to thymectomy is in young patients early in their disease
    3. Anti cholinesterase and Azathioprine not helpful for myasthenic crisis but cortiocosteroid, plasma exchange and IVIG are helpful
    4. Anaphylactic reaction in patients with IgA deficiency, Transmission of hepatitis, vascular headache is a legitimate concern with administration of IVIG for myasthenia gravis
    5. Cholinesterase inhibitors are the treatment of choice for initial treatment of patients with pure ocular myasthenia
Ans. All of the above are true
  • Patient with Myasthenia Gravis without increased antibody may still respond to plasma exchange
  • Approx 55% of patients with ocular myasthenia have binding antibodies and approx 80% of patients with generalized myasthenia have binding antibodies
  • Thymectomy usually produces maximal improvement 2–5 years after surgery
  • Repeat theymectomy can be performed
  • Transthoracic thymectomy preferred against endoscopic thymectomy
  • Thymectomy may be helpful not only for seropositive patients but also seronegative patients
  • Vascular headache may be sufficient to limit the use of IVIG
  1. Following statements concering muscle biopsy and diagnosis of neuromuscular condition
    1. Dennervation characteristically produces fiber type grouping, best seen on oxidative stains
    2. Increased central nuclei are typically thought of as being caused by myopathy although some increase may be seen in neuropathic disorders
    3. Perifascicular atrophy is typically seen in patients with dermatomyositis a myopathy but is not typically seen with other myopathies
    4. Atrophy of an entire fascicle is typically seen in chronic progressive dennervation
    5. Necrosis and phagocytosis of individual muscle fibres is the principle change in polymyositis with large number of T-cells found in the intramuscular inflammatory exudates
    195
Ans. All of the above are true
  • The grouping is due to reinnervation of denervated muscle fibers by adjacent motor nerves
  1. The most common agent implicated in Bell's Palsy is
    1. HSV-1
    2. HSV-2
    3. VZV
    4. EBV
Ans. A.
  • HSV-1 is the most common agent implicated in Bell's Palsy with VZV a distant second
  1. Following statements regarding traumatic cauda equina lesions are
    1. Incomplete lesions are more likely to spare sensory function than spare motor function
    2. Prognosis for motor recovery is poorer than with central cord syndrome
    3. Most injuries causing cauda equina damage are as or below the L1 – L2 level
    4. Bowel and bladder incontinence occurs only with complete or near – complete lesions
    5. Knee and ankle jerk will be lost as compared to conus syndrome in which knee and ankle jerk are preserved
Ans. A, C and E.
  • Prognosis for motor recovery is generally better for patients with cauda equina lesions than central cord lesion because the LMN have greater resilience and regenerative capacity than the upper motor neuron
  • Sphincter disturbance is common with cauda equina lesion even in patients with preservation of leg motor function
  • Epiconus [L4, L5, S1, S2] segment conus → S3, S4, S5, Co segment cauda equino → L2 – Coccygeal roots
  1. Which of the following pathology does not involve disc space?
    1. Potts spine
    2. Metastasis
    3. Pyogenic infection
    4. All of the above
196
Ans. B
  • Of the many lytic or destructive lesions that involve the vertebrae, destruction of the disc space is highly suggestive of infection which often involves at least two adjacent vertebral level. Although tumors may involve adjacent vertebral levels and cause collapse of disc height, the disc space is usually not destroyed, possible exceptions include some vertebral plasmacytomas. Unlike pyogenic infection, the disc may be relatively resistant to tuberculous involvement in Potts disease
  1. Following statements regarding lumbar spine disorders are
    1. The prognosis of cauda equina compression is better than that of spinal cord compression
    2. Urinary incontinence in the absence of other finding is an indication for immediate surgical evacuation
    3. The most common cause of cauda equina compression is extrusion of the L4 – L5 disc
    4. MRI is the first choice for evaluation of suspected cauda equina syndrome
Ans. All of the above are true
  1. Statements regarding Cerebellar hemorrhage are
    1. Cerebellar hemorrhage can be treated by ventriculostomy of the lateral ventricle to treat hydrocephalus
    2. The prognosis for good neurologic function for patients with cerebellar hemorrhage who require posterior fossa decompression is poor
    3. A patient with cerebellar alaxia who develops stupor with a gaze palsy and a facial palsy usually needs decompression
    4. A patient with cerebellar ataxia who has horner's syndrome ipsilateral facial sensory loss is likely to require cerebellar decompression
    5. If GCS ≥ 14 and hemorrhage is less than 4cm decompression is usually not done
Ans. C and E.
  • Ventriculostomy with decompression will lead to an upward herniation
  • In option 4, it is likely to be PICA syndrome. It has to be differentiated. Because in the setting of lateral medullary syndrome no role for decompression197
  1. Which of the following is least likely to be helpful for management of vasospasm in patients with SAH?
    1. Plasma expansion with colloid
    2. Lowering of blood pressure with labetolol
    3. Nimodipine
    4. Angioplasty
Ans. B.
  • Insetting of vasospasm triple (HHH) therapy is instituted i.e. (i) Hypertension (ii) Haemodilution (iii) Hypervolemia
  • So lowering of blood pressure with labetelol will lead to further compromise of blood flow [CPP = MAP – ICP] giving labelelol will decrease MAP, ICP is already decreased leading to ↑ CPP
  1. Following statements regarding extracranial carotid dissection
    1. Angioplasty with stent placement may be performed
    2. TIA may develop from extracranial carotid dissection
    3. Horner's syndrome is common
    4. Anti-cogulation is contraindicated
Ans. A to C.
  • Extracranial carotid dissection usually respond to anti-coagulation and surgery is rarely necessary
  1. Following statements regarding air way pressure
    1. PEEP should be kept below 15cm H2O to avoid impending venous return and producing hypotension
    2. Sustained high PEEP may cause tension pneumothorax
    3. High respiratory rate in a patient with COPD may precipitate high airway pressure and subsequent hypotension
    4. PEEP is reserved for patients who have impaired oxygenation despite high FIO2
    5. In PEEP the physiological dead space is increased
Ans. A to C and E.
  • PEEP is used routinely by most CCU and need not reserved for those with ↑FIO2 level
  • A level of 5cm H2O is used
  • ↑ Airway pressure in a patient with COPD is termed auto PEEP198
  1. Which of the following procedures should not be used as supportive evidence of brain death?
    1. EEG
    2. BAEP
    3. VEP
    4. Ceretral blood flow
    5. Apnoea test
Ans. C.
  • VEP cannot be used because there is no certainty that the signal has been transduced in the eye and relayed to the brain
  1. Which of the following antiepileptic drug act on glutamate receptor?
    1. Carbamazepine
    2. Felbamate
    3. Topiramate
    4. Phenobarbital
    5. Valproic acid
Ans. A to D.
  1. Following statements are regarding acetylcholine receptor
    1. Sympathetic ganglion receptors are only nicotinic
    2. Motor endplate receptors are muscarinic
    3. Patients with myasthenia have increased receptor turnover, causing fewer receptor to be available for binding with acetylcholine
    4. Botulinum toxin blocks binding of acetylcholine to the post synaptic receptors
    5. Nicotinic receptor on muscle has 4 type of subunits and on ganglia two type of subunits
Ans. C and E.
  • Sympathetic ganglion receptors are nicotinic and also muscarinic (M2)
  • Motor endplate receptors are nicotinic Botulinumtoxin blocks the fusion of Ach Vesicle with the membrane. On ganglia there are two type of subunits with combination of any other199
  1. Statements regarding the pharmacology of neurotransmitter in the brain
    1. Dopamine is metabolized by MAO-A
    2. NE and serotonin is metabolized by MAO-B
    3. The action of dopamine is ended by enzymatic degradation at the receptor
    4. Dopamine is also metabolized by COMT (catechol methyl transferase)
    5. Reserpine depletes dopamine by preventing re-uptake into storage vesicles
Ans. D and E.
  • Dopamine is metablozied by MAO-B.
  • NE and serotonin is metabolized by MAO-A
  • Dopamine is subjected to re-uptake before degradation
  1. Statements regarding Acromegaly
    1. Most peripheral effect of GH is due to (somatomedin C or IGF-I)
    2. Carpal tunnel syndrome is common
    3. The patient may have a vertex headache
    4. Hypotension is common
    5. Proximal muscle weakness is common
Ans. A to C and E
  • Hypertension is common not hypotension
  1. Statements regarding Vestibular control of eye movements
    1. The vestibular system stabilizes gaze during head movement
    2. The vestibular system initiate eye movement in response to a novel visual stimulus
    3. Rotation of the head to the side activates the ampulla of the horizontal semicircular canal on the same side as the rotation
    4. Activation of the horizontal vestibular canal results in excitation of ipsilateral abducens nucleus and inhibition of the contralateral abducens nucleus
    5. Semicircular canal signals are insensitive to movement at a constant angular velocity
Ans. A, C and E.
  • The vestibular system is responsible for stabilization of visual fixation during head movements, but not for initiation in response to stimuli, which is the province of the frontal eye 200field and associated connections with other cortical areas. A horizontal semicircular canal is stimulated by warming of the ipsilateral par or rotation of the head to that side. Stimulation of one horizontal semicircular canal results in activation of the contralateral abducens and inhibition of the ipsilateral abducens nucleus such that eyes look toward the opposite side
  1. Following statements regarding visual perceptive deficit are
    1. True color perception deficit is only due to congenital retinal abnormality
    2. Ishihara plates are most sensitive for central (CNS) causes of color perceptive deficit
    3. Visual form agnosia prevent patient from navigating through complex environments
    4. Prosopagnosia is usually due to a right posterior cerebral lesion
    5. V8 area of the visual cortex is responsible for color vision
Ans. D and E.
  • True color perceptive defect can be due to central lesions as well as retinal lesions
  • The Ishihara plates are designed to identify predominantly color perceptive deficit related to retinal lesions but deficit with central processing can also result the abnormal interpretation of the Ishihara plates
  • Patients with visual form agnosia are often able to navigate through novel environment despite their deficit in interpretation of forms
  1. What proportion of left handed individuals have the right hemisphere dominant for language function?
    1. 90%
    2. 70%
    3. 30%
    4. 15%
    5. 10%
Ans. D.
  • In human population 91% are right handed. Among them 96% has left hemisphere dominant, and 4% right hemisphere dominant201
  • In left handed individuals 70% have left hemisphere dominant, 15% right hemisphere, and 15% no lateralization
  1. Statements regarding Cortical function
    1. Left hemisphere lesion may produce constructional apraxia
    2. Left hemisphere lesion often give more difficulty with writing to dictation than copying written material
    3. Ideomotor apraxia can be manifest on examination as difficulty mimicking the examiner's movements
    4. Left hemisphere neglect does not occur
    5. Lesion of left angular gyrus causes Gerstmann's syndrome
Ans. A to C and E.
  • Left hemisphere lesions are less likely to produce constructional apraxia than right hemisphere lesion, but this defect can occur, but has a different character than constructional apraxia due to a lesioin of the right hemisphere
  • Ideational or Ideosensory apraxia more involves the performance of a sequence of movement composing a previously learned task
  1. Regarding Neuroimaging
    1. Functional MRI has best spatial resolution
    2. SPECT has least sensitivity to patient co-operation and motion
    3. PET has best absolute quantification of neuronal function
    4. In PET patient in complex partial seizures, there is focal zone of hypometabolism in the interictal state which becomes hypermetabolic during seizures
    5. Volume flow rate is the best indication of the degree of stenosis in Doppler ultrasound
Ans. All of the above are true
  1. Statements regarding Neuroimaging
    1. MRA tend to over estimate the degree of carotid artery stenosis
    2. CT angiography is less susceptible than MRA to errors introduced by tortuosity of the vessels
    3. CT angiography has better localization of the level of the carotid bifurcation than MRA
    4. Spinal vessels cannot usually be visualized well on MRA
    5. CT aniography is relatively insensitive for ulcerated plaques
    202
Ans. All of the above are true
  • MRA cannot detect virtually all intracranial aneurysm in a co-operative patient
  • MRA and carotid Doppler sonography have comparable sensitivity and specificity
  • Flow gap is not due to technical issues
  • MRA has better visualization of tandem lesion than CT angiography
  • CT angiography is more sensitive for level of moderate to high grade stenosis than MRA
  • MRA can visualize venous thrombosis well.
  • MRA provide direction of flow which CT angiography does not provide
  1. Regarding Neuroimaging
    1. Fat suppression techniques on MRI are used for visualization of skull based and orbital lesions
    2. CSF appear bright on T-2 weighted images and dark on FLAIR images
    3. Medulloblastoma appear high density on CT and isointense on T2 wt. MRI
    4. MRI is less sensitive to subarachnoid blood than CT
    5. MRI with contrast is the best screening test for cerebral metastases of systemic neoplasm
Ans. All of the above are true
  • Fluid-attentuated inversion recovery (FLAIR) produces attenuation of CSF signal whereas white matter lesion remain bright
  • MS plaques appear hyperintense on both T-2 wt. image and FLAIR
  • Frontal and temporal atrophy are charactesistic of Pick's disease rather than Parkinson's disease
  • MRI does not show subarachnoid blood acutely because, methemoglobin has not yet formed in sufficient quantities to be visualized
  • MRI in acute herpes encephalitis shows increased intensity on T-2 wt. image in the anterior temporal and inferior frontal lobe
  • Nerve root avulsions may be detected better on post myclogrophic CT than on MRI203
  1. Demyelinating neuropathy will produce following neurophysiologic finding
    1. Increased F-wave latency
    2. Slowed nerve conduction velocities
    3. Increased distal motor latency
    4. Dispersal of the compound motor action potential
Ans. All of the above are true
  • Dispersal of the compound motor action potential waveform is because the change in conduction velocity is to a different extent in different axon
  1. Increased Jitter on single-fiber EMG can occur in all of the following EXCEPT
    1. Myasthenia Gravis
    2. Lambert-Eaton myasthenic syndrome
    3. Partial dennervation
    4. Muscular dystrophy
    5. All of the above
Ans. E.
  • While neuromuscular transmission defect such as myasthenia gravis and Lambert-Eaton myasthenic syndrome are the classically described cause of increased jitter
  1. Fasciculation potential is present in
    1. Amyotrophic Lateral Sclerosis
    2. Spinal Muscular Atrophy
    3. Acetylcholinesterase toxicity
    4. In the legs in cervical myelopathy
Ans. All of the above are true
  1. Myokymic discharges are common in
    1. Radiation plexopathy
    2. Brain stem tumor
    3. Multiple sclerosis
    4. Neoplastic infiltration of a plexus
Ans. A to C.
  • Neoplastic infiltration typically does not produce myokymic discharges, a fact that help to differentiate this condition from radiation plexopathy, in which myokymic discharges are common204
  1. Following statements regarding vascular imaging
    1. Diagnosis of suspected AVM is best performed by MRI with MRA
    2. Duplex USG has diagnostic sensitivity for carotid plaque and ulceration which is comparable to that of a conventional angiography
    3. Magnetic resonance angiography tends to over estimate the level of carotid stenosis
    4. Conventional angiography should be performed on all patients with transient cerebral ischemia in the carotid circulation
Ans. A to C.
  1. Elevated CSF protein content is present in
    1. Pseudotumor cerebri
    2. Guillain – Barre syndrome
    3. CIDP
    4. Spinal cord compression
    5. Tuberculous meningitis
Ans. B to E.
  • Pseudotumor cerebri increases ICP but CSF protein remain normal or even low but is NEVER RAISED
  1. Statements regarding lumbar radiculopathy are
    1. L3 radiculopathy can produce a depressed knee reflex
    2. Patients with fibromyalgia usually have spinal pains spanning multiple dermatomal distribution
    3. A knee reflex graded 3 is pathologic and indicates a cortiospinal lesion
    4. Weakness of the extensor hallucis longus may be the only clinical sign of L5 radiculopathy
    5. Isolated back pain without extremity pain, numbness, or weakness is seldom due to focal lumbar radiculopathy
Ans. A, B, D and E.
  • Knee reflex grade 3 is hyperactive but is not always pathological, it can be due to anxiety
  1. Neurogenic claudication due to lumber stenosis and peripheral arterial claudication concerning statements are
    1. Both produces pain which is prominent with walking but is alleviated by sitting205
    2. Both can produce distal leg pain
    3. Standing is most likely to alleviate pain of peripheral vascular claudication
    4. Both are related to reduced blood flow and change in metabolic pathway for energy production
    5. Anthropoid posture is characteristic for neurogenic claudication
Ans. All of the above are true
  1. Carpal tunnel syndrome will produce following clinical finding
    1. Electric sensation in digit 1 and 2 with percussion of the wrist crease over the median nerve
    2. Decreased sensation over the thenar eminence
    3. Increased distal latency of motor fibers to the abductor pollicisbrevis
    4. Increased distal latency of motor fibers to the opponent pollicis
    5. Denervation of the median innervated portion of the flexor digitorum profundus
Ans. A, D and E
  1. Which of the following nerve is not a branch of the sciatic nerve?
    1. Superficial peroneal
    2. Saphenous
    3. Sural
    4. Lateral cutaneous nerve of the call
    5. All are branches of the tibial
Ans. B.
  • Saphenous nerve is a branch of femoral nerve
  1. Match the following muscle with the root which is most responsible for its innervation
    1. Tibialis anterior/L4
    2. Extonsor hallucis longus/L5
    3. Deltoid/C5
    4. Gastrocnemius/S1
    5. Brachiordialis/C6
Ans. All of the above are true206
  1. Left hemiparesis from a Cortical lesion, such as MCA CVA would be expected to produce
    1. Deltoid weakness more prominent than hand intrinsic muscle weakness
    2. Proximal lower limb muscle weakness more than distal muscle
    3. Deficit in visuospatial orientation
    4. Left hemianopia
    5. Neglect
Ans. B to E
  • With hemiparesis or monoparesis from cerebral cortical lesions, distal muscles are usually affected more prominently than proximal muscle, especially for the arm. With MC Alesion Proximal muscle of the leg may be more affected than distal muscle because the vascular supply to the proximal leg overlaps between MCA and ACA whearas cortex serving the lower limb is clearly in the ACA distribution
  1. Statements regarding cerebellar ataxia
    1. Hemisphere lesions produce predominantly appendicular ataxia
    2. Paraneoplastic cerebellar degeneration may cause predominantly gait and leg ataxia relatively sparing arm coordination
    3. Ataxia due to lesion of the vermis may have relatively preserved appendicular coordination
    4. The phylogenetically oldest portion of the cerebellum serves appendicular coordination
    5. Medulloblastoma produces truncal ataxia rather than gait or leg ataxia
Ans. A, B, C and E. D is false.
  • Lesion of the cerebellar hemisphere produce predominantly limb incoordination rather than gait ataxia. Vermis lesion produces predominantly gait ataxia with relative preservation of appendicular coordination paraneoplastic cerebellar degeneration commonly produces gait and leg ataxia.207
  1. Potential causes of Internuclear ophthalmoplegia includes
    1. Multiple sclerosis
    2. Cerebral hemorrhage with herniation
    3. Lacunar infarction
    4. Myasthenia gravis
Ans. A to C
  • Medial rectus paralysis or weakness look like INO but it is not INO
  1. Parinaud syndrome or dorsal midbrain syndrome can be caused by
    1. Pineal tumor
    2. Aqueductal stenosis
    3. Vascular malformations
    4. Stroke
    5. Multiplesclerosis
Ans. All of the above are true
  • Trauma also
  1. Statements regarding skull are
    1. Anterior cranial fossae is formed by cribriform plate of ethmoid, frontal bone body of sphenoid and lesser wing of sphenoid
    2. Middle cranial fossae is formed by body of sphenoid, greater wing of sphenoid, squamous part of temporal bone and pterous part of temporal bone
    3. Posterior cranial fossa is formed largely by occipital bone and pterous part of temporal bone
    4. Superior, Middle and inferiorconchae are part of ethmoid bone
    5. Nasal septum is formed by perpendicular plate of ethmoid and vomer bone
Ans. A to C and E
  • Ethmoid bone contain following parts
  • Cribriform plate
  • Crista galli
  • Sup nasal conchae
  • Middle nasal Conchae
  • Perpendicular plate of ethmoid
  • Inferior conchae is an independent bone attatched to maxilla208
  1. Statements regarding the skull are
    1. Optic canal lies between the body of the sphenoid and its lesser wing
    2. Superior orbital fissure is bounded above and medially by the lesser wing of the sphenoid and below and laterally by the greater wing of sphenoid
    3. Inferior orbital fissure is bounded above and laterally by the greater wing of the sphenoid and below and medially by the orbital surface of the maxille and is continuous anteriorly with the infraorbital groove which open on the surface through the infraorbital foramen
    4. Superior nuchal lines run laterally from external occipital protuberance and inferior nuchal line run laterally from external occipital crest
    5. Four bones, the parietal, frontal, sphenoid (greater wing) and temporal (squamous part) form the pterion
Ans. All of the above are true
  1. Which of the following are openings of the pterygoplatine fossa?
    1. Foramen ovale
    2. Foramen rotundum
    3. Greater palatine canal
    4. Palatinovaginal canal
    5. Sphenopalatine foramen
Ans. B to D
  • Opening into pterygopalatine fossa are
  • Inferior orbital fissure communicate to orbit
  • Foramen rotundum communicate to middle fossa
  • Petrygoid canal communicate to foramen lacerum
  • Palatinovaginal canal
  • Sphenopalatine foramen communicate to nasal only
  • Greater palatine canal communicate posterior part of palate
  1. Which of the following bone form the medial wall of the orbit?
    1. Greater wing of sphenoid
    2. Body of sphenoid
    3. Ethmoid
    4. Lacrimal bone
    5. Orbital plate of palatine
Ans. B to D209
  1. Which of the following foramina are present in the greater wing of sphenoid?
    1. Foramen rotundum
    2. Foramen ovale
    3. Emissary sphenoidal foramen
    4. Foramen spinosum
Ans. A, B and D
  1. Structures passing through the tendinous ring (annulus of zinn) of superior orbital fissure are
    1. Superior ophthalmic vein
    2. Trochlear nerve
    3. Abducent nerve
    4. Nasocilliary branch of Vth nerve
    5. Lacrimal branch of Vth nerve
Ans. C and D.
  • Structures passing through superior orbital fissure—Upper and lateral part are trochlear nerve, Frontal and lacrimal branch of Vth, recurrent branch of the ophthalmic superior ophthalmic vein
  • Middle within the Tendinous Ring or Occulomotor Foramen passes
  • Superior and inferior division of occlumotor nerve
  • Abducens nerve
  • Naso-cilliary nerve branch of V nerve
  • And Lower part contain inferior ophthalmic vein
  1. Structures passing through the foramen ovale are
    1. V2 (maxillary branch of V cranial nerve)
    2. V3 (mandibular branch of V cranial nerve)
    3. Accessory meningeal artery
    4. Emissary vein connecting cavernous sinus to pterygoid venous plexus
    5. Lesser pterosal nerve
Ans. B to E
  • V2 (maxillary branch of V cranial nerve passes through foramen rotundum
  • Lesser pterosal nerve passes through canalis innominatus but when it is absent it passes through foramen ovale210
  1. Statements regarding the skull
    1. Through anterior part of juglar foramen inferior pterosal sinus pass
    2. Through middle part of the jugular foramen cranial nerve IX, X, XI and meningeal branch of ascending pharyngeal artery pass
    3. Posterior part contain the sigmoid sinus, emissary vein connecting sigmoid sinus to occipital vein and meningeal branch of the occipital artery
    4. Nerve of pterygoid canal is formed by combining of deep pterosal nerve and greater pterosal nerve
    5. Foramen magnum transmit spinal cord through it
Ans. A to D
  • Foramen magnum transmit lower medulla which continue as spinal cord lower down
  1. Structure opening into the middle meatus of the nasal cavity are
    1. Maxillary sinus
    2. Frontal sinus
    3. Nasolacrimal canal
    4. Anterior ethmoidal sinus
    5. Posterior ethmoidal sinus
Ans. A to D.
  • Sphenoid sinus open into the sphenoethmoidal recess above the superior concha
  • In the superior meatus, posterior ethmoidal sinus and sphenopalatine foramen opens
  • Into the ethmoidal infudibulum opens the for frontal sinus, and anterior ethmoidal sinus
  • Maxillary sinus open into the hiatus semilunaris
  • Nasolacrimal canal opens into the inferior meatus middle ethmoidal sinus opens into ethmoidal bulla
  1. About Ossification of skull
    1. Inferior nasal concha and ethmoid are formed due to ossification of cartilage
    2. Mandible, sphenoid, occipital, temporal are formed partly in cartilage and partly in membrane
    3. Frontal bone has two centers of ossification on each side and at birth, the two halves are united by a midline suture called the metopic suture211
    4. Ethmoid bone has three centers of ossification
    5. Basilar part of the occipital bone fuses with the corresponding part of the sphenoid bone between 18 and 25 years
Ans. All of the above are true
  1. Conditions involving the Abducen's nerve or causing its paralysis are
    1. Pseudotumor cerebri
    2. Sphenoid sinusitis
    3. Gardenigos syndrome
    4. Carotid cavernous fistula
    5. Clival fracture
Ans. All of the above are true
  1. Regarding the cerebellopontine angle neuromas
    1. Acoustic neuroma show “transhiatal” extension from posterior fossa to middle fossa
    2. Facial neuroma shows spread across the midpetrosal bone
    3. Trigeminal neuroma shows trans apico pterosal extension
    4. Facial neuroma tend to erode the anterosuperior aspect of the IAC
    5. Acoustic neuroma tend to enlarge the IAC
Ans. All of the above are true
  1. Nystagmus (to and fro movement of the eye ball)
    1. Is not seen in people whose nervous system is normal
    2. May result from disease of the cochlea
    3. Is a feature of cerebellar disease
    4. Does not affect the acuity of vision
    5. Result from cerebral lesion
Ans. C.
  1. Statement about first order neuron
    1. Tactile receptor pass up the spinal cord in the ipsilateral column to the medulla
    2. Pressure receptors synapse in the dorsal horn with second order neurons which pass up the contralateral ventral spinothalamic tract to the thalmus
    3. Touch and pressure receptors are not segregated from one another in the spinal cord212
    4. Touch and pressure receptors are well characterized by all of the above statements
    5. Sensory neurons are not segregated in the spinal cord by function
Ans. A to D
  1. Unipolar or pseudounipolar neurons are present in
    1. Cranial nerve ganglia
    2. Dorsal root ganglia
    3. Mesencephatic nucleus of the trigeminal nerve (cranial V)
    4. Vestibular ganglia
    5. Spiral ganglia
Ans. A to C
  • Vestibular and spiral ganglia contain the bipolar neuron
  • Also in the eye and olfactory neuron
  1. Following structures are derived from alar plate
    1. Nucleus ambigus [SVE]
    2. Nucleus solitarius [GVA + SVA]
    3. Inferior olivary nucleus
    4. Pontine nucleus
    5. Hypoglossal nuclei [GSE]
Ans. B to D
  • Nucleus ambigus develop from basal plate and hypoglossal nuclei
  1. Regarding lesions of the central nervous system
    1. In lateral medullary syndrome, there is contralateral long tract sign
    2. In medial medullary syndrome, tongue deviation is toward contralateral site
    3. In Arnold chiari malformation II tectal beaking is characteristic
    4. In Millard Gubler syndrome, there is ipsilateral loss of lateral gaze to that side
    5. In weber syndrome there is ipsilateral long tract sign
Ans. A, C and D
  • XII nerve → (ipsilateral deviation not contralateral deviation213
  1. Which of the following cause myelopathy?
    1. Varicella Zoster Virus
    2. HSV-II
    3. HSV-I
    4. CMV
    5. HTLV-I
Ans. A, B, D and E
  • Varicella Zoster Virus → Necrotizing myelopathy
  • HSV-II → Ascending myclitis
  • Cyto Megalo Virus → Transverse myelitis
  1. Statement regarding auditory system
    1. In auditory receptors (hair cells) deflection of stereocillea toward the basal body causes depolaristion
    2. Deflection of stereo cilia of hair cells away from the basal body would hyperpolarise the cells
    3. The hair cells depolarize when stereocillia moves away from limbus and basilar membrane move upward
    4. The hair cell hyperpolarize when stereocillia moves toward the limbus and basilar membrane move downward
    5. Regarding primary auditory cortex the anterior part receive only low frequency tone arising from apex of the cochlea and the posterior part of the gyrus receives impulses arising from the base of the cochlea i.e. high frequency
Ans. All of the above are true
  1. Disease affecting lower motor neurons are
    1. Poliomyelitis
    2. Amyotrophic Lateral Sclerosis
    3. Werdnig Hoffman disease
    4. Kugelberg – welander disease
    5. Tetanus
Ans. All of the above are true
  1. GSE fibers are contained in the
    1. III
    2. V
    3. VII
    4. XI
    5. XII
Ans. A and E.214
  1. SSA fibers are contained in the cranial nerves
    1. I
    2. II
    3. IV
    4. VIII
    5. X
Ans. B and D.
  1. About sulcus
    1. Central sulci is a limiting sulcus
    2. Calcarine sulcus is an example of axial and complete sulcus
    3. Lunate is an operculated sulcus
    4. Parieto occipital sulcus is associated with the development of the corpus callosum
    5. Lateral sulcus is an example of secondary sulcus
Ans. All of the above are true
  1. Match the following Neurotransmitter with the site
    1. Ventral tegmentum → Dopamine
    2. Dorsal tegmentum → Enkephalin
    3. Lateral legmentum → Norepinephrine
    4. Pars reticulata of substatia nigra → Dopamine
    5. Locus ceruleus → Norepinephrine
Ans. A, B, C and E
  • Pars compacta of substantia nigra - Dopamine
  • Pars reticulata of substantia nigra → GABA
  1. Which layer of cerebral cortex is the major source of corticothalamic fibres is
    1. Layer II (External granular layer
    2. Layer III (External pyramidal layer)
    3. Layer IV (Internal granular layer)
    4. Layer V (Internal pyramidal layer)
    5. Layer VI (Multi form layer)
Ans. E.
  1. Regarding pathophysiology of pain
    1. Ascending pathway consist of spinoreticular fibre projecting pain impulses to the periaqueductal gray of the midbrain215
    2. Excitatory neurons of the periaqueductal gray project to the nucleus raphae magnus of the pons
    3. Excitatory neurons of the nucleus raphe magnus project serotonergic fibre to enkephalinergic inhibitor neurons of the substantia gelatinosa
    4. Enkephalinergic neurons of the substantia gelatinosaa inhibit afferent pain fibers (substance P) and tract neurons that give rise to the spinoreticular and spinothalamic tract
    5. Descending ceruleospinal pathway from the locus cerulus is thought to directly inhibit tract neurons that give rise to ascending pain pathways
Ans. All of the above are true
  1. Thalamic nuclei having sensory function are all EXCEPT
    1. VPL nuclei
    2. VPM nuclei
    3. LD nuclei
    4. MGB nuclei
    5. VL nuclei
Ans. C and E.
  • VL is a motor nuclei
  • LD is a limbic nuclei
  1. Neurofibrillary tangles are found in
    1. Progressive supranuclear palsy
    2. Alzheimer's disease
    3. Pick's disease
    4. Dementia complex of Guam
    5. Post encephalitic parkinsonism
Ans. A, B, D and E.
  • Also in chronic traumatic encephalapathy
  1. Phenytoin causes following side effects except
    1. Nystagmus
    2. Vertigo
    3. Diabetes Insipidus
    4. SIADH
Ans. D.
  • It is a side effect of carbamazepine causing retention of water and hyponatremia in elderly216
  1. All of the following are nerve supplying dura mater except
    1. Mandibular nerve
    2. Anterior ethmoidal nerve
    3. Posterior ethmoidal nerve
    4. Auricotemporal nerve
Ans. D.
  1. True regarding cilliary ganglion is
    1. Lies near apex of orbit, between optic nerve and medial rectus
    2. Preganglionic fibre comes from EW nucleus
    3. Sensory root comes from lacrimal nerve
    4. Sensory fibres relay in the ganglion
Ans. B.
  • Option 1 should be between optic nerve and lateral rectus
  • Option 3 should be sensory root comes from nasocilliary nerves
  • Option 4 should be sensory fibres passes through the ganglion
  1. Chromosomal alteration associated with medulloblastoma is
    1. 17P
    2. 6Q
    3. 16Q
    4. 1P
    5. 19Q
Ans. A, C
  • Chromosome alteration include 1q, 8p, 10q, 16q, 17p. Deletions on chromosome 1q and 10q are found in 20–40% of MB. Sonic hedge hog (shh) pathway, wingless (wnt) and ErB signaling pathway are defective in MB.
  1. Chromosomal alteration associated with meningioma is
    1. 19Q
    2. 20Q
    3. 22P
    4. 22Q
Ans. D.217
  1. Chromosomal alteration associated with oligodendroglioma is
    1. 1P
    2. 9P
    3. 19Q
    4. 22Q
    5. 6Q
Ans. A and C.
  1. According to WHO classification Grade I astrocytic tumor include all Except
    1. Subependymal giant cell astrocytoma
    2. Pleomorphic xanthoastrocytoma
    3. Juvenile pilocytic astrocytoma
    4. oligodendroglioma
Ans. D.
  1. A young hypertensive housewife suddenly severe right retro-orbital pain, prostration and a right–third cranial nerve palsy. The most probable cause is rupture of aneurysm of
    1. Anterior communicating artery
    2. Posterior inferior cerebellar artery
    3. Posterior communicating artery
    4. Middle cerebral artery
Ans. C.
  1. Schwannoma arise from following cranial nerve except
    1. V and VIII
    2. V and VI
    3. VI and VIII
    4. I and II
Ans. D.
  • Schwannomas may arise from any cranial or spinal roots except the optic and olfactory nerves because they are myelinated by oligodendroglia rather than schwann cell
  • Superior vestibular division of eighth cranial nerve common site.
  • 2nd most common Vth nerve
  • Schwannoma characteristically present as progressive unilateral hearing loss rather than with dizziness or other vestibular symptoms218
  • Acoustic schwannoma enlarge the internal audiory meatus and imaging feature that help distinguish the other CP angle mass
  1. A 46 – year old woman, had the sudden onset of jargon speech and hysteria. She was admitted to the psychiatric ward with a diagnosis of schizophraenia. Physical examination reveals only a right Babinski sign and right hemiparesis. The most probable diagnosis is
    1. Malingering
    2. Drug induced parkinsonism
    3. Left temporoparietal lesion
    4. Right temporoparietal lesion
Ans. C.
  1. Infratrochlear nerve is a branch of
    1. Frontal nerve
    2. Nasocilliary nerve
    3. Maxillary nerve
    4. Mandibular
Ans. B.
  1. Supraorbital and supratrochlear nerve are branches of
    1. Frontal nerve
    2. Nasocilliary nerve
    3. Maxillary nerve
    4. Mandibular nerve
Ans. A.
  1. Cranial nerves not having Obsteiner-Redlich zone are
    1. I and III
    2. IV and V
    3. V and VIII
    4. I and II
Ans. D.
  1. Trigemino thalamic tract project to which of the thalamic nuclei
    1. VPL nuclei
    2. VPM nuclei
    3. Pulvinar nuclei
    4. LP nuclei
Ans. B.219
  1. Teres major is supplied by
    1. Upper subscapular nerve
    2. Lower subscapular nerve
    3. Dorsal subscapular nerve
    4. Axillary nerve
Ans. B.
  • Upper subscapular nerve supplies subscapularis
  • Dorsal scapular nerve → Rhomboids and Levator scapulae
  • Axillary nerve → deltoid
  1. Glomus juglare arise from
    1. Superior vagal ganglion
    2. Inferior vagal ganglion
    3. Pterosal ganglion
    4. Auricular branch of vagus
Ans. A.
  • Inferior vagal ganglion [Nodose] give rise to glomus intravagale
  • Auricular branch give rise to glomus tympanicum
  1. Rathk's ceft cyst arise from
    1. Rathke's pouch
    2. Pars tuberalis of pituitary
    3. Pars intermedia of pituitary
    4. Pars distalis of pituitary
Ans. C.
  • Craniopharyngioma arise from Rathke's pouch
  1. VIII nerve is not damaged in which of the following # of pterous part of temporal bone
    1. Tansverse
    2. Oblique
    3. Tip
    4. Longitudinal
Ans. D.
  1. Naevus flammeus is
    1. A naevi
    2. A hemangioma
    3. Inflammation occurring in a naevi
    4. A premalignant lesion
220
Ans. B.
  • Naevus flammeus is port wine stain [capillary hemangioma]
  1. Which of the following cause the cerebral vasodilatation?
    1. ↑CO2
    2. ↓Pao2
    3. Adenosine
    4. ↑K+
    5. ↓PH
Ans. All of the above are true
  1. Structure derived from Neural ectoderm
    1. Retina with its pigment epithelium
    2. Epithelial layer of cilliary body
    3. Epithelial layer of iris
    4. Sphincter and dilator pupillae muscle
    5. Melanocytes
Ans. A to E.
  1. Which of the following sequence is not present in MRI ?
    1. FLAIR
    2. GRE
    3. HASTE
    4. SMASH
    5. BEST
Ans. E.
  • FLAIR is fluid attentuated inversion recovery, GRE is gradient echo imaging, HASTE is half acquisition single shot turbo spin echo, SMASH is simultaneous acquisition of spatial harmonics.
  1. Which of the following muscle is not attached to Occiput ?
    1. Rectus capitis major
    2. Obliqus capitis superior
    3. Obliqus capitis inferior
    4. Rectus capitis minor
Ans. C.221
  1. Chroid plexus is absent in
    1. Anterior horn or Frontal horn of lateral ventricle
    2. Occipital horn of lateral ventricle
    3. Aqueduct of Sylvius
    4. Lateral recess of the IVth ventricle
Ans. A to C
  1. Spinal leminiscus contain the
    1. Lateral spinothalamic tract
    2. Anterior spinothalemic tract
    3. Spinotectal tract
    4. Fasciculus Gracilis
Ans. A to C.
  1. Markers for Asrocytes are
    1. Glutamine synthase
    2. Glycogen
    3. GFAP (Glial Fibrillary Acidic Protein)
    4. Vimentin
Ans. A to C
  1. About mechanisms of headache in tumors
    1. Traction on the veins draining into the large venous sinuses
    2. Traction on the middle meningeal artery
    3. Traction on the major arteries at the base of brain
    4. Distension and dilatation of the intracranial and extracranial arteries
    5. Most of the dura and pia, arachnoid the brain parenchyma and the lining of the ventricles were sensitive to pain
Ans. A to D
  • The mechanism of pain in patients who have brain tumor is presumed to be the result of traction on the large blood vessels and dura and direct pressure on cranial and cervical nerve fibers by tumor. The venous sinuses, some of dura at the base of brain were sensitive to pain
  1. About central neurocytoma
    1. Well differentiated intraventricular tumor affecting young men and women equally
    2. Present in anterior portion of the lateral ventricle around the foramen of Monro222
    3. CNC metabolism is more oxidative than that of other brain tumor
    4. Hallmark characteristic of CNC is positivly for synaptophysin
    5. Derived from bipotential progenitor cells from the subependymal plate
Ans. A to E
  1. Commisural and association fibers arise from
    1. Layer II, III, IV, V of cerebral cortex
    2. Layer III and VI of cerebral cortex
    3. Layer II, III and VI of cerebral cortex
    4. Layer III and IV of cerebral cortex
Ans. C
  • Layer II give rise to Association fibers
  • Layer III give rise to Association and commissural fibers
  • Layer VI give rise to Association, commissural and projection fibers
  1. About Basal ganglia
    1. Globus pallidus is derived from Telencephalon
    2. GPi: and Pars reticularis are the main output centres
    3. Field of forel H2 is Thalamic fasciculus
    4. Field of forel H1 is lenticular fasciculus
Ans. B.
  1. Match the following
    1. Superior cerebral vein - Sagittal sinus (superior)
    2. Superficial middle cerebral vein - Cavernous sinus
    3. Decepmiddle cerebral vein - Anastomotic vein → Cavernous sinus
    4. Inferior pterosal sinus - Internal jugular vein
    5. Superior pterosal sinus - Sigmoid sinus
Ans. A, B, D and E.
  • Deep middle cerebral vein drain into either the saggital sinus (superior) or Transverse sinus – through two anastomotic vein. Vein of labbe drain into transverse sinus. Vein of trolard drain into superior sagittal sinus.223
  1. Match the following
    A.
    Tensor veli palatani
    - Vagus nerve
    B.
    Posterior belly of digastric
    - Mandibular nerve
    C.
    Levator veli Palatani
    - Mandibular nerve
    D.
    Palatoglossus
    - Hypo glossal nerve
    E.
    Cricothyroid
    - Internal branch of superior Laryngeal nerve
Ans. All of the above are false
  • Tensor veli palatani is supplied by Mandibular division of Trigeminal nerve
  • Posterior belly of digastric is supplied by facial nerve
  • Levator velipalatani is supplied by accessory nerve through pharyngeal branch of vagus
  • Palatoglossus is supplied by vagus nerve
  • Cricothyroid is supplied by external branch of superior laryngeal nerve. Internal branch is sensory
  • All muscle of soft palate except tensor veli palatani is supplied by accessory nerve [XI] through pharyngeal branch of vagus
  1. Match the following
    A.
    Olivocerebellar fibre
    - Mossy fiber
    B.
    Dorsal spinocerebellar tract
    - Superior cerebellar peduncle
    C.
    Ventral spinocerebellar tract
    - Inferior cerebellar peduncle
    D.
    Pontocerebellar fibre
    - Middle cerebellar peduncle
    E.
    Juxta restiform body
    - Superior cerebellar peduncle
Ans. D.
  • Olivocerebellar fibers are climbing fibres. Dorsal spinocerebellar tract enter through inferior cerebellar peduncle. Ventral spinocerebellar tracts enter through one superior cerebellar peduncle. Juxta restiform body is a part of inferior cerebellar peduncle
  1. Solitary nucleus is shared by nerves
    1. V
    2. VII
    3. IX
    4. X
    5. XII
Ans. B, C and D224
  1. Ambigus Nucleus is shared by nerves
    1. VI
    2. VII
    3. IX
    4. X
    5. XI
Ans. C to E
  1. About craniopharyngiomas
    1. Dual theory of embryological development is implicated
    2. Bimodal incidence with highest incidence between 5–15 years and between 50–75 years
    3. Papillary variant are less aggressive than the adamantinomous variety
    4. Papillary variant more common in children
    5. Lack of calcification predicts a better survival rate
Ans. A, B, C and E
  • The embryonic rest cells from the craniopharyngeal duct are responsible for the development of craniopharyngioma. The second theory says that the existing rest cells of the adenohypophysis undergo metaplasia. Form up to 12% in pediatric brain tumor and 7–9% of all intracranial tumor. The pediatric population in more than 95% have adamantinomatous variety whereas papillary variant is more common in adults. Admantinomatous craniopharyngioma are more often in the suprasellar area. They contain crank case oil/Machine oil like fluid due to cholestrol droplets. Papillary craniopharyngioma are solid and lack cholestrol droplet. Immunostaining for cytokeratin is valuable in the investigation of craniopharyngioma.
  1. Branches of cervical plexus are
    1. Supraclavicular nerve
    2. Suprascapular nerve
    3. Phrenic nerve (C3, C4, C5)
    4. Great auricular nerve (C2, C3)
    5. Lesser occipital nerve (C2)
Ans. A, C to E.225
  1. Muscle supplied by Ansacervicalis
    1. Sternohyoid
    2. Sternothyroid
    3. Thyrohyoid
    4. Geniohyoid
    5. Omohyoid [inferior belly]
Ans. A, B and E.
  • Superior belly of omohyoid not supplied by Ansa cervicalis
  1. Basal vein of Rosenthal is formed by
    1. Superior cerebral vein
    2. Anterior cerebral vein
    3. Superficial middle cerebral vein
    4. Deep middle cerebral vein
    5. Inferior striate veins through anterior perforated substance
Ans. B, D and E.
  1. About verebral column
    1. Only a small amount of movement is possible at the atlanto-occipital joint, most of which is in flexion and extension (nodding movement) of the head
    2. The atlanto-axial joint allows rotation and in fact accounts for most of the rotation which occurs in the cervical spine
    3. The vertebral artery and vein does not pass through C7 instead accessory vertebral vein pass through C7 [Foramen transversium]
    4. The articular facet joint in the upper thoracic spine is in coronal plane thus allowing rotation in addition to flexion extension and lateral flexion
    5. The articular facet joint in the lower thoracic and lumbar spine is in sagittal plane thus preventing rotation between vertebrae
Ans. All of the above are true
  • The coronal orientation of the facets in the upper thoracic spine leads to significant resistance to anterior translation, but little resistance to rotation. In the lower thoracic spine, the facet becomes more-saggitaly oriented and less resistance to anterior translation is offered226
  1. Axons of the First order neuron are present in the
    1. Fasciculus gracilis
    2. Dorsal spinocerebellar tract
    3. Fasciculus cuneatus
    4. Ventral spinocerebellar tract
    5. Lateral spinothalamic tract
Ans. A and C.
  1. About Spinal cord
    1. Substantia gelatinosa is present at all levels
    2. Dorsal intermediate sulcus is present only rostral to T6
    3. Nucleus Dorsalis of Clarke is present from L1 to S3 level
    4. Intermedio lateral horn extend from T1 to L2 or L3
    5. White rami communicans is present at all levels
Ans. A, B and D.
  1. About central nervous system development
    1. Mantle zone of neural tube form the grey matter of the spinal cord and the alar and basal plate separated by sulcus limitans form the Dorsal and ventral grey horn respectively
    2. Leptomeninx is formed by neural crest cell
    3. Microglia and Pachymeninx are mesodermal in origin
    4. Marginal zone form the white matter of the spinal cord
    5. Anterior and posterior neuropore closer at day 24 and day 27 of the development
Ans. All of the above are true
  1. About Filum terminale
    1. Consist of arachnoid, glial fibres and often contains a vein
    2. It is difficult to differentiate a tethered cord from a congenitally low lying cord. Filum diameter is generally less than 1mm in latter
    3. The filum is differentiated from nerve roots by presence of characteristic squiggly vessel on surface of filum
    4. Filum has a distinctively whiter appearance than the nerve root
    5. Myxopapillary ependymoma occur in the filum terminale
Ans. B to E.
  • Filum terminale consist of Pia, glial fibres and often a vein
  • Normal diameter is less than 1mm and >2mm is definitely pathological227
  • Arachnoid and dural meninges surround the filum terminale in its upper 15cm. The central canal is continued in the filum terminale for [5.6mm]
  1. Various opening in the pterygopalatine fossa are
    1. Foramen Rotundum
    2. Pterygoid canal
    3. Palatinovaginal canal
    4. Sphenopalatine foramen
    5. Greater palatine canal
Ans. All of the above are true
  • Other is the inferior orbital fissure
  1. Bones of the skull that are formed in membrane are
    1. Frontal
    2. Parietal
    3. Occipital
    4. Temporal
    5. Maxilla
Ans. A, B and E.
  • Bones that are formed in cartilage are → Inferior nasal concha, Ethmoid
  • Bones formed from both [membrane + Cartilage] → Mastoid, sphenoid, temporal
  1. Sphenoid bone form the following boundary of the orbit
    1. Roof
    2. Floor
    3. Medial wall
    4. Lateral wall
Ans. A, C and D.
  • Lesser wing of sphenoid along with the orbital plate of the frontal bone form the roof of the orbit
  • Floor of the orbit is formed by maxilla, Zygomatic and in posterior part by orbital process of palatine
  • Lateral wall is formed by Zygomatic process and greater wing of sphenoid
  • Medial wall is formed by body of sphenoid Ethmoid Lacrimal bone and Maxilla228
  1. Match the following opening of Sinuses
    A.
    Sphenoid sinus
    - Bulla ethmoidalis
    B.
    Maxillary sinus
    - Upper part of hiatus semilunaris or infundibulum
    C.
    Anterior ethmoidal sinus
    - Lower part of hiatus semilunaris
    D.
    Middle ethmoidal sinus
    - Superior meatus
    E.
    Posterior ethmoidal sinus
    - Sphenoethmoidal recess
Ans. All of the above are false
  • Sphenoid sinus open above the superior concha into sphenoethmoidal recess
  • Maxillary sinus open into the hiatus semilunaris (lower part)
  • Anterior ethmoidal sinus open into the upper part of the hiatus semilunaris or infundibulum
  • Middle ethmoidal sinus open into the bulla ethmoidals posterior ethmoidal sinus open into the superior meatus
  • Frontal sinus open into middle meatus
  • Nasolacrimal duct open into inferior meatus sphenopalatine foramen open into the superior meatus
  1. About association fibers
    1. Superior longitudinal fasciculus connect the occipital to temporal lobe
    2. Inferior longitudinal fasciculus connect the Frontal to temporal and occipital lobe
    3. Uncinate fasciculus connect the orbito frontal gyri to the temporal lobe
    4. Arcuate fasciculus connect the superior and middle frontal convolution to the anterior temporal lobe
    5. Cingulate fasciculus connect the medial part of the frontal and parietal lobe to parahippocam pal gyrus
Ans. C to E.
  • Inferior longitudinal fasciculus connect occipital to temporal lobe. It is essential for visual recognition. Cerebral damage as that caused by carbon monoxide poisoning can destroy the inferior longitudinal fasciculus bilaterally. In such cases the individual has intact elementary vision but cannot identify the nature of objects (object agnosia) or individual faces (prosopagnosia) although he/she can depict and match them
  • Superior longitudinal fasciculus connect the frontal to temporal and occipital lobe229
  • Uncinate fasciculus is important in the regulation of behaviour
  • Arcuate fasciculus is important in language function
  • Inferior longitudinal fasciculus constitutes to the function of visual recognition
  1. Following statement about derivative of the pharyngeal arch
    1. Facial nerve is the nerve of second arch
    2. Lesser cornu of hyoid bone and body is derivative of first arch
    3. Maeckl's cartilage, malleus incus are derivative of second arch
    4. Cricothyroid muscle, Thyroid cartilage is the derivative of fourth arch
    5. Arytenoid cartilage, Greater cornu of the hyoid derivative of sixth arch
Ans. A and D.
  • Lesser cornu of hyoid bone and body is derivative of second arch
  • Maeckl's cartilage, malleus, incus are derivative of the first arch
  • Arytenoid cartilage is derivative of the sixth arch
  • Greater cornu of hyoid is derivative of third arch
  1. About nerves of the arches
    1. Mandibular branch of the trigeminal nerve is the nerve of second arch
    2. Facial nerve is the nerve of the first arch
    3. Glossopharyngeal nerve is the nerve of the fourth arch
    4. Superior laryngeal nerve is the nerve of the sixth arch
    5. Recurrent laryngeal nerve is the nerve of the third arch
Ans. All of the above are false
  • First arch → Mandibular division of the trigeminal nerve
  • Second arch → Facial nerve [VII]
  • Third arch → Glossopharyngeal nerve [IX]
  • Fourth arch → Superior laryngeal nerve [X]
  • Sixth arch → Recurrent laryngeal nerve [X]
  1. Nerve conduction velocity is reduced in
    1. Gullian Barre Syndrome
    2. Amytotrophic lateral sclerosis230
    3. Myasthenia gravis
    4. Multiple sclerosis
    5. HMSN II
Ans. A.
  1. SVE fibers are present in
    1. III [Occulomotor nerve]
    2. V [Trigeminal nerve]
    3. VII [Facial nerve]
    4. IX [Flossopharyngeal nerve]
    5. XI [Spinal accessory nerve]
Ans. B to E.
  1. About ataxia
    1. Romberg sign is positive in sensory ataxia
    2. Frederich ataxia has the same lesion in the spinal cord as of subacute combined degeneration of cord
    3. Destruction of the neocerebellum result in the Truncal ataxia
    4. Destruction of the vermis result in appendicular ataxia
    5. Most of hereditary spinocerebellar ataxia are due to trinucleotide repeat
Ans. A, B and E.
  1. Following statements concerning brainstem auditory evoked potentials are
    1. Increased I – III interpeak latency is a suggestive for acoustic neuroma
    2. Absence of all waves is supportive of brain death
    3. Wave I is generated by charge movement in the cochlea
    4. In acoustic neuroma, there is prolongation of the III – V interpeak interval
    5. There is loss of all waves including I in acoustic neuroma
Ans. All of the above are false
  • Explanation →
  • Acoustic neuroma is strongly suspected in patients with increased I – III interpeak interval, although this finding is not specific. Absence of all waves beyond wave I or distortion of following wave forms can occur, making measurement of this interval impossible. BAEP can be supportive of brain death, but not if there is no wave I since one cannot rule 231out failure of transaduction of the auditory stimulus to a neural signd. Wave I is generated by the dibtal portion of CN-8.
  • Wave I is present in the patient of acoustic neuroma
  • Increased III-V interpeak interval could not be expected since the segment which serve this conduction is in the brain stem
  • Wave I—Cochlear nerve
  • Wave II—Cochlear nuclei
  • Wave, III—Superior olivary nucleus
  • Wave IV—Lateral leminiscus
  • Wave V—Nucleus of inferior colliculus
  • Wave VI—Medial geniculate nucleus
  • Wave VII—Auditory radiation
  1. Light – Near dissociation is seen in
    1. Holmes-adie pupil
    2. Argyll robertson pupil
    3. Parinauds syndrome
    4. Marcus gunn pupil
    5. Lupus Erythematosus
Ans. A, B, C and E
  1. About Fornix
    1. Fornix contain 27 million fibers and is the largest projection to the hypothalamus
    2. Bilateral destruction result in acute amnestic syndrome
    3. Hippocampal commisure or commisure of the fornix band of transverse fiber connecting two column of fornix
    4. Part of the fornix in correct order are Fimbria, Crura, Alveus, Body, column
    5. Axons from the subicular neurons via post commissural fornix enter the medial mammilary nucleus
Ans. A, B and E
  • Hippocampal commisure or commisure of the formix connect two crura of the fornix
  • Correct order of part in the fornix relaying from hippocampal formation are Alveus, Fimbria, Crura, Body and Column232
  1. About Commisures
    1. Corpus callosum is the largest commisure and the first commisure to appear
    2. Anterior commisure connect the superior and middle temporal gyri of the two lobes (temporal) and anterior olfactory nuclei
    3. Destruction of posterior commisure will result in light-Near dissociation
    4. In patient with agenesis of corpus callosum only minimal deficit can be demonstrated with neuropsychological testing as compared to a patient with commisurotomy
    5. Commisure of the fornix (Hippocampal) commisure is the second commisure to appear in development
Ans. D and E.
  • Development of commisure in order
  • Anterior commisure1st to appear
  • Hippocampal or Fornical commisure 2nd to appear
  • Corpus callosum 3rd to appear
  • Anterior commisure connect the middle and inferior temporal gyri of the temporal lobe
  • Posterior commisure contain the puppilary reflex fibres from the pretectal nuclei to the parasympathetic nuclei bilaterally
  • Destruction of posterior commisure will abolish the consensual reflex
  1. Following are found in Kluver-Bucy syndrome
    1. Hyperorality
    2. Hypersexuality
    3. Psychic Blindness or visual agnosia
    4. Docility
    5. Due to destruction of bilateral frontal lobe
Ans. A to D
  • It is due to destruction in the bilateral temporal lobe
  1. Destruction of the Otic ganglion will result in
    1. Inability of secretion of parotid gland
    2. Decreased sensation
    3. Hyperacusis
    4. Paralysis of tensor veli palatni
    5. Paralysis of stapedius muscle233
Ans. A to D.
  • A few somatic motor fibers from the trigeminal nerve pass through the otic ganglion and supply the tensor tympani and tensor veli palatani
  1. Deviation of the eyes to the right is most likely to occur with occlusion of the
    1. Calcarine artery on the ipsilateral side
    2. Calcarine artery on the contralateral side
    3. Ipsilateral paramedian branch of the basilar artery
    4. Ascending frontal branch of the ipsilateral middle cerebral artery
    5. Ipsilateral superior cerebellar artery
Ans. A and D.
  1. Grestmann's syndrome comprise of
    1. Agraphia [copying is not affected]
    2. Dressing apraxia
    3. Finger agnosia
    4. Right-left disorientation
    5. Dyscalculia
Ans. A, C to E.
  1. GABA ergic neurons are present in
    1. Pars reticularis of substantia negra
    2. Neostriatum
    3. Pyramidal cell of the cerebral cortex
    4. Stellatecell, Basket cell purkinje cell of the cerebellar cortex
    5. Climbing fibres of the cerebellar cortex
Ans. A, B and D
  1. Dopaminergic neurons are present in the
    1. Pars compacta of the substania nigra
    2. Ventral tegmental area
    3. Raphe nuclei
    4. Periaqueductal grey matter
    5. Superior colliculus, arcuate nuclei of the hypothalamus
Ans. A, B, and E.234
  1. About Lesion
    1. Occlusion of left posterior cerebral artery result in alexia without agraphia or aphasia
    2. Occlusion of right posterior cerebral artery may result in propsagnosia
    3. Destruction of inferior parietal lobule of dominant side result in dressing apraxia, hemineglect, topograptic memory loss, constructional apraxia, anosognosia
    4. Destruction of arcuate fasciculus result in fluent, good comprehension and non-repetitive speech
    5. Destruction of right angular gyrus in inferior parietal lobule result in Gerstmann's syndrome
Ans. A, B and D
  1. About sweat glands
    1. Holocrine, eccrine, apocrine are three type of sweat glands
    2. Apocrine sweat gland has got the highest concentration in the axilla
    3. Sweat is the ultrafiltrate of the plasma
    4. Evaporation of sweat relased by the eccrine gland result in thermoregulation
    5. Apocrine sweat gland receive post ganglionic norephinephine releasing sympathetic fibres
Ans. A, B, D and E.
  • Eccrine or merocrine sweat glands are distributed through out the body. They receive cholinergic sympathetic fibres
  1. About Internuclear opthalmoplegia
    1. Lesion causing it is always intra-axial
    2. Result in horizontal diplopia
    3. Eyes never cross the midline
    4. When bilateral in adults is suggestive of multiple sclerosis
    5. When occur in the elderly is usually due to hypertensive bleed and is unilateral
Ans. B, D and E
  • Internuclear opthalmoplegia can also occur in transtentorial herniation. But commonly it is due to intra-axial lesion.235
  1. About olfactory and gustatory pathway
    1. Parabrachial nucleus of the pons is related with emotional aspect of the taste
    2. Pyriform and entorhinal cortex are associated with olfaction
    3. Olfactory fibers are not relayed in thalamus
    4. Gustatory fibers are relayed in small cell of the ventro posteromedial nucleus of the thalamus
    5. Sweet and sour taste area are present on the tip of tongue and dorsum of the tongue respectively
Ans. All of the above are true
  1. About olfaction and gustation
    1. There are four types of primary taste
    2. Alpha Gusducin is related to salty and sour taste
    3. Conscious perception of smell take place in Pyriform cortex
    4. Olfactory nerve comprise of the axons of the granule and tufted cells
    5. Entorhinal cortex (Area 28) is the association alfactory cortex
Ans. E.
  • There are five types of primary taste: salty, sour, sweet, bitter and umami
  • Alpha Gusducin linked to G-Protein is related to sweet and bitter taste
  • Conscious perception of smell take place in the prefrontal cortex (9–12)
  • Olfactory nerve arise from receptor cells. Axons of the mitral and tuffted cell form lateral and medial olfactory tract
  1. About Internal capsule
    1. Anterior radiation and fronto pontine fibres are present in the anterior limb
    2. Sensory radiation is the superior radiation
    3. Hemorrhage lateral to the internal capsule has worst prognosis than when it is medial to the internal capsule
    4. Genu of the internal capsule is supplied by recurrent artery of heubner
    5. Anterior choroidal artery supply the genu and posterior limb of the internal capsule236
Ans. A, B and E.
  • Putaminal hemorrhage or hemorrhage just lateral to the posterior limb of internal capsule tend to compress the fiber but the hematoma that is medial to the internal capsule invade or distruct the fibres. So medial hemorrhage has got worst prognosis than the lateral one
  • Anterior limb of internal capsule is supplied by medial striate branch i.e. recurrent artery of heubner branch of anterior cerebral artery and lateral striate branches of middle cerebral artery
  • Genu is supplied by anterior choroidal artery and the some direct branches from the internal carotid artery
  1. The Last Fontanelle to close is
    1. Posterior
    2. Anterolateral
    3. Posterolateral
    4. Anterior
Ans. D.
  • Anterior closes in the middle of second year. Posterior and sphenoidal (anterolateral) closes in 2–3 months
  • Posterolateral (mastoid) closes at the end of first year
  1. About CSF
    1. CSF is produced only by choroid plexus
    2. Compensatory CSF absorption may be done by choroids plexus
    3. CSF drainage occur along the I, II, V, VI cranial nerves into the cervical lymphatics
    4. Arachnoid villi which result in CSF absorption is mainly distributed along the cavernous sinus
    5. PH of CSF is 7.3 and density is 1.007
Ans. B and E.
  • CSF is also produced by ependymal cell and along the dural sleeves of spinal nerves also produce it
  • CSF drainage occur along the I, II, VII, VIII cranial neve into the cervical lymphatics
  • Arachnoid villi is mainly distributed along the superior sagittal sinus
  • Compensatory CSF aborption done by choroids plexus, epedymal cell and dilated central canal of the spinal cord237
  1. Number of arteries taking part in the blood supply of Optic chiasma are
    A. 9
    B. 10
    C. 11
    D. 12
Ans. C.
  • Branches from pairs of internal carotid artery, middle cerebral aretery, anterior cerebral artery, anterior choroidal artery, posterior communicating and anterior communicating artery
  1. Normal position of optic chiasma is
    1. Sulcus chiasmaticus
    2. Diaphgram sellae
    3. Dorsum sellae
    4. Posterior to dorsum sellae
Ans. C
  • Sulcus chiasmaticus – 5%
  • Diaphgram sellac – 12%
  • Dorsum sellae – 79%
  • Posterior to Dorsum sellae – 4%
  1. About Craniovertebral Junction development
    1. First two occipital scleretome form the basiocciput
    2. Third occipital scleretome form the anterior tubercle of the clivus
    3. Odontoid process has two primary and one secondary ossification centre
    4. First cervical vertebrae [Atlas] is formed entirely by first cervical sclerotome
    5. Proatlas is fourth occipital sclerotome
Ans. A, C and E.
  • First two occipital sclerotome leads to development of basiocciput
  • Third occipital scelrotome leads to development of two jugular tubercles
  • Fourth occipital sclerotome (Proatlas) leads to formation of anterior margin of the foramen magnum, two occipital condyles, anterior tubercle of clivus, apical ligament of odontoid, Superior articular facet of the atlas, superior part of the posterior arch of the atlas and the apical segment of the odontoid238
  • The first cervical sclerotome form the rest of the atlas
  • The second cervical sclerotome form the body of the axis.
  • Development of axis
  • Odotoid process, apical segment is developed from proatlas and remaining portion from first cervical scelerotome and the body of the axis from the second cervical sclerotome
  1. Biochemical defect present in the majority of brain tumor measured by Magnetic resonance spectroscopy are
    1. Decreased NAA
    2. Decreased Lactate
    3. Decreased Lipid
    4. Decreased Choline
    5. Increased Creatinine
An. A.
  • There is increase in lactate, lipid and choline but decrease in NAA and total creatinine
  1. About Cerebral abscess
    1. In both developed and developing countries otogenic abscess is the commonest cause of brain abscess
    2. Cut off margin of the size of the abscess above which it should be treated by surgical means is 5cm
    3. Metastatic brain abscess is more common in occipital lobe
    4. Risk for epilepsy is more after surgical excision than after aspiration of abscess
    5. Staph-aureus is the most common cause for epidural spinal abscess but not for intradural and intermedullary abscess
Ans. All of the above are false
  • In developed countries, the cause of brain abscess is due to haematogenous spread secondary to lung infection. In developing countries, it is still otogenic abssess
  • Cut off margin is 3cm
  • Metastatic brain abscess is more common in frontal lobe
  • Risk for epilepsy is more after aspiration of abscess than after surgical excision because scar after aspiration is much more denser than after surgical excision
  • Staph aureus is the common cause for epidural, Intradural and Intramedullary abscess239
  1. Which of the following is the correct match?
    A.
    PICA
    - Choroid plexus of the IVth ventricle
    B.
    PCA
    - Choroid plexus of the lateral and third ventricle
    C.
    Anterior choroidal artery
    -Choroid plexus temporal horn of the lateral ventricle
    D.
    AICA
    - Labyranthine artery
    E.
    PICA
    - Posterior spinal artery
Ans. All of the above are true
  1. Which of the following is a frequent symptom of extramedullary intradural tumor?
    1. Muscle fasciculations
    2. Bladder and rectal involvement
    3. Muscle atrophy and trophic skin changes
    4. Spinal fluid changes
Ans. D.
  1. Destruction of right occlumotor nucleus will result in
    1. Drooping of the only right eyelid
    2. Paralysis of only left superior rectus
    3. Paralysis of right inferior rectus, inferior oblique and medial rectus
    4. Right pupil will be dilated
    5. Right “Down and out” position of the eye
Ans. C to E.
  • Drooping of the both eyelid will take place because of presence of single caudal subnucleus of Parelia for levator palebrae superioris
  • Paralysis of both right and left superior rectus will take place because of the fibres of the right superior rectus from left side pass very close to the right subnucleus for superior rectus
  1. The content of ambient cistern are all EXCEPT
    1. Superior cerebellar artery
    2. P3 of PCA
    3. P2 of PCA
    4. IVth cranial nerve
Ans. B
  • P3 of PCA present in quadrigerminal cistern.240
  1. About spinal cord
    1. Cord transverses the whole vertebral column at birth
    2. Neurofibroma mainly involves the structure derived from basal plate
    3. Lumbosacral cistern is present from [L2 – S5]
    4. C6 is the most common nerve root to be involved in the cervical spondylosis
    5. Weight of spinal cord is about 5% of the weight of the brain
Ans. D.
  • At birth, cord ends at the L3 level. Alarplate give rise to the sensory structures and neurofihbroma has predilection for dorsal root. Lumbo sacral cistern is present from [L2 – S5]. Weight of spinal cord is about 2% of the weight of the brain [28gm]
  1. Which of the following tumor is correctly matched to its histology?
    A.
    Prolactin secreting adenoma
    - Fibrous body
    B.
    Growth hormone secreting
    - Nebenkernsadenoma
    C.
    Chordoma (Notochord)
    - Veroccay bodies
    D.
    Acoustic schwanomima
    - Physalipharous cell
    E.
    Medulloblastoma
    - Homer Wright Rosettes
Ans. E.
  • Prolactin secreting adenoma is characterized by concentric whorl of rough endoplasmic reticulum called [NEBENKERNS]
  • Growth hormone secreting adenoma is characterised by Fibrous body
  • Chordoma is characterized by presence of physalipharous cells or swollen or vacuolated or Bubbly cells
  • Acoustic schwannoma is characterized by Antoni A, Antoni B areas and veroccay bodies
  1. About Derivatives of arches
    1. Posterior belly of digastric, stapedius and orbicularis oris are derivative of hyoid arch
    2. Tensor tympani, anterior belly of digastric mylohyoid, Tensor levi palatani are the derivative of first arch
    3. Meckel cartilage, Malleus, incus, Sphenomandibular ligament, Stylomandibular ligament are derivative of first arch241
    4. Arytenoid cartilage is derivative of sixth arch
    5. Styloid process, stylohyoid ligament, lesser cornu and upper body is derived from second arch or hyoid arch
Ans. All of the above are true
  1. Stylohyoid ligament is derived from
    1. First arch
    2. Second arch
    3. Third arch
    4. Fourth arch
Ans. B.
  1. Unprovoked uncontrolled laughter is present in
    1. Chordoma
    2. Chondroma
    3. Clival meningioma
    4. Nasopharyngeal growth
    5. Basilar aneurysm
Ans. All of the above are true
  • Unprovoked, uncontrolled laughter is a characteristic symptom of Prepontine Tumors. May respond to Selective serotonin reuptake inhibitor therapy (e.g. Paroxetine)
  1. About Papilledema
    1. Earliest indication of papilledema is the enlargement of the blind spot
    2. Loss of visual acuity is earlier in papilledema
    3. Papilledema is due to stasis of axoplasm
    4. Fluorescein angiograply helps to differentiate from pseudopapilledema
    5. Spinal neurofibroma may present with papilledema
Ans. A, C to E.
  • Loss of visual acuity is late in papilledema
  1. The ratio of crossed and uncrossed fibres in the optic chiasma is
    1. 70 : 30
    2. 30 : 70
    3. 50 : 50
    4. 53 : 47242
Ans. D.
  • The ratio of crossed and uncrossed fibres is about 53:47, a large proportion of uncrossed fibres that has been reported in the past. Approximately 53% of optic nerve axons cross in the optic chiasm. These subserve the temporal field for each eye
  1. Regarding scotomal progression in pituitary and craniopharyngioma
    1. Clockwise in right eye in pituitary adenoma and craniopharyngioma
    2. Anticlockwise in left eye in Pituitary adenoma and craniopharyngioma
    3. Anticlockwise in right eye in Pituitary and clockwise in left eye in craniopharyngioma
    4. Clockwise in right eye in pituitary and clockwise in left eye in craniopharyngioma
Ans. D.
  • Pressure from Pituitary tumor is from downward and from craniopharyngioma is from above
  1. According to Collins law, the risk of recurrence of tumor for a patient presented at an age of 2 years with Medulloblastoma is within
    1. 32 months
    2. 33 months
    3. 34 months
    4. 35 months
Ans. B
  • Collins law – the rule states that the period of risk for recurrence is equal to the age at presentation plus 9 month of gestational age
  • Applying collins law – [12 + 12] + 9 months = 24 + 9 months = [33] months
  1. About Obsteiner – Redlich zone
    1. Found at the junction of the central and peripheral myelin
    2. Give rise to acoustic shwanomma
    3. Is absent in cranial nerve II
    4. Is often a site for microvascular compression
    5. It is present at internal acoustic meatus in case of vestibulocochlear nerve
Ans. All of the above are true243
  1. Thalamic peduncle which carry sensory information is
    1. Anterior
    2. Superior
    3. Posterior
    4. Inferior
Ans. B
  1. The principal postnatal change in the pyramids is due to
    1. an increase of corticospinal neurons from the paracentral lobule
    2. an increase in the total number of corticospinal axons
    3. an increase in endoneural tubes to guide sproutingaxons
    4. myelination of pre-existing corticospinal axons
Ans. D
  • The corticospinal fibers are not completely myelinated at birth. This does not occur until between 18 months and 2 years of age. During this time, the Babinski reflex is elicitable later it is suppressed
  1. Which of the following gas increases the cerebral metabolism?
    1. Halothane
    2. Enflurane
    3. Isoflurane
    4. N2O
Ans. D.
  1. Isoelectric EEG produced by
    1. Morphine + fentanyl
    2. Mepridine
    3. Barbiturate
    4. Barbiturate + Isoflurane
Ans. D
  • Arbiturate + Isoflurane produces Isoelectric EEG. Narcotics do not produce Isoelectric EEG
  1. Regarding steroids
    1. In primary adreno cortical insufficiency (Addison's disease) both mineralocorticoid and glucocorticoid has to be replaced
    2. In secondary adrenocortical insufficiency due to decreased release of ACTH only glucocorticoid has to be replaced and mineralocorticoid is normal244
    3. Most common cause of cushing's syndrome is iatrogenic
    4. Cushing disease is due to Pituitary adenoma releasing ACTH
    5. In Nelson's syndrome, there is increased level of blood ACTH but no increase incortisol.
Ans. All of the above are true
  1. Side effects of steroid due to prolonged administration are
    1. Progressive multifocal encephalopathy
    2. Hyperkalemia
    3. Spinal cord compression from spinal epidural lipomatosis
    4. Hyperosmolar nonketotic coma
    5. Avascular necrosis of the rib or other bone or vertebral compression fracture
Ans. All of the above are true
  • Possible side effect are the following:
  • Cardiovascular and renal include hypertension and Na and H2O retention.and hypokalemic alkalosis
  • CNS- Progressive multifocal leukoencephalopathy (PML), Steroid psychosis, Spinal cord compression from spinal epidural lipomatosis. Pseudotumor cereberi
  • Endocrine- growth supression in children, secondary, amenorrhoea, risk of adrenal insufficiency with steroid withdrawal, cushingoid feature with prolonged usage
  • GI (risk) on with steroid therapy > 3 weeks’ duration and regimen of prednisolone >400– 1000 mg/ day. Gastritis, pancreatitis, intestinal or sigmoid diverticular perforation
  • Inhibition of fibroblast
  • Subcutaneous atrophy, Posterior subcapsular cataract, Glaucoma, Hyperlipidemia
  • Imparied wound healing
  1. In Addisonian crisis, there is
    1. Hyponatremia
    2. Hypokalemia
    3. Hyperglycemia
    4. Hypothermia
    5. For emergency start methyl prednisolone
Ans. A.
  • In addisonian crisis there is postural hypotension or shock, hyponatremia, Hyperkalemia, hypoglycemia, hyperthermia, as high as 105°F245
  • For emergency treatment Methyl prednisolone is not recommended
  • For “gluco corticoid” emergency use hydrocortisone sodium succinate and cortisone acetate. For mineralocorticoid emergency use deoxycorticosterone and fludicortisone
  1. In Schwartz Bartter syndrome there is
    1. Serumosmolality < 280mosm/L
    2. Serum Na+ < 13 meq/L
    3. Ratio of urine : Plasma osmolality [1.5 – 2.5:1]
    4. Fluid restriction should be done
    5. Normal adrenal or Normal renal function and thyroid function
Ans. All of the above are true
  • Schwartz Bartter syndrome is another name for (SIADH). Urine sodium -> 20meq
  1. Following are associated with major depletion of K+
    1. Steroid therapy
    2. Addison's disease
    3. Insulin
    4. Renal tubular acidosis
    5. SIADH
Ans. A, C and D
  • Prolonged steroid therapy causes hypokalemia there is hyperkalemia in addison's disease. Insulin shift the K+ inside the cell. Type I – III Renal tubular Acidosis result in Hypokalemia but Type IV Renal Tubular Acidosis in hyperkalemia. SIADH result in hyponatremia
  1. A Lindegaard ratio Vmca / V ica consistent with vasospasm is
    1. greater than 1
    2. greater than 2
    3. greater than 3
    4. None of the above
Ans. C i.e. Ratio greater than 3 is consistent with vasospasm.
  1. Which of the following Low Molecular Weight Heparin is primarily responsible for spinal epidural hematomas in patient undergoing spinal / epidural anaesthesia
    1. Dalteparin
    2. Ardeparin246
    3. Logiparin
    4. Enoxaparin
Ans. D.
  • [Primarily Enoxaparin]
  1. Lab findings in DIC are
    1. FDP > 16ug/ml (Most common abnormality)
    2. Fibrinogen < 100 ug/dl
    3. PT > 16 sec
    4. PTT > 50 sec
    5. Platelet < 50,000 (Most uncommon abnormality)
Ans. All of the above are true
  • FDP (Fibrin degradation product) – Normal 1–8 ug/ml Borderline 8–16 ug/ml. Some labs require >40 ug/ml for diagnosis of DIC
  • Fibrinogen- < 100 ug/ml (some use 130)
  • In chronic DIC, PT and APTT may be normal, platelet and fibrinogen is low, Fibrin split product elevated
  • Pseudo – DIC – FDP increased, Normal Fibrinogen seen in Liver failure
  1. Brain tumor responsible for highest risk for DVT and Pulmonary Embolism include
    1. Meningioma
    2. Malignant glioma
    3. Metastasis
    4. Medulloblastoma
Ans. A
  • Brain tumor autopay prevalence of DVT is 28% and that of Pulmonary Embolism is 8.4% by using 125 I fibrinogen. Meningioma (72%), malignant glioma (60%), metastasis (20%) – Risk is decreased by using aspirin pre operatively
  1. About deep vein thrombosis
    1. Clinical diagnosis of DVT is very reliable
    2. Risk of cmbolism is greater with Calf DVT than proximal DVT
    3. Doppler ultrasound with B mode imaging is 95% sensitive and 99% specific both for proximal and calf DVT
    4. Impedance plathysmography is more sensitive for calf DVT than proximal DVT
    5. Radiolabled 125 I fibrinogen is better for calf DVT than proximal DVT247
Ans. E.
  • The clinical diagnosis of DVT is very unreliable. A patient with the classic signs of hot, swollen and tender calf or a positive homanssign will have a DVT only 20% – 50% of the time. 50% – 60% of patients with DVT will not have these findings
  • Reported mortality from DVT in LE is from 9% – 50% DVT limited to the calf has a low threat (< 1%) of embolization but in proximal deep veins it is [40% – 50%]
  • Doppler ultrasound with B mode and impedance plethymsagrophy both are less effective for calf DVT
  1. About pulmonary embolism
    1. Post-op PE generally occur 10–14 days following surgery and the reported incidence vary from 0.4–5%
    2. The most frequent finding is sudden dyspnea and the triad of haemoptysis, pleuritic chest pan, dyspnea is rare
    3. A negative D-dimer test reliably exclude PE in patient with a low clinical probability of PE or in those with nondiagnostic VQ seen
    4. In ECG “classic” S, Q3, T3 is rare and usually nonspecific ST and T changes occur
    5. Chest X-ray is normal in 25 – 30% and the use of heparin shortly after surgery and in patient with brain tumors is controversial and vena caval interruption may be a consideration (e.g. Greenfield filter)
Ans. All of the above are true
  1. Which of the following may induce seizures?
    1. Phenothiazines
    2. Methohexital
    3. Enflurane
    4. Metrizamide
    5. Thiopental
Ans. A to D
  1. About pediatric intramedullary spinal cord tumors following statements are
    1. Accounts for 4–6% of all CNS tumors
    2. Equal male and female distribution248
    3. Ependymomas are the largest group of pediatric intramedullary spinal cord tumor
    4. Astrocytomas are mostly located in the thoracic region
    5. Gross total resection cannot be achieved in most ependymonas
Ans. A, B and D
  • Astrocytomas are the largest group approximately 60% of pediatric intramedullary spinal cord tumors. They are typically large and mostly located in the thoracic region. In children the most common intramedullary spinal cord tumor is the pilocytic astrocytoma, which is a well-circumscribed and often cystic tumor with a indolent course. Spinal cord ependymoma occur commonly in the cervical region. The treatment of choice for intramedullary tumors ependymomas and astrocytoma is surgical resection. Gross total resection can be achieved in most ependymomas and is likely to result in cure
  1. About cervical vertebrae all are true EXCEPT
    1. Vertebrae prominens is C6
    2. Cuneiform vertebrae is another name for hemivertebrae
    3. Axis vertebrae has 5 primary and 2 secondary ossification centre
    4. Vertebral artery do not traverse the foramen transversium of C-7
Ans. A.
  • [Vertebrae prominens is C-7]
  1. About Multisystem atrophy or shy dragger syndrome is
    1. Autonomic substrate present
    2. Idiopathic orthostatic hypotension
    3. IPA (Idiopathic paralysis agitans)
    4. Loss of IML nucleus
    5. No response to L-dopa
Ans. All of the above are true
  • Striatonigral destruction
  1. About Myasthenia gravis
    1. It is due to auto antibodies against Muscarinic Ach Receptor
    2. In 70% of cases there is Thymoma249
    3. Edrophonium or Tensilon test can be used for diagnosing for patient with limb or respiratory muscle weakness
    4. During myasthenic crisis tt is O2 inhalation and anticholinesterases
    5. Aetiology of congenital MG is same as of adult form
Ans. All of the above are false
  • Auto antibodies are present against Nicotinic Acetylcholine receptor
  • In 15% there is thymoma and 70% thymic hyperplasia is present
  • Anti striated muscle antibody present in case of thymoma
  • Edrophonium or Tensilon is very short acting so it is used in testing for ocular or cranial nerve weakness. Neostigmine is used in patient with limb or respiratory muscle weakness
  • During Myasthenic crisis anticholinesterases are withdrawn and O2 inhalation is given after only the patient is able to walk without assistance then the anticholinesterases are added
  • Congenital Myasthenia Gravis is due to abnormal structure of the receptor. NA chR antibodies are not present. Thymectomy is contraindicated
  1. Following are the examples of involvement of basal ganglia
    1. Kernicterus
    2. Status marmoratus [double athetosis]
    3. Carbon monoxide induced hypoxia
    4. Methanol toxicity
    5. Wilson's disease
Ans. All of the above are true
  • In neonates, the BBB is not developed so that billirubin is deposited in globuspallidus
  • Hypoxia in perinatal period result in destruction of striatum. Again there is myelination resulting in marble like appearance hence called status marmoratus or double athetosis
  • Carbon monoxide induced hypoxia causes bilateral cavitation or destruction of globus pallidus
  • Methanol causes destruction of bilateral putamen.
  • Wilson's disease causes destruction of striatum250
  1. Basal ganglia includes
    1. Amygadla
    2. Claustrum
    3. Subthalamus
    4. Substantia Nigra
    5. Globus Pallidus
Ans. A, B and E
  • Subthalamus and substantia nigra are included in striatal motor system complex
  1. About Neurotransmitters in genesis of sleep
    1. NREM sleep is produced by stoppage of serotonergic neurons of raphe nuclei
    2. Discharge of nor-ephinephrine from neurons in pontine reticular formation produce REM sleep
    3. Ponto Geniculo Occipital spikes due to discharge of cholinergic neurons is necessary for shifting NREM to REM sleep
    4. NREM sleep to waking is a reversible process with 5 HT acting as a neurotransmitter
    5. Waking to REM sleep is never possible except in narcolepsy
Ans. All of the above are true
  1. Match the following
    1. Hepatic encephalopathy – Alzheimer type II astrocytes
    2. Neurosyphillis – red cells (Microglia)
    3. Shy dragger syndrome or multi system atrophy – Lewy bodies
    4. Pilocytic astrocytoma (cerebellar) – Rosenthal fibres
    5. Alzheimer's disease – Hirano bodies
Ans. A, B, D and E. C is false
  • In Shy dragger syndrome, there is loss of striatonigral fibres and intermedialateral nucleus column. There is no lewy body and in contrast to IPA there is no response to L-dopa
  1. About Parinaud's syndrome
    1. Due to compression from upward at inferior colliculus level
    2. Loss of downward gaze
    3. Lid retraction251
    4. Mostly due to pineal tumors
    5. Nystagmus retractorius
Ans. C, D and E
  • Parinaud syndrome is due to compression by a mass on quadrigeminal plate or midbrain tectum at superior colliculus level. Thus leading to loss of upward gaze first, there is lid retraction, light near dissociation and after that loss of convergence accomodation when there is loss of downward gaze also than it is called syndrome of sylvian aqueduc
  1. About Jugular foramen syndrome
    1. Villaret syndrome is caused by an intracranial lesion
    2. Vernet syndrome is posterior retropharyngeal syndrome
    3. In Collet sicard syndrome IX, X, XI, XII is involved
    4. Horner's syndrome is present only in villaret syndrome
    5. X nerve involvement is present in all Jugular Foramen syndrome
Ans. C, D and E.
  • Jugular foramen syndromes include
  • Vernet syndrome caused due to an intracranial lesion and involve IX, X, XI nerves
  • Colletsicard syndrome involve IX, X, XI, XII nerves
  • Villaret syndrome also known as Posterior retropharyngeal syndrome involve IX, X, XI, XII, sympathetic plexus
  • Tapia include X, XII, XI sympathetic plexus
  • Jackson include X, XI, XII
  1. About Paraneoplastic syndrome
    1. The most common paraneoplastic syndrome is type II B muscle atrophy
    2. Pan cerebellar degeneration is due to anti-yo antibodies present in ca breast, ca uterus or ovary
    3. Pure motor neuropathy is almost always due to Lymphoma usually (Hodgkins)
    4. Opsoclonus myoclonus syndrome usually in paeds indicate neuroblastoma
    5. In Lambert Eaton syndrome, 66% of patients will have ca that too most commonly oat cell Ca having anti Ca voltage gated channel antibodies
Ans. A to E.252
  1. Most patients with intrinsic brain stem gliomas initially present with
    1. Cranial neuropathies
    2. Headache
    3. Hydrocephalus
    4. Nausea and vomiting
Ans. A
  • Initial symptoms with brain stem glioma are cranial nerve palsies (68%). Headache used to occulate
  1. Surgical approach for thoracic disc herniation is associated with maximal neurologic injury is
    1. Costoransversectomy
    2. Transpedicular
    3. Transthoracic
    4. Midline laminectomy
Ans. D.
  • Morbidity with laminectomy for thoracic disc herniation is around 59%
  1. Which approach favoured for a patient with an 6 mm acoustic neuroma in which hearing preservation is a goal?
    1. Middle forsa approach
    2. Suboccipital
    3. Translabyrinthine
    4. None of the above
Ans. A.
  1. Uncinate seizures typically produce
    1. Auditory hallucination
    2. Gustatory hallucination
    3. Olfactory hallucination
    4. Vertiginious sensations
Ans. C.
  1. The ossification centers of the odontoid consist of
    1. One primary and two secondary centres
    2. One secondary and three primary centres
    3. Two primary centres
    4. Two primary and one secondary centre
Ans. D253
  • The secondary center is apical and the primary center lie inferiorly on either side of midline
  1. The most common single suture synostosis is
    1. Coronal
    2. Lambdoid
    3. Metopic
    4. Sagittal
Ans. D
  • Sagittal: Most common in male (scaphocephaly or dolicocephaly)
  1. Each of the following is true of basilar impression EXCEPT
    1. Cerebellar and vestibular complaint more common than motor and sensory complaint
    2. McGregor's line is helpful in routine screening
    3. McRac's line is most specific measurement
    4. Short neck and torticollis are common
Ans. A
  1. Following fracture has the poorest prognosis for healing without surgical intervention
    1. Hangman's
    2. Jefferson fracture
    3. Type I adontoid
    4. Type II adontoid
Ans. D.
  1. Statements regarding fracture of cervical spine along with its mechanism, all are true EXCEPT
    1. In Jefferson's fracture, the force is compresing and neck neutral
    2. In Burst fracture, the force is compresing and neck flexed
    3. In Hangman's fracture, the force is distracting and neck extended
    4. In Tear drop fracture, the force is compresing and neck posture flexed
Ans. B
  • Force should be compressing and neck neutral position254
  1. Lateral recess syndrome is most commonly caused by
    1. Hypertrophied pedicles
    2. Inferior articular facet hypertrophy
    3. Ligamentum flavum hypertrophy
    4. Superior articular facet hypertrophy
Ans. D
  1. Lateral recess syndrome is most common at which level
    1. L3 – L4
    2. L4 – L5
    3. L5 – S1
    4. None of the above
Ans. B.
  • L4 – L5 level
  1. Trilateral retinoblastoma describes bilateral ocular retinoblastomas and
    1. Astrocytoma
    2. Medulloblastoma
    3. Neurofibroma
    4. Pinealoblastoma
Ans. D.
  1. Adherence of posterior communicating artery aneurysm to the temporal lobe is most likely in a patient that presents with
    1. Loss of consciousness
    2. Seizures
    3. Third nerve involvement
    4. No third nerve invovement
Ans. D
  • No third nerve invovelemt
  1. The articular facet joint in the upper thoracic and lower thoracic region are oriented
    1. Axially and coronally respectively
    2. Coronally and sagittally
    3. Sagittally and coronally
    4. Sagittally and axially
Ans. B.
  • Coronal orientation of the facet in upper thoracic region leads to significant resistance to anterior transation but little to rotation in the lower thoracic spine the facet become sagittally oriented and less resistance to anterior translation255
  1. The single most important factor in the recurrence of meningiomas is
    1. Age of the patient
    2. Bone invasion
    3. Postoperative tumor residual
    4. Atypical histology
Ans. C.
  • Postoperative tumor residual
  1. The transverse crest separates the
    1. Cochlear and inferior vestibular nerves from the facial and superior vestibular nerves
    2. Facial and cochlear nerves from the superior and inferior vestibular nerves
    3. Cochlear and inferior vestibular nerves from the facial and superior vestibular nerves
    4. None of the above
Ans. C.
  • Bill's bar is a vertical bar that separate facial from superior vestibular nerve. Transverse crest is also known as crista falciformis.
  1. Trigonocephaly result from the premature closure of the
    1. Coronal suture bilaterally
    2. Coronal suture unilaterlly
    3. Lamboid sutures
    4. Metopic sutures
Ans. D.
  • Metopic sutures
  1. Most important factor determining aggressive clinical course of a dural AVM are
    1. Durations of symptoms
    2. Location
    3. Size
    4. Leptomeningeal venous drainage
Ans. D.256
  1. The Least-common location of intracranial menengiomas is
    1. Olfactory grove
    2. Intraventricular
    3. Posterior forsa
    4. Sphenoid Ridge
Ans. B.
  1. Cerebral salt wasting and SIADH may be distinguished by
    1. Serum osmolality
    2. Serum sodium
    3. Urine sodium
    4. Volume status
Ans. D.
  • Volume status hypervolemia in SIADH, and hypovolemia in CSW syndrome
  1. Rosenthal fibers are associated with all except
    1. Reactive gliosis
    2. Alexander's disease
    3. Pilocytic artrocytoma
    4. Pick's disease
Ans. D.
  • Pick's disease
  1. Which vascular malformation has no intervening brain parenchyma between blood vessels?
    1. Capillary telangiectasias
    2. Cavernous hemangiomas
    3. Cryptic arteriovenous malformation
    4. Venous angiomas
Ans. D.
  1. Each of the following is a result of the use of PEEP in the ventilated patient EXCEPT
    1. Decreased cerebral perfusion pressure
    2. Decreased physiologic dead space
    3. Decreased work of breathing
    4. Improve lung compliance
Ans. B.257
  1. The most common sites of hypertensive hemorrhage, in decreasing order of frequency are
    1. Lobar, putamen, cerebellum, thalamus, pons
    2. Putamen, lobar, thalamus, cerebellum, pons
    3. Putamen, thalamus, pons, lobar, cerebellum
    4. Thalamus, lobar putamen, cerebellum pons
Ans. B.
  1. The most sensitive monitor of venous air embolism is
    1. Transoesephageal echocardiography
    2. Precordial doppler
    3. End-tidalp CO2
    4. End-tidal pN2
Ans. A.
  • TEE > Precordial Doppler > EpCO2 > EpN2
  1. The pterion corresponds to the following EXCEPT
    1. Anterior pole of insula
    2. Middle cerebral artery
    3. Transverse sinus
    4. Lateral cerebral sulcus
Ans. C.
  • Transverse sinus
  1. Sigmoid – Transverse sinus junction corresponds to
    1. Pterion
    2. Asterion
    3. Bregma
    4. Stephanion
Ans. B.
  1. The approximate time for S phase in the biological cell cycle is
    1. 2–10 hours
    2. 1 hour
    3. 0–30 hours
    4. 10–20 hours
Ans. D.258
  1. Multiple organ failure includes the failure of the following organs except
    1. Lung
    2. GI tract
    3. Kidneys
    4. Pancreas
Ans. D.
  1. Betz cells are present in
    1. Area 6, Anterior paracentral lobule
    2. Area 4, Posterior paracentral lobule
    3. Area 4, Area 6, Anterior Paracentral lobule
    4. Area 4, Anterior paracentral lobule
Ans. D.
  1. Broca aphasia may result from occlusion of which of the following arteries
    1. Anterior temporal artery
    2. Anterior choroidal artery
    3. Operculofontal artery
    4. Medial striate artery of heubner
Ans. C.
  1. Which of the following groups of cranial nerves is closely related to the corticospinal tract?
    1. CN III, CN IV, and CN V
    2. CN III, CN V, and CN VII
    3. CN III, CN VI and CN VIII
    4. CN III, CN VI and CN XII
Ans.D.
  1. Neurologic examination reveals paralysis of upward and down ward gaze, absence of convergence and absence of papillary reaction to light the lesion site responsible is the
    1. Rostral midbrain tectum
    2. Caudal midbrain tectum
    3. Rostral pontine segment
    4. Caudal midbrain segment
Ans. A.259
  1. Paramedian infraction of the base of the pons involves which of the following structure
    1. Trapezoid body
    2. Descending trigeminal tract
    3. Rubrospinal tract
    4. Pyramidal tract
Ans. D.
  1. Statements concerning INO (Internuclear Ophthalmoplegia) are correct EXCEPT
    1. Result from a lesion in the dorsal pontine tegmentum
    2. It has no effect on convergence
    3. It is frequently seen in multiple scerosis
    4. It results in a lateral rectus palsy on attempted lateral conjugate gaze
Ans. D.
  • It results in a medial rectus palsy on attempted lateral conjugate gaze
  1. Most common anomaly of odontoid maldevelopment is
    1. Type 1
    2. Type 2
    3. Type 3
    4. Type 4
Ans. B.
  • Type 2 (ossiculum terminale) when there is failure of fusion of apex to base. Type 1 (osodontoiedum) when the fully developed dens does not unite with the body of the axis
  1. Statements concerning the dura mater are correct except
    1. It forms the perioesteum of the vertebral canal
    2. It forms the wall of the venous sinuses
    3. It forms the roof of the pituitary fosa
    4. It is innervated by two cranial nerves
Ans. A.
  1. The criterion for tonsillar ectopia in the fourth to the eighth decades is
    1. 6 mm
    2. 5 mm
    3. 4 mm
    4. D. 3mm260
Ans. C. i.e 4 mm
  • In general, the cerebellar tonsils ascend with increasing age. In the first decade of life, 6mm should be used as the criterion for tonsillar ectopia. This decreases to 5mm in the second and the third decades, to 4mm between the fourth to the eighth decades, and to 3mm by the ninth decade.
  1. Which of the following ocular muscle not originating from annulus of zinn
    1. Superior oblique
    2. Inferior oblique
    3. Superior rectus
    4. Inferior rectus
Ans. B.
  1. During operating a fourth ventricular tumor, facial colliculus get damaged resulting in paralysis of all EXCEPT
    A.
    Buccinator
    B.
    Lateral rectus
    C.
    Lateral pterygoid
    D.
    Orbicularis oculi
Ans. C. Facial colliculus is the elevation seen in the floor of the fourth ventricle which overlies abducens nucleus and the fibres of the facial nerve
  1. The middle cerebral artery irrigates each of the following structures or areas EXCEPT
    1. The paracentral lobule
    2. The inferior parietal lobule
    3. Brocas speech area
    4. Wernicke's speech area
Ans. A.
  1. Statements regarding the pupillary light pathway are correct EXCEPT
    1. Transection of the optic tract eliminates the direct pupillary light response
    2. Transection of the optic nerve would not eliminate the consensual pupillary light reflex
    3. Destruction of LGB would not interrupt the pupillary light pathway
    4. The efferent limb of the pupillary light reflex is the oculomotor nerve (CN III)
Ans. A.261
  1. The thalamus receives precortical sensory input from all of the following modalities EXCEPT
    A.
    General somatic sens
    B.
    Gustation
    C.
    Audition
    D.
    Olfaction
Ans. D.
  1. Epiconus syndrome involves segments
    A.
    S3 –C0
    B.
    L4 – S2
    C.
    L3 – C0
    D.
    L5 – S4
Ans. B.
  • Epiconus involve L4 – S2
  1. Conus medullaris syndrome involves segments
    A.
    L4 – S2
    B.
    L3 – C0
    C.
    S3 – C0
    D.
    L2 – S4
Ans. C.
  • S3 – C0 is conus
  1. During third ventriculostomy stoma is made in
    1. Roof of the third ventricle
    2. Anterior wall of the third ventricle
    3. Lateral wall
    4. Pre-mamillary membrane
Ans. D.
  • Pre-mamillary membrane is pierced
  1. About skull, statements are true except
    1. 90% of adult head size is achieved by age 1 year
    2. Skull growth essentially ceases at age 7 years
    3. Diploe spaces appear by 4th year and reach maximum by 25th year
    4. Mastoid process aerates till end of 2nd year
Ans. D.
  • Mastoid process aerate till 6th year
  1. Tumor involving posterior element of the vereterae are all except?
    1. Osteoid osteoma
    2. Osteoblastoma
    3. Eosinophilic granuloma
    4. Aneurysmal bone cyst262
Ans. C.
  • Primary tumor of posterior element are Aneurysmal bone cyst, Hydatid cyst, Hemangioma, Osteoblastoma, Osteoid osteoma and Giant cell tumor
  1. Statement about myelination—all are true except
    1. Occurs between 24 weeks’ gestation and 18 months
    2. Caudal to cranial in progression
    3. Motor nerve myelination occurs earlier than sensory nerve myelination
    4. Dorsal to ventral
    5. Premature infants myelinate at the same rate as in utero
Ans. C.
  1. Cystic metastases in brain is due to
    1. Squamous cell carcinoma of lung
    2. Malignant melanoma
    3. Choriocarcinoma
    4. Renal cell carcinoma
Ans. A.
  1. Resting membrane potential and action potential of a nerve fiber is
    A.
    −20 mV and +40 mV
    B.
    − 80 mV and +40 mV
    C.
    −60 mV and + 40 mV
    D.
    − 40 mV and +40 mV
Ans. B.
  • In resting unstimulated state, a nerve fiber is polarized so that the interior is negative to the exterior, potential difference is about −80 mV and is called resting membrane potential. The action potential is about +40 mV with the outside of the membrane negative to inside. In small diameter axons, the action potential may not rise to as much as 40 mV
  1. CSF is absorbed along the perineural lymphatics of cranial nerves
    A.
    I, III, VI, IX
    B.
    I, II, VII, VIII
    C.
    I, IV, VII, VIII
    D.
    I, III, VII, X
Ans. B.263
  1. Chance fracture is a
    1. Compression fracture
    2. Burst fracture
    3. Seat belt fracture
    4. Fracture dislocation
Ans. C.
  • Seat-belt fracture – failure of both posterior and middle column
  1. Presence of following genotype pretends a worse prognosis following traumatic brain injury
    A.
    ApoE allele
    B.
    ApoA allele
    C.
    ApoC allele
    D.
    ApoD allele
Ans. A.
  • Apolipoprotein E (ApoE) E4 allele also a risk factor for Alzheimer's disease as well as for chronic traumatic encephalopathy
  1. Chassaignac's tubercle also known as carotid tubercle is
    1. Posterior tubercle of transverse process of C5
    2. Anterior tubercle of transverse process of C6
    3. Anterior tubercle of transverse Process of C7
    4. Anterior tubercle of transverse process of C4
Ans. B.
  • Anterior tubercle of transverse process of C6 most prominent in C spine usually at the level of the cricoid cartilage approx 1.5–2 inches above clavicle.
  1. The intracranial length of the shunt to be inserted should be
    1. Approximately 1/3rd of the length of the skull
    2. Approximately 2/3rd of the length of the skull
    3. Approximately 1/4th of the length of the skull
    4. None of the above
Ans. B.
  • In adults without macrocrania, the inserted length is usually 12cm. In hydrocephalic infant usually 7–8 cm.
  1. Which of the following point place the shunt in Trigone?
    A.
    Dandy's point
    B.
    Kocher's point
    C.
    Keen's point
    D.
    Frazier burr hole
    264
Ans. C.
  • Keen's point – 2.5 or 3cm posterior and superior to pinna
  1. Safe extent of resection of temporal lobe on dominant side from temporal tip is
    A.
    3 – 4 cm
    B.
    4 – 5 cm
    C.
    5 – 6 cm
    D.
    6 – 7 cm
Ans. B.
  • One can usually safely resect up to 4 – 5 cm from temporal tip on dominant side. One can resect up to 6 – 7 cm in non-dominant hemisphere before running the risk of injury to optic radiation
  1. Bone wax for use in neurosurgery originated by
    A.
    Sir Victor Horsely
    B.
    Harvey cushing
    C.
    Walter Dandy
    D.
    Ambroise Pierre
Ans. A.
  • Sir Victor Horsely. Bone wax inhibit bone formation
  1. Which of the following are encapsulated receptors?
    1. Merkels disc
    2. Hair follicle receptor
    3. Ruffinis corpuscle
    4. Meissners corpuscle
    5. Neuromuscular spindle
Ans. C, D and E
  • Non capsulated reeceptors include free nerve ending, Merkels disc (found in hairless skin and are slowly adapting touch receptor), hair follicle receptor. Encapsulated receptors include Meissners corpuscle (very sensitive to touch and are rapidly adapting mechanoreceptors). It enables two point discrimination. Pacinian corpuscle is rapidly adapting mechanoreceptor sensitive to vibration. Ruffinis corpuscle is a slow adapting mechanoreceptor. Neuromuscular spindles and Neurotendinous spindles are encapsulated.
  1. Regarding statement about central nervous system all are false except
    1. There are roughly equal number of glial cells and neurons
    2. Microglia are phagocytic265
    3. Astrocytes regulate interstitial calcium concentration
    4. L-glucose is preferentially transported across the blood-brain barrier
Ans. B.
  • Astrocytes regulate interstitial potassium concentration. D-glucose is the isomer transported across the blood-brain barrier. There are many more glial cell than neurons
  1. While examining a patient there is papilledema differential diagnosis should include all EXCEPT
    1. Intra cranial tumor
    2. Schwanomma
    3. Gulliain – Barr syndrome
    4. Hypercapnea
Ans. D.
  • Hypercarbia causes papilledema. High CSF fluid protein level causes papilledema in GB syndrome and schwanomma
  1. True statement regarding optic fundus
    1. The optic disc is darker than the surrounding retina
    2. If the margin of the disc is indistinct, then papilledema is a possibility
    3. Arteriovenous nipping is a sign of diabetic retinopathy
    4. Pigmentation is always caused by a disease process
Ans. B.
  1. Destruction of the ventral horn results in all of the following deficit except
    1. Loss of muscle stretch reflex
    2. Loss of muscle bulk
    3. Flaccid Paralysis
    4. Babinski sign
Ans. D.
  1. Corticospinal tract receives contribution from all of the following areas except
    1. Prefrontal cortex
    2. Premotor cortex
    3. Motor cortex
    4. Somaesthetic cortex
Ans. A.266
  1. Spinothalamic tract arises from rexed laminae
    1. I, II, V
    2. I, IV, V
    3. I, IV, VII
    4. II, IV, VII
Ans. B.
  • From rexed laminae – I, IV, V
  1. Which of the following is superior anastomotic vein?
    1. Vein of Galen
    2. Vein of Labbe
    3. Vein of Trolard
    4. Pre central cerebellar Vein
Ans. C.
  • Inferior anastomotic vein is vein of Labbe
  1. Choroid plexus of the IVth ventricle is supplied by
    1. AICA
    2. PICA
    3. AICA and PICA
    4. None of the above
Ans. C.
  • Supplied by both. Vertical and proximal part by AICA and lateral half, plexus at CP angles by PICA. Choroid plexus of the fourth ventricle is T shaped with vertical limb being duplicate.
  1. Choroid plexus of the atrium or trigone supplied by
    1. Anterior choroidal artery
    2. Posterior choroidal artery
    3. Branches from AICA
    4. Branches from PICA
Ans. B.
  • Posterior choroidal artery from posterior cerebral artery supplies choroids plexus of the body. Choroid plexus of temporal horn supplied by anterior choroidal artery267
  1. Trendelenburg sign will be positive in which nerve injury
    1. Superior gluteal nerve
    2. Inferior gluteal nerve
    3. Common peroneal nerve
    4. Sciatic nerve
Ans. A.
  • Superior gluteal nerve that supply thigh adductor (gluteus medius and minimus)
  1. All of the following require urgent Neurosurgical evaluation and treatment EXCEPT
    1. Cerebellar infarction
    2. Subdural hematoma
    3. Sub arachnoid hemorrhage
    4. Embolic stroke
Ans. D.
  • Embolic stroke can be managed conservatively
  1. All of the following characterize Peripheral vertigo except
    1. Acute onset
    2. Non-fatiguable nystagmus
    3. Nausea and vomiting
    4. Positive Nylen – Baranay manoeuvre
Ans. B.
  • Nystagmus is fatiguable
  1. All of the following are potential cause for the comatose state of a patient present in coma in Emergency EXCEPT
    1. Hypoglycemia
    2. Hyperammonemia
    3. Hyponatremia
    4. Lacunar infarct
Ans. D.
  1. Which electrolyte is most affected by acid-base changes?
    A.
    Sodium
    B.
    Chloride
    C.
    Calcium
    D.
    Magnesium
    E.
    Potassium
Ans. E.
  • Potassium – A change in the PH of 0.10 results in a decrease or increase in K+ concentration in 0.5 mg/L increment268
  1. Chordomas are nervous system neoplasm that have all of the following characteristic except
    1. Occurrence in childhood
    2. Location in the sacrococcygeal region
    3. Characteristic physalipharous cells
    4. Slow growth
Ans. A.
  1. Astrocytomas of the cerebellum are characterized by
    1. Multiple recurrences
    2. Poor survival rates
    3. Transformation into glioblastoma
    4. Occurrence in childhood and adolscence
Ans. D.
  1. A pt received in emergency. CT scan showed SAH with acute blood intraventricularly and in the septum pellucidum probably the aneurysm to bleed is
    1. Anterior communicating artery aneurysm
    2. ICA bifurcation aneurysm
    3. M2 segment aneurysm
    4. Posterior communicating artery aneurysm
Ans. A.
  1. Following statements concerning the nuclear characteristic of tumors as determined by DNA flow cytometry are true except
    1. A diploid lesion can be benign or malignant
    2. An aneuploid tumor is a malignant tumor
    3. A malignant tumor can be diploid
    4. Reactive lesions should be diploid
Ans. B.
  • An aneuploid result does not equate with malignancy, most aneuploid lesions are neoplastic there are exceptions
  1. Following statement regarding central dopamine receptors are true except
    1. D1 receptor activation inhibit adenylate cyclase system
    2. D2 receptors are inhibitory in some brain tissue
    3. Neuroleptic side effects are thought to be mediated through D2 receptors in the pituitary
    4. D3 receptors found principally in limbic system269
Ans. A.
  • D1 receptor is excitatory and directly activates the adenylate cyclase system
  1. The revised trauma score is the most commonly used physiologic, estimates of injury used in trauma centers it is based on all of the following except which
    1. Systolic blood pressure
    2. Pulse rate
    3. Respiratory rate
    4. Glasgow coma scale
Ans. B.
  • Pulse rate – Revised Trauma Score based on GCS, systolic blood pressure and respiratory rate
  1. Nerve root existing between C4 – C5, C7 – D1, D8 – D9 and L4 – L5 vertebrae are
    1. C5, D1, D8 and L4 nerve roots respectively
    2. C5, C8, D8 and L4 nerve roots respectively
    3. C4, C8, D9 and L5 nerve roots respectively
    4. C5, D1, D9 and L4 nerve roots respectively
Ans. B.
  1. Most preferred approach for lumbar discectomy is
    1. Anterior
    2. Posterior
    3. Anterolateral
    4. Posterolateral
Ans. B.
  1. Most preferred surgical approach for pituitary tumors is
    1. Peritoneal
    2. Subfrontal
    3. Transcallosal
    4. Trans sphenoidal
Ans. D.
  1. About transcranial Doppler ultrasound all are true except
    1. Is becoming part of the standard care for sub-arachnoid hemorrhage270
    2. Useful in diagnosis and monitoring of cerebral vasospasm
    3. It measures the velocity of flowing blood
    4. Useful in detecting brain tumors
Ans. D.
  1. Victor Horsley's sign in a case of intracranial hemorrhage consist of
    1. Elevation of body temperature above 30°C
    2. Elevation of body temperature as high as 34°C
    3. Difference of one degree of temperature between the two sides of the body, higher on the paralysed side
    4. Difference of two degrees of temperature lower on the paralysed side
Ans. C.
  1. A seizure arising in one motor cortex starts most frequently in any of the following except
    1. Thumb
    2. Eyelid
    3. Angle of mouth
    4. Great toe
Ans. B.
  1. A patient with loss of function of posterior columns of spinal cord will present with all except
    1. Normal plantar response
    2. Romberg sign positive
    3. Partial loss of pain sensation
    4. Diminished vibration sense
Ans. C.
  1. Which type of seizure among the following does not follow head trauma
    1. Absence (petitmal)
    2. Partial complex, psychomotor variety
    3. Partial motor with Jacksonian march
    4. Partial motor with secondary generalization
Ans. A.271
  1. After lumbar puncture recumbency for which duration will prevent headache
    A.
    6 hours
    B.
    12 hours
    C.
    24 hours
    D.
    1 weeks
Ans. C.
  1. Characteristic of congenital hydrocephalus include all the following except
    1. Large head with wide and bulging fontanelle
    2. Transillumination
    3. Crackpot sign
    4. Convulsions
Ans. D.
  1. Following artery most frequently affected in embolic or thrombotic cerebrovascular disease
    1. Anterior cerebral
    2. Anterior communicating
    3. Middle cerebral
    4. Posterior cerebral
Ans. C.
  1. Following are reliable signs of upper motor neuron lesion except
    1. Babinski sign
    2. Positive Hoffman's sign
    3. Positive Rossolimos sign
    4. Generalized increase in tendon reflexes
Ans. D.
  1. CSF volume is maximum in
    1. Lateral ventricle
    2. Subarachnoid spaces centrally
    3. Spinal subarachnoid space
    4. IIIrd ventricle
Ans. C.
  • Total volume of CSF in adults is 100–150 ml, Lateral ventricles consist 10–15 ml each. Rest of ventricular system contain 5ml. Cranial subarachnoid spaces consist of about 25 ml of CSF. Spinal subarachnoid space consist of 75 ml of CSF.272
  1. In pseudopapilloedema, which of the following is the most diagnostic sign in fundus
    1. Exudates
    2. Hemorrhages
    3. Elevation of optic disc
    4. Unchanging appearance of fundus on repeated examination
Ans. D.
  1. If corticosteroids are used for cerebral decongestation which of the following should be monitored
    A.
    Blood pressure
    B.
    S. cortisol
    C.
    Urine output
    D.
    Serum potassium
Ans. D.
  1. In Intracerebral hemorrhage, lumber puncture will be negative for first
    A.
    8 – 10 hours
    B.
    12 – 16 hours
    C.
    16 – 24 hours
    D.
    24 – 48 hours
Ans. A. i.e. 8–10 hours
  1. The venous thrombosis is most frequent during pregnancy pueperium in
    1. Superior sagittal sinus
    2. Inferior sagittal sinus
    3. Cortical venous system
    4. Transverse sinus
Ans. A.
  1. Which of the following is often the site of origin of Jacksonian epilepsy?
    1. Orbitofrontal area
    2. Frontal lobe
    3. Pre rolandic gyrus
    4. Post rolandic gyrus
Ans. C.
  1. Following symptoms indicate the extension of thrombosis from transverse sinus to superior sagittal sinus
    A.
    Faciobrachial paresis
    B.
    Hemiparesis
    C.
    Paraparesis
    D.
    Hemianopia
Ans. C.273
  1. Foetal circulation present in
    A.
    10 – 15%
    B.
    15 – 35 %
    C.
    30 – 45%
    D.
    25 – 45%
Ans. B.
  1. First branch of supraclinoid ICA is
    1. Ophthalmic artery
    2. Superior hypophyseal artery
    3. Inferior hypophysed artery
    4. Posterior communicating artery
Ans. B.
  • Superior hypophyseal artery supplies anterior lobe of pituitary and stalk
  1. PICA (Posterior inferior cerebellar artery) has
    1. 3 segments, 3 branches
    2. 4 Segments, 3 branches
    3. 5 segments, 4 branches
    4. 6 segments, 6 branches
Ans. B.
  • 4 segments – anterior medullary, lateral medullary, posterior medullary and supratonsillar
  • 3 branches – choroidal artery, tonsillohemispheric and inferior vermian artery
  1. Benign external hydrocephalus all are true except
    1. Ventricles are normal or minimally enlarged
    2. Does not resolve spontaneously
    3. Enlarged subarachnoid spaces over the frontal poles
    4. May be associated with plagiocephaly
Ans. B.
  • Usually resolve spontaneously by 2 years of age
  1. The most common cause of Communicating hydrocephalus
    1. Infections
    2. Post hemorrhagic
    3. Secondary to masses
    4. Neurosarcoidosis
Ans. A.
  • Post-hemorrhagic second most common274
  1. After Endoscopic third ventriculostomy the most common aneurysm tend to occur as a result of long term complication is
    1. AICA aneurysm
    2. PICA aneurysm
    3. SCA aneurysm
    4. Basilar artery aneurysm
Ans. D.
  1. Percent risk of seizures in the first year after placement of a shunt is
    1. 5.5 %
    2. 6.5%
    3. 8.5%
    4. 10.5%
Ans. A.
  1. The most common cause and site of shunt malfunction is
    1. Obstruction, Distal end
    2. Obstruction, Proximal end
    3. Disconnection, Distal end
    4. Disconnection, Proximal end
Ans. B.
  • The most common cause of shunt malfunction is obstruction and proximal ventricular catheter is the most common site
  1. Regimen of antibiotics for an operated patient of cerebral abscess which is true
    1. IV antibiotics for 6 weeks and oral antibiotics for 4 weeks
    2. IV antibiotics for 2 weeks and oral antibiotics for 2 weeks
    3. IV antibiotics for 1 week and oral antibiotics for 1 week
    4. IV antibiotics for 2 weeks only
Ans. A.
  • IV antibiotics for 6–8 weeks and oral antibiotics for 4–8 weeks. IV antibiotic for 6–8 weeks (most commonly 6) may then discontinue even if the CT abnormalities persist (neovascularity remains). Oral antibiotics may be used following IV course. 5–20% of abscesses recur within 6weeks of discontinuing antibiotics.275
  1. Which of the following Midline brain cavities is associated with congenital anomaly?
    1. Cavum septum pellucidum
    2. Cavum vergae
    3. Cavum velum interpositum
    4. None of the above
Ans. B.
  • Cavum vergae is associated with apert syndrome
  1. The most common posterior circulation aneurysm is
    1. Basilar top aneurysm
    2. Basilar – SCA aneursym
    3. Basilar – AICA aneurysm
    4. Verterbral – PICA aneurysm
Ans. A.
  • Account for 5% of all intracranial aneurysm
  1. Loss of chromosome arms in patient of oligodendroglioma resulting in good response to chemotherapy and longer free tumor survival
    1. 1P, 19Q
    2. 2P, 19Q
    3. 5Q, 22Q
    4. 3P, 18Q
Ans. A.
  1. Spondylolysis is an alternative term for
    1. Isthmic spondylolisthesis
    2. Traumatic spondylolisthesis
    3. Degenerative spondylolisthesis
    4. Congenital spondylolisthesis
Ans. A.
  • Isthmic spondylolisthesis defect in pars inter articularis
  1. Intradural disc herniation has an incidence of about
    A.
    .01 – 0.08%
    B.
    .04 – 0.09%
    C.
    .04 – 1.1%
    D.
    .08 – 1.8%
Ans. C.276
  1. H reflex which measures sensory conduction through nerve roots used mostly to assess
    1. L4 radiculopathy
    2. L5 radiculopathy
    3. S1 radiculopathy
    4. L3 radiolopathy
Ans. C.
  1. About SCIWORA all are true EXCEPT
    1. Age range of children is 1.5 – 16 years
    2. Attributed to increased elasticity of spinous ligament
    3. Higher incidence in age >9 years
    4. Increased risk with asymptomatic Chiari 1 malformation
Ans. C. Higher incidence in age < 9 years. SCIWORA means spinal cord injury without obvious radiographic abnormality
  1. The precentral cerebellar vein usually drains into the
    1. Internal cerebral vein
    2. Lateral mesencephalic vein
    3. Straight sinus
    4. Vein of Galen
Ans. D.
  1. Anterior temporal lobe masses characteristically displace the
    1. Anterior choroidal artery laterally
    2. Anterior choroidal artery medially
    3. Anterior choroidal artery upward
    4. Posterior choroidal artery downward
Ans. B
  1. The most common site of origin of the recurrent artery of Heubner is the
    1. A1 segment
    2. A2 segment
    3. Internal carotid artery
    4. M1 segment
Ans.B277
  1. Most common site for occurrence of Choroid plexus papilloma in children and adults respectively
    1. Right atrium and Fourth ventricle
    2. Left atrium and Third ventricle
    3. Left atrium and Fourth ventricle
    4. Right atrium and Third ventricle
Ans.C.
  1. Lesions in diffuse axonal injury are commonly found in the following EXCEPT
    1. Corpus callosum
    2. Gray-White junction
    3. Rostral brain stem
    4. Temporal lobe
    5. Superior cerebellar peduncle
Ans. A, B, C, and E
  1. Acute subarachnoid hemorrhage is more difficult to diagnose on MRI than on CT because
    1. Extracellular methemoglobin is isointense on T1 and T2
    2. Hemosiderin is isointense on T1 and T2
    3. The high oxygen tension in the subarachnoid space prevents conversion of oxyhemoglobin to deoxyhemoglobin
    4. The low oxygen in the subarachnoid space prevents conversion of deoxyhemoglobin to oxyhemoglobin
Ans. C.
  1. Which of the following is true of myelination?
    1. It has no effect on transmembrane resistance, but increases membrane capacitance
    2. It decreases both transmembrane resistance and membrane capacitance
    3. It decreases transmembrane resistance and increases membrane capacitance
    4. It increases transmembrane resistance and decreases membrane capacitance
Ans. D.278
  1. Testing the sense of taste would be most important in patient presenting with
    1. Loss of hearing
    2. Diplopia
    3. Dystaxia
    4. Hemifacial paralysis
Ans. D.
  1. The posterior plane of cutaneous innervation by the trigeminal nerve is at the
    1. Occipito cervical junction
    2. Lambdoid suture plane
    3. Interaural plane
    4. Intercanthal line
Ans. C.
  1. The descending root of CNV ends at the
    1. Obex
    2. Mid-medullary level
    3. Pontomedullary junction
    4. Upper cervical level
Ans. D.
  1. In locked in syndrome, the lesion usually involves the
    1. Midbrain tectum
    2. Midbrain tegmentum
    3. Pontine tegmentum
    4. Basis pontis
Ans. D.
  • Basis ponits – complete paralysis of all voluntary movement except vertical eye movement, Pt retain consciousness
  1. Disorders resulting due to defect in secondary neurulation are
    1. Tethered cord syndrome
    2. Meningocele
    3. Encephalocele
    4. Lipomeningocele
    5. Chiari malformations
    279
Ans. A, B and D
  • The formation of the brain and spinal cord is referred to as a dorsal induction. The two general stages of dorsal induction are primary and secondary neurulation. Primary neurulation involves the formation of the brain and upper spine. Secondary neurulation refers to formation of the distal spine. Disorders of primary neurulation are mostly neural tube closure defects and early central nervous system anomalies occurring at around 3 or 4 weeks gestational weeks. These include chiari malformations, cephaloceles, and myelomeningoceles. During secondary neurulation, interaction between the notochord and mesoderm form the skull, dura, pia and vertebrae. These occur at 4–5 gestational weeks. Abnormalities of secondary neurulation result in spinal dysraphic disorders that range from simple, isolated anomalies such as spina bifida occulta to more complex malformations such as meningocele and lipomeningocele, neurentric cysts, dermal sinus and the caudal regression syndrome
  1. The dececrebrate posture generally indicates a lesion of the
    1. Pons
    2. Medulla
    3. Midbrain
    4. Diencephalon
Ans. C.
  1. Dystaxia resulting from lesions of the cerebellar hemisphere reflects dysmodulation of impulses relayed through which pathway
    1. Dentato vestibulospinal
    2. Olivo cerebellar
    3. Dentatorubral
    4. Dentatothalamo cerebro cortical
Ans. D.
  1. Part of the cerebellum that may undergo transforminal herniation and strangle the medullocervical junction is the
    1. Rosteral vermis
    2. Flocculo nodular lobe
    3. Inferior peduncle
    4. Tonsil
Ans. D.280
  1. A lesion affecting CN III, CN IV and CN VI would most likely to be in the
    1. Pontine tegmentum
    2. Superior orbital fissure
    3. Cavernous sinus
    4. Midbrain tegmentum
Ans. C.
  1. After CN III enters the subarachnoid space of the interpenduncular fossa, it immediately encounters
    1. CN IV
    2. Infundibular stalk
    3. The posterior cerebral artery
    4. The posterior clinoid process
Ans. C.
  1. The development of the human embryo may be divided into 23 stages. At which stage of embryo formation the cranial defect use to occur
    A.
    Stage 8
    B.
    Stage 9
    C.
    Stage 11
    D.
    Stage 12
Ans. C.
  • The closure process leaves an opening rostrally, the anterior neuropore, which closes around the 24–25th day of gestation (stage 11). And an opening caudally, the posterior neuropore, which closes by the 27 to 30th day of gestation (stage 12). This explains why most congenital abnormalities are located in the cranial and caudal region.
  1. The most important structure for describing the location of various thalamic nuclei is the
    1. Nucleus pulvinaris
    2. Nucleus reticularis
    3. Stratum zonale
    4. Zona incerta
    5. Internal medullary lamina
Ans. E.281
  1. A single lesion that interrupts axons of the fornix, striaterminalis and stria medullaris would involve the
    1. Roof of the third ventricle
    2. Floor of the third ventricle
    3. Genu of the corpus callosum
    4. Midregion of the anterior commissure
Ans. D.
  • The fornix which arches over the roof of the third ventricle, splits into an anterior and a posterior component around the anterior commisure, the stria medullaris and the stria terminalis course across the midportion of anterior commissure
  1. In addition to field cuts, lesion that destroy the optic chiasm and immediately related overlying part of hypothalamus would most likely cause
    1. Hypertension
    2. Precocious puberty
    3. Hypothermia
    4. Diabetes Insipidus
Ans. D.
  1. Main efferent layers of the cerebral cortex are
    A.
    Layer IV and VI
    B.
    Layer V and VI
    C.
    Layer I and II
    D.
    Layer III and IV
Ans. A.
  1. The sublenticular and retrolenticular parts of the internal capsule convey
    1. Thalamoparietal and Thalamogeniculate radiation
    2. Thalamoparietal and geniculocal carine radiations only
    3. Thalamofrontal and thalmoparietal radiation
    4. Geniculocalcarine, temporal and occipital radiation
Ans. D.
  1. The spinal cord reflects the Law of Bell and Magendie since
    1. The dorsal horn is sensory and the vertal horn is motor
    2. Spinal cord develops as intermediolateral cell column
    3. Corticospinal tracts run in the lateral column
    4. Ventral horn contains no interneuror
Ans. A.282
  1. The original somite-spinal nerve relationship is best shown at which level of the neuraxis
    A.
    Cervical
    B.
    Thoracic
    C.
    Lumbar
    D.
    Sacral
Ans. A.
  1. A lesion that would most effectively disconnect the frontal and temporal lobes would interrupt the
    1. Uncinale fasciculus
    2. Inferior longitudinal fasciculus
    3. Central segmental tract
    4. Fornix
Ans. A.
  1. The subdivisons of the CNS that lacks direct sensory connection with the external environment is the
    1. Telencephalon
    2. Diencephalon
    3. Midbrain
    4. Pons
Ans. C.
  1. During adduction of an eye, the elevator action of the superior rectus and inferior oblique
    1. Decrease simultaneously
    2. Increases simultaneously
    3. Decreases and increases simultaneously
    4. Increases and decreases simultaneously
Ans. C.
  • The superior recti rotate the eye ball superiorly as the primary action when the eyes are turned outward. Inferior oblique rotates the eye superiorly with eyes adducted.
  1. A patient's MRI shows right frontal space occupying lesion extending 4.5cm behind the coronal suture with minimal deficit. All are true except
    1. Anterior approach preferred to prevent damage to motor cortex
    2. Posterior approach will result in hemiplegia283
    3. After complete removal, the sensory deficitis are prominent-than motor deficits
    4. None of the above
Ans. C.
  1. Following form of aphasia would most likely to be associated with a right sided UMN facial paresis
    1. Auditory word agnosia
    2. Dyslexia plus dysgraphia
    3. Auditory word aphasia plus dyslexia (Fluent aphasia)
    4. Expressive aphasia, non fluent type
Ans. D. i.e. Broca's aphasia
  1. The afferent axons for the muscle stretch reflexes (MSRS) synapse on the
    1. Substantia gelatinosa
    2. Lamina I of Rexed
    3. Ventral motoneuron
    4. Nucleusreticularis
Ans. C.
  1. To eliminate pain in a particular dermatome without causing loss of the sense of touch would make a cut at the following site
    1. Where the lateral fibers of the dorsal root attach to the cord
    2. Where the medial fibres of the dorsal root attach to the cord
    3. In the mid portion of the root as it crosses the subarachnoid space
    4. In the nerve trunk about 1cm distal to the ganglion
Ans. A.
  1. The ultimate function of blood brain barrier is to
    1. Exclude small molecules
    2. Actively transport protein
    3. Keep out white blood cells
    4. Protect the polarization of the neuronal membrane
Ans. D.284
  1. The cellular neuropathology of Alzheimer's disease resembles most closely that seen in
    1. Huntigton disease
    2. Multi-infarct dementia
    3. Pick's disease
    4. Trisomy 21
Ans. D.
  1. The most common cause of cerebral embolism is
    1. Cardiac prosthetic valves
    2. Endocarditis
    3. Atrial fibrillation
    4. Dilated cardiomyopathy
Ans. C.
  1. Following are common causes of seizures in adults older than 50 years of age except
    1. Cerebrovascular disease
    2. Degenerative disease
    3. Subdural hematoma
    4. Mesial temporal sclecosis
Ans. D.
  1. Following are side effects of phenytoin except
    1. Ataxia
    2. Gum hyperplasia
    3. Leukopenia
    4. Lymphadenopathy
Ans. C.
  • Leukopenia more common with carbamezepine
  1. The most common presenting finding or symptom of multiple sclerosis is
    1. Internuclear opthalmoplegia
    2. Optic neuritis
    3. Transverse myelitis
    4. Cerebellar ataxia
Ans. B.285
  1. Types of cancer commonly metastasize to the central nervous system (CNS) except
    1. Ovarian
    2. Breast
    3. Melanoma
    4. Acute lymphoblastic leukemia
Ans. A.
  1. Arcuate eminence is present in the
    1. Squamous part of the temporal bone
    2. Pterous part of the temporal bone
    3. Body of sphenoid
    4. Greater wing of sphenoid
Ans. B.
  1. Vertical conjugate saccade controlled by
    1. Para pontine reticular formation
    2. Medial longitudinal fasciculus
    3. Rostral interstitial nucleus of MLF
    4. None of the above
Ans. C.
  1. What percent of the cardiac output and total body oxygen consumption is utilized by brain
    1. 20% of the cardiac output and 10% of total body oxygen consumption
    2. 20% of the cardiac output and 20% of total body oxygen consumption
    3. 30% of the cardiac output and 20% of total body oxygen consumption
    4. 20% of the cardiac output and 20% of total body oxygen consumption
Ans. B.
  • 20% of the cardiac output and 20% of total body oxygen consumption286
  1. Paralysis of tenth cranial nerve may be produced by thrombosis of artery
    1. Middle cerebral artery
    2. Posterior cerebral artery
    3. Posterior communicating artery
    4. Posterior inferior cerebellar
Ans. D.
  1. Among which of the following embolic material is not used for definitive (non operative) cure
    1. Arterial minicoils
    2. Cyanoacrylate adhesive
    3. Polyvinyl alcohol particles
    4. Silk suture pieces
Ans. C.
  • Polyvinyl alcohol particles. Most interventional neuro radiologist feel polyvinyl alcohol particle embolization should not be used for definitive (non operative) cure
  1. Kanofsky performance scale of a patient who cares for self but unable to carry on normal activities is
    1. 80
    2. 70
    3. 60
    4. 50
Ans. B.
  • 100 – Normal
  • 90 – Normal activity with effort, some moderate symptoms
  • 70 – Care for self but unable to carry on normal activities
  • 60 – Cares for most needs but require occasional assistance
  • 50 – Require considerable assistance to carry on activities of daily living
  1. The first removal of a ruptured lumbar intervertebral disc was by
    1. Mixter and Barr
    2. Victor Horsely
    3. Walker Dandy
    4. Harvey Cushing
Ans. A.287
  1. About Walter Dandy all are correct EXCEPT
    1. Developed the technique of pneumoencephalography
    2. First to show acoustic neuromas could be totally removed
    3. Introduce the technique of choroids plexectomy to reduce production of CSF
    4. Trained under William Halsted
Ans. D.
  • He was trained under Harvey William Cushing. William Halsted was the mentor of Harvey William Cushing. Harvey William Cushing is also known as father of American neurosurgery
  1. Which of the following are true about growing skull fractures
    1. Can cross suture line
    2. May be associated with underlying bone injury
    3. Dural defect is always greater than bone defect
    4. Most common in frontoparietal region
    5. Most common in the three five year old age group
Ans. A to D. Most common in less than 3 years of age
  1. The most common mechanism of translational C1 – C2 subluxation is
    A.
    Axial loading
    B.
    Distraction
    C.
    Extension
    D.
    Flexion
Ans. D.
  1. Nervus intermedius contain
    1. SVA + GVE fibres
    2. SVA + GSA fibres
    3. SVA fibres
    4. SVA + GSA + GVE fibres
Ans. D.
  1. The anterior choroidal artery supplies portion of each of the following structure EXCEPT
    1. Amygdala
    2. Globus pallidus
    3. Hippocampus
    4. Hypothalamus
Ans. D.288
  1. Most common cause of epidural metastases in childrenis
    1. Osteosarcoma
    2. Ewing's sarcoma
    3. Neuroblastoma
    4. Rhabdomyosarcoma
Ans. B. Approximately 3–5% of children with a systemic malignancy develop spinal cord compression. The responsible tumor in order of decreasing frequency are Ewing's sarcoma, osteosarcoma, neuroblastoma, rhapdomyosarcoma, non-Hodgkins lymphoma, germ cell tumor, Hodgkins lymphoma
  1. In a Peripheral nerve, sensory fibers are about
    A.
    20%
    B.
    30%
    C.
    40%
    D.
    50%
Ans. C.
  • 40% Unmyelinated to myelinated fibre ratio 4:1
  1. Position of head relative to orbitomeatal line for superior visualization of the brain stem and cerebellum in CT scan is
    1. Angled 15° relative to orbitomeatal line
    2. Angled 25° relative to orbitomeatal line
    3. Angled 35° relative to orbitomeatal line
    4. Angled 45° relative to orbitomectal line
Ans. A.